natres

78
I. Concept of Jura Regalia Natural Resources First Set_Case Digests Page | 1 I. CONCEPT OF JURA REGALIA 1. Collado vs CA (G.R. No. 107764, October 4, 2002) FACTS: Petitioner Edna T. Collado filed with the land registration court an application for registration of a parcel of land (“Lot”), situated in Antipolo Rizal. Attached to the application was a technical description, stating “this survey is inside IN-12 Mariquina Watershed.” The Solicitor General filed oppositions to the application. Petitioners (Edna Collado and her co-applicants) allege that they have occupied the Lot since time immemorial. Their possession has been open, public, notorious and in the concept of owners. They paid all real estate taxes and submitted evidence to prove that there have been 9 transfers of rights among them and their predecessors-in-interest. RTC ruled in favor of the petitioners for having presented sufficient evidence to establish registrable title over the property. ISSUE : (1) WON petitioners have registrable title over the Lot. NO. (2) Did petitioners acquire private rights over the parcel of land prior to the issuance of EO 33? NO. HELD: (1) Petitioners concede that the Lot is inside the literal description of Marikina Watershed Reservation (MWR). Their main claim over the Lot is that “all Presidential proclamations like the proclamation setting aside the MWR are subject to private rights.” EO 33 (which established the MWR) has a saving clause tha t the reservations are “subject to existing private rights, if any there be.” Under the Regalian Doctrine, all lands not otherwise appearing to be clearly within private ownership are presumed to belong to the State. The Spaniards first introduced the doctrine to the Philippines through the Laws of the Indies and the Royal Cedulas, specifically, Law 14, Title 12, Book 4 of the Novisima Recopilacion de Leyes de las Indias which laid the foundation that "all lands that were not acquired from the Government, either by purchase or by grant, belong to the public domain ." Upon the Spanish conquest of the Philippines, ownership of all "lands, territories and possessions" in the Philippines passed to the Spanish Crown. The Laws of the Indies were followed by the Ley Hipotecaria or the Mortgage Law of 1893. The Spanish Mortgage Law provided for the systematic registration of titles and deeds as well as possessory claims. The Royal Decree of 1894 or the "Maura Law" partly amended the Mortgage Law as well as the Law of the Indies. The Maura Law was the last Spanish land law promulgated in the Philippines. It required the "adjustment" or registration of all agricultural lands, otherwise the lands would revert to the state. Four years later, Spain ceded to the government of the United States all rights, interests and claims over the national territory of the Philippine Islands through the Treaty of Paris of December 10, 1898. In 1903, the United States colonial government, through the Philippine Commission, passed Act No. 926, the first Public Land Act, which was described as follows: "Act No. 926, the first Public Land Act, was passed in pursuance of the provisions of the Philippine Bill of 1902. The law governed the disposition of lands of the public domain. It prescribed rules and regulations for the homesteading, selling and leasing of portions of the public domain of the Philippine Islands, and prescribed the terms and conditions to enable persons to perfect their titles to public lands in the Islands. It also provided for the "issuance of patents to certain native settlers upon public lands," for the establishment of town sites and sale of lots therein, for the completion of imperfect titles, and for the cancellation or confirmation of Spanish concessions and grants in the Islands." In short, the Public Land Act operated on the assumption that title to public lands in the Name:

Upload: piarodriguez-es

Post on 08-Dec-2015

110 views

Category:

Documents


32 download

DESCRIPTION

natres

TRANSCRIPT

I. Concept of Jura Regalia Natural Resources First Set_Case Digests

Page | 1

I. CONCEPT OF JURA REGALIA

1. Collado vs CA

(G.R. No. 107764, October 4, 2002)

FACTS: Petitioner Edna T. Collado filed with the land registration court an application for registration of a parcel

of land (“Lot”), situated in Antipolo Rizal. Attached to the application was a technical description, stating “this survey is inside IN-12 Mariquina Watershed.” The Solicitor General filed oppositions to the application.

Petitioners (Edna Collado and her co-applicants) allege that they have occupied the Lot since time immemorial. Their possession has been open, public, notorious and in the concept of owners. They paid all real estate taxes and submitted evidence to prove that there have been 9 transfers of rights among them and their predecessors-in-interest.

RTC ruled in favor of the petitioners for having presented sufficient evidence to establish registrable title over the property. ISSUE: (1) WON petitioners have registrable title over the Lot. NO. (2) Did petitioners acquire private rights over the parcel of land prior to the issuance of EO 33? NO. HELD: (1) Petitioners concede that the Lot is inside the literal description of Marikina Watershed Reservation (MWR).

Their main claim over the Lot is that “all Presidential proclamations like the proclamation setting aside the MWR are subject to private rights.” EO 33 (which established the MWR) has a saving clause that the reservations are “subject to existing private rights, if any there be.”

Under the Regalian Doctrine, all lands not otherwise appearing to be clearly within private ownership are

presumed to belong to the State. The Spaniards first introduced the doctrine to the Philippines through the Laws of the Indies and the Royal Cedulas, specifically, Law 14, Title 12, Book 4 of the Novisima Recopilacion de Leyes de las Indias which laid the foundation that "all lands that were not acquired from the Government, either by purchase or by grant, belong to the public domain." Upon the Spanish conquest of the Philippines, ownership of all "lands, territories and possessions" in the Philippines passed to the Spanish Crown.

The Laws of the Indies were followed by the Ley Hipotecaria or the Mortgage Law of 1893. The Spanish Mortgage Law provided for the systematic registration of titles and deeds as well as possessory claims. The Royal Decree of 1894 or the "Maura Law" partly amended the Mortgage Law as well as the Law of the Indies. The Maura Law was the last Spanish land law promulgated in the Philippines. It required the "adjustment" or registration of all agricultural lands, otherwise the lands would revert to the state.

Four years later, Spain ceded to the government of the United States all rights, interests and claims over the national territory of the Philippine Islands through the Treaty of Paris of December 10, 1898. In 1903, the United States colonial government, through the Philippine Commission, passed Act No. 926, the first Public Land Act, which was described as follows:

"Act No. 926, the first Public Land Act, was passed in pursuance of the provisions of the Philippine Bill of 1902. The law governed the disposition of lands of the public domain. It prescribed rules and regulations for the homesteading, selling and leasing of portions of the public domain of the Philippine Islands, and prescribed the terms and conditions to enable persons to perfect their titles to public lands in the Islands. It also provided for the "issuance of patents to certain native settlers upon public lands," for the establishment of town sites and sale of lots therein, for the completion of imperfect titles, and for the cancellation or confirmation of Spanish concessions and grants in the Islands." In short, the Public Land Act operated on the assumption that title to public lands in the

Name:

I. Concept of Jura Regalia Natural Resources First Set_Case Digests

Page | 2

Philippine Islands remained in the government; and that the government’s title to public land sprung from the Treaty of Paris and other subsequent treaties between Spain and the United States. The term "public land" referred to all lands of the public domain whose title still remained in the government and are thrown open to private appropriation and settlement, and excluded the patrimonial property of the government and the friar lands."

Thus, it is plain error for petitioners to argue that under the Philippine Bill of 1902 and Public Land Act No. 926, mere possession by private individuals of lands creates the legal presumption that the lands are alienable and disposable.

Both the 1935 and 1973 Constitutions prohibited the alienation of all natural resources except agricultural lands of the public domain. The 1987 Constitution readopted this policy. Indeed, all lands of the public domain as well as all natural resources enumerated in the Philippine Constitution belong to the State.

Watershed Reservation is a Natural Resource: The term "natural resource" includes "not only timber, gas, oil coal, minerals, lakes, and submerged lands, but also, features which supply a human need and contribute to the health, welfare, and benefit of a community, and are essential to the well-being thereof and proper enjoyment of property devoted to park and recreational purposes." (2) An applicant must overcome the presumption that the land he is applying for is part of the public domain and

that he has an interest to warrant registration in his name arising from an imperfect title (may have been derived from old Spanish grants or titles). In the case at bar, petitioners were unable to acquire a valid and enforceable right or title because of the failure to complete the required period of possession (at least 30 years).

Assuming that the Lot was alienable and disposable land prior to the issuance of EO 33 in 1904, EO 33

reserved the Lot as a watershed. Since then, the Lot became non-disposable and inalienable public land. At the time petitioners filed their application on April 25, 1985, the Lot has been reserved as a watershed under EO 33 for 81 years prior to the filing of petitioners’ application.

NOTES:

2. Republic vs Dela Rosa

(160 SCRA 228, G.R. No. L-43938, April 15, 1988)

FACTS:

The case is about a parcel of land whose ownership is disputed by four parties: the dela Rosas, Benguet

Consolidated Inc (BCI), Atok Corp, and the Bureau of Forestry Development (BFD).

In 1965, Jose de la Rosa on his and on his three children’s behalf, applied to register a parcel of land

divided into 9 lots in Benguet. According to the children, they acquired the land by virtue of prescription. As

evidence they produced tax declarations and realty tax receipts.

Benguet Consolidated Inc (BCI) opposed their application, claiming that half the lots were covered by mineral claim sold to it in 1934. Since 1934, BCI had been in actual, continuous and exclusive possession of the land in concept of owner. As evidence BCI presented geological mappings, payment of taxes, and construction on the land. Atok Corp’s claim is similar to BCI, that a mineral claim covering the lots had been sold to it in 1931. Their evidence is similar to BCI: construction and tax payments.

I. Concept of Jura Regalia Natural Resources First Set_Case Digests

Page | 3

The BFD also objected, saying that the land was covered by the Central Cordillera Forest Reserve (CCFR) under Proc. No. 217, dated 1929. As it was forest land, it was not subject to alienation as stated in the 1935 and 1973 Constitutions.

In the end, the CA affirmed the BFD’s rights on the surface of the land, while reaffirming the rights to the underground to both BCI and Atok. In other words, the CA ruled that the surface was to be agricultural on the surface, and mineral underneath.

ISSUE: Who has the best claim over the land? BCI and Atok Corp WON land can be agricultural and mineral at the same time. NO HELD:

BCI and Atok have vested rights over the land. The Court found that the mineral claims sold to both BCI and Atok have been perfected prior to the approval of the 1935 Constitution. The court had earlier declared the legal effects of a valid mineral claim: it segregates the area from the public domain and confers to the locator the beneficial ownership pf the claim. As of 1935, they were removed from the public domain and had become private properties of BCI and Atok. Even if the land was included in the CCFR, it did not impair the rights vested in both mining companies. The claim of the dela Rosas were utterly disregarded for weak evidence, and even so, they could not have acquired the land through prescription as the same had already been converted to mineral land.

Land cannot be half agri and half mineral. The SC said the classification of land must be categorical. In this

case, while the land was intiailly classified as forest land, it ceased to be so and became completely mineral when the mining claims were perfected. Even if the surface was being tilled, it is still to be considered mineral land.

Jura regalia. The SC discussed jura regalia in Sections 3-6 of Commonwealth Act No. 137. The Regalian

doctrine is intended for the benefit of the State, and not of private persons. Example, if a person is the owner of agricultural land and minerals are discovered underneath, his ownership pf the land does not give him the right to extract or utilize the minerals without the permission of the State, to which the minerals belong. To further expound, once minerals are discovered in the land, the State can discontinue the surface tilling so as to not impede the mining undertaken therein. POLICY: The classification of land must be categorical, it cannot be half and half. The perfection of mining claims convert land to mineral land even if they were previously classified as forest land.

NOTES:

3. Carino vs Insular Government

( 212 U.S. 449 February 23, 1909)

FACTS:

Mateo Carino, an Igorot from the province of Benguet, applied for the registration of a certain land. He

and his ancestors had held the land as owners for more than 50 years, which he inherited under Igorot customs.

There was no document of title issued for the land when he applied for registration. The government contends

that the land in question belonged to the state. Under the Spanish Law, all lands belonged to the Spanish Crown

except those with permit private titles. Moreover, there is no prescription against the crown.

ISSUE: WoN Carino owns the land. – YES.

HELD: (The US Supreme Court decided the case through Mr. Justice Holmes.)

I. Concept of Jura Regalia Natural Resources First Set_Case Digests

Page | 4

Law and justice require that the applicant should be granted what he seeks. It is true that Spain, in its

earlier decrees, embodied the universal feudal theory that all lands were held from the Crown, and perhaps the

general attitude of conquering nations toward people not recognized as entitled to the treatment accorded to

those in the same zone of civilization with themselves. It is true, also, that, in legal theory, sovereignty is absolute,

and that, as against foreign nations, the United States may assert, as Spain asserted, absolute power. But it does

not follow that, as against the inhabitants of the Philippines, the United States asserts that Spain had such power.

The province of Benguet was inhabited by a tribe that the SG, in his argument, characterized as a savage

tribe that never was brought under the civil or military government of the Spanish Crown. It seems probable, if not

certain, that the Spanish officials would not have granted to anyone in that province the registration to which

formerly Carino was entitled by the Spanish laws, and which would have made his title beyond question good.

Whatever may have been the technical position of Spain, it does not follow that, in the view of the United States,

he had lost all rights and was a mere trespasser when the present government seized his land. The argument to

that effect seems to amount to a denial of native titles throughout an important part of the island of Luzon, at

least, for the want of ceremonies which the Spaniards would not have permitted and had not the power to

enforce.

It is true that the government of the Philippines is empowered to enact rules and prescribe terms for

perfecting titles to public lands where some, but not all, Spanish conditions had been fulfilled, and to issue patents

to natives for not more than 16 hectares of public lands actually occupied by the native or his ancestors before

August 13, 1898. BUT this section perhaps might be satisfied if confined to cases where the occupation was of

land admitted to be public land, and had not continued for such a length of time and under such circumstances

as to give rise to the understanding that the occupants were owners at that date.

In the case at bar, it might, perhaps, be proper and sufficient to say that when, as far back as testimony or

memory goes, the land has been held by individuals under a claim of private ownership, it will be presumed to

have been held in the same way from before the Spanish conquest, and never to have been public land. Thus

there is an existence of native title to land, or ownership of land by Filipinos by virtue of possession under a claim

of ownership since time immemorial and independent of any grant from the Spanish Crown, as an exception to the

theory of jura regalia. The older decrees and laws cited by the Carino’s counsel in error seem to indicate pretty

clearly that the natives were recognized as owning some lands, irrespective of any royal grant. In other words,

Spain did not assume to convert all the native inhabitants of the Philippines into trespassers or even into tenants

at will.

NOTES:

4. Lee Hong Kok vs David

(48 SCRA 372, G.R. No. L-30389, December 27, 1972)

FACTS:

Respondent, Aniano David, acquired lawful title to the disputed lot located in Naga City pursuant to his miscellaneous sales application.

I. Concept of Jura Regalia Natural Resources First Set_Case Digests

Page | 5

An order of award and issuance of a sales patent was made by the Director of Lands on June 18, 1958. This covered Lot 2892 containing an area of 226 square meters, which is a portion of Lot 2863 of the Naga Cadastre (a register of property showing the extent, value, and ownership of land for taxation)

The basis for the award of the Director of Lands the Undersecretary of Agriculture and Natural Resources issued on August 26, 1959, Miscellaneous Sales Patent No. V-1209 pursuant to which OCT No. 510 was the issuance by the Register of Deeds of Naga City to defendant-appellee Aniano David on October 21, 1959.

Since the filing of the sales application of Aniano David and during all the proceedings in connection with said application, up to the actual issuance of the sales patent in his favor, the petitioners (Lee Hong Kok) did not put up any opposition or adverse claim thereto.

After the registration and issuance of the certificate and duplicate certificate of title based on a public land patent, the land covered thereby automatically comes under the operation of Republic Act 496.

Petitioners sought the declaration of the Torrens Title of Aniano David as null and void. Their basis of ownership of the disputed lot was through accretion (process of growth or increase).

RTC and Court of Appeals dismissed their complaint. Hence, the case at bar. ISSUES: (1) WON petitioners (Lee Hong Kok) can question the validity of the Torrens Title of respondent. - NO

(2) WON David has original acquisition of title. - YES HELD:

Decision of the Court of Appeals is affirmed.

There is no legal justification for nullyfing the right of David to the disputed lot arising from the grant

made in his favor by respondent officials.

Only the Government, represented by the Director of Lands, or the Secretary of Agriculture and Natural

Resources, can bring an action to cancel a void certificate of title issued pursuant to a void patent.

The legality of the grant is a question between the grantee and the government. Private parties like the

plaintiffs cannot claim that the patent and title issued for the land involved are void since they are not the

registered owners thereof nor had they been declared as owners in the cadastral proceedings of Naga Cadastre

after claiming it as their private property.

It is a well-settled rule that no public land can be acquired by private persons without any grant, express

or implied, from the government

In Cabacug v. Lao, a holder of a land acquired under a free patent is more favorably situated than that of

an owner of registered property. Not only does a free patent have a force and effect of a Torrens Title, but in

addition the person to whom it is granted has likewise in his favor the right to repurchase within a period of 5

years.

It was cited in Cariño v. Insular Government that "Spain in its earlier decrees embodied the universal

feudal theory that all lands were held from the Crown." This was a manifestation of the concept of jura

regalia which was adopted by the present Constitution, ownership however being vested in the state as such

rather than the head thereof.

I. Concept of Jura Regalia Natural Resources First Set_Case Digests

Page | 6

It could therefore be affirmed in Montano v. Insular Government" 18

that "as to the unappropriated public

lands constituting the public domain the sole power of legislation is vested in Congress" They continue to possess

that character until severed therefrom by state grant.

On Imperium v. Dominium

1. Imperium - government authority possessed by the state appropriately embraced in the concept of

sovereignty

2. Dominium - capacity to own or acquire property. The use of this term is appropriate with reference to

lands held by the state in its proprietary character. In such capacity, it may provide for the exploitation

and use of lands and other natural resources, including their disposition, except as limited by the

Constitution.

NOTES:

5. Sunbeam Convenience Food vs CA (181 SCRA 443, G.R. No. 50464, 1990)

FACTS: The Director of Lands issued sales patents in favor of defendant Sunbeam Convenience Foods, Inc., over

the parcels of land both situated in Mariveles, Bataan. Subsequently, such sales patent was transferred to Coral Beach Development Corporation.

The Solicitor General in behalf of the Republic of the Philippines filed before the CFI an action for reversion,

which was dismissed.

The Solicitor General then filed an appeal contending that the CFI committed grave abuse of discretion in

not considering (in its decision to dismiss) the allegation that the subject lans were forest lands. The CFI , therefore, erred in finding the subject lots disposable and alienable lands of public domain under the jurisdiction of the Director of Lands. The CA set aside the decision of the CFI, hence the present petition. ISSUE: Were the sales patents validly issued? Cannot be determined unless the land’s classification is identified.

HELD: The case is remanded. There is a need to rule on the classification of the land. If the land is indeed a forest

land, then it is not subject to alienation or disposition and the case is moot and academic.

Because of the Regalian doctrine, there is a presumption that unclassified lands are of public dominion. There must be a positive act from the government classifying a land into an agricultural land which is alienable.

NOTES:

I. Concept of Jura Regalia Natural Resources First Set_Case Digests

Page | 7

6. Republic vs Sayo

(191 SCRA 71, G.R. No. L-60413, October 31, 1990)

FACTS:

In 1961, the spouses Casiano Sandoval and Luz Marquez filed an original application to register a 33ha.

tract of land known as Lot 7454. The land was part of Isabela but transferred to Nueva Vizcaya by virtue of RA 236.

Opposition (including the Bu of Lands and the Bu of Forest Devt) dragged the cases for 20 yrs until 1981, when the

parties entered a compromise agreement, dividing the land amongst themselves. In 1981, Judge Sayo approved

the compromise agreement and confirmed the title and ownership of the parties. The parties then applied to

register the lot under the Torrens system. The SocGen then stepped in, alleging that since he was counsel for the

Republic, he should have been given notice or allowed to participate.

The respondents claim that the SocGen had no authority as Lot 7454 is not public land. Their primary

evidence to support this was a photocopy of a certificate of the National Library dated 1932 that the property was

registered under the Spanish system of land regsitration in the name on Don Liberato Bayaua.

ISSUE: WON the compromise agreement executed over ownership of Lot 7454 is valid. NO

HELD:

Under the Regalian Doctrine, all lands not otherwise appearing to be clearly within private ownership are

presumed to belong to the State. Unless the applicant succeeds in showing by clear and convincing evidence that

the property involved was acquired by him or his ancestors either by composition title from the Spanish

Government or by possessory information title, or any other means for the proper acquisition of public lands, the

property must be held to be part of the public domain. The applicant must present competent and persuasive

proof to substantiate his claim; he may not rely on general statements, or mere conclusions of law other than

factual evidence of possession and title.

The certificate the respondents present to bolster their case cannot be considered title to property, as it

was not a grant made during the Spanish regime or any other primary evidence of ownership.

Likewise, the parties cannot claim that the land is private, and then apply to register it under the Torrens

Act. The purpose of the Torrens Act is to establish the private character of lands. The compromise agreement was

likewise stuck down as it included persons who have not substantially proven their ownership of the land.

NOTES:

7. Seville vs National Development Company (DAYHON)

(G.R. No. 129401, February 2, 2001)

FACTS:

- Through PD No. 625, Leyte Sab-a Basin Development Authority (LSBDA) was created. LSBA integrated the private and government sector efforts for a planned development and balanced growth of the Sab-a Basin in the province of Leyte. LOI 962 authorized LSBDA to acquire privately-owned lands restricted in the Leyte Industrial Development Estate through negotiated sales with the landowners.

- (1980) Calixtra Yap sold to LSBDA parcels of land (464,920 sq.m), which through tax declaration reflects Yap as the owner of said parcels.

I. Concept of Jura Regalia Natural Resources First Set_Case Digests

Page | 8

- (1982) Appellant LSBDA filed a Miscellaneous Sales Application before the Bureau of Lands covering the land obtained from Yap together with other lots acquired by LSBDA (aggregate area: 442,7508 sq.m)

- Bureau of Lands issued a Miscellaneous Sales Patent in LSBDA’s favor, basing on the fact that the original certificate of title was in the name of LSBDA as transcribed in the Registration Book.

- (1989) LSBDA assigned all its rights over the subject property to National Development Council (NDC), which led to the issuance of the new TCTs in NDC’s favor. The subject property was then leased to Philippine Smelting & Refining Corporation (PASAR), Philippine Phosphate Fertilizer Corporation (PHILPHOS), and Lepanto Consolidated Mining Co., Inc (LEPANTO).

- (1988) Herein respondent filed for a complaint for recovery of real property, rentals and damages against herein respondents.

- The RTC declared: o the deed of Sale executed by Yap as null and void o herein petitioner as owner of the subject real property of the present action and ordered NDC to

segregate from the OCT the area of the subject property and convey it to petitioners o the respondents were also ordered to pay jointly and severally the rentals that were due in

petitioner’s favor o with regard to the improvements introduced to the subject property, petitioners and respondents

were to discuss on the possibility of a compromise agreement on the disposal of said improvements - The CA applying the regalian doctrine, however, reversed the decision of the RTC, ruling that:

o No competent evidence was presented to prove that subject property is indeed private in character o Granting arguendo that such evidences can be provided, failure to prove that such property is

alienable, cannot ripen into ownership o Even prior to the issuance of the Miscellaneous Sales Patent in LSBDA’s favor, the property had been

untitled o Petitioners are guilty of laches for their failure to apply for judicial confirmation of their title o Failure to present evidence of bad faith on the part of respondent LSBDA

ISSUE: WON respondent LSBDA’ title is null and void? NO HELD:

Petitioners argue that LSBDA’s title was void as Yap had no right over said property. They, petitioners, claim that they have acquired title over the property through acquisitive possession of said land for more than 30 years. To refute said allegation, respondents claim that failure of the petitioners to show that such land is alienable and disposable. Because of which, possession of the land cannot ripen into ownership.

Granting arguendo that petitioners are correct in claiming that Yap’s ownership over the property is questionable, however imputing the same to LBDA’s title is misplaced. Petitioners failed to consider the fact that LSBDA’s title was based on the Miscellaneous Sales Patent from the Bureau of Lands and not on the conveyance executed by Yap.

The Court held that because of petitioners’ failure to establish that the land had been classified as alienable prior to the issuance of the title in favor of LSBDA, petitioners cannot claim that they have gained ownership through acquisitive prescription.

Further, the Court held that all the lands of the public domain belong to the State, and that all lands not appearing to be unequivocally owned privately are presumed to belong to the State. To overcome this presumption, evidence must be shown that the land sought to be registered remains inalienable. Consequently, occupation in the concept of owner, no matter how long, cannot ripen into ownership.

NOTES:

I. Concept of Jura Regalia Natural Resources First Set_Case Digests

Page | 9

8. Republic vs ROD Quezon

(244 SCRA 537, G.R. No. 73974, Mar 31, 1995)

FACTS:

Manuel G. Atienza was awarded FP (free patent) No. 324198 over a parcel of land located in Ila, Malicboy, Pagbilao, Quezon an investigation was conducted by the Bureau of Lands in connection with alleged land grabbing activities in Pagbilao.

It appeared that some of the free patents, including that of Atienza's, were fraudulently acquired. A criminal complaint for falsification of public documents was filed in the CFI against Atienza and four other persons for

allegedly falsifying their applications for free patent, the survey plans, and other documents pertinent to said applications. In its decision, the court acquitted the accused of the crime charged but, finding that the land covered by the application

for free patent of private respondent was within the forest zone, declared as null and void OCT (Original certificate of title) No. P-13840 in Atienza's name and ordered the Register of Deeds of Quezon to cancel the same.

Before the promulgation of said decision, the Solicitor General filed for the petitioner a complaint against Atienza, the Register of Deeds of Quezon, and the Rural Bank of Sariaya, which was later dropped as defendant and, in an amended complaint, substituted by the Development Bank of the Philippines as actual mortgagee of the subject parcel of land.

In his answer, Atienza claimed that the land in question was no longer within the unclassified public forest land because by the approval of his application for free patent by the Bureau of Lands, the land "was already alienable and disposable public agricultural land." Since the subject land was a very small portion of Lot, an area which had been declared disposable public land by the cadastral court.

Atienza also averred that the Director of Lands had given due course to free and homestead patent applications of claimants of the said lot.

He further alleged that he had been in possession of the land since the Japanese occupation, cultivating it and introducing improvements thereon.

The DBP, after due and proper investigation and inspection of his title, even granted him a loan with the subject property as collateral.

Finally, he stated that his acquittal in the criminal case proved that he committed no fraud in his application for free patent.

Lower court rendered a decision with the categorical finding based on "solid evidence" that "the land in question was found definitely within the forest zone

On appeal, Atienza maintained that the land in question was not within the unclassified public forest land and therefore alienable land of the public domain.

The then Intermediate Appellate Court relied only on the arguments he raised since petitioner had not filed any brief, and arrived at the conclusion that "the litigated land is part of public land alienable and disposable for homestead and [F]ree Patent."

CA- set aside the lower court's decision, declared as valid and subsisting Atienza's OCT, and dismissed the cross-claim of the DBP.

ISSUE: WON the land in question is part of the alienable and disposable public land. NO HELD: Under the Regalian Doctrine, all lands not otherwise clearly appearing to be privately-owned are presumed to belong to

the State. Forest lands, like mineral or timber lands which are public lands, are not subject to private ownership unless they under the Constitution, become private properties. In the absence of such classification, the land remains unclassified public land until released therefrom and rendered open to disposition.

In our jurisdiction, the task of administering and disposing lands of the public domain belongs to the Director of Lands, and ultimately, the Secretary of Agriculture and Natural Resources (now the Secretary of Environment and Natural Resources).

Classification of public lands is, thus, an exclusive prerogative of the Executive Department through the Office of the President. Courts have no authority to do so.

Thus, in controversies involving the disposition of public agricultural lands, the burden of overcoming the presumption of state ownership of lands of the public domain lies upon the private claimant who, in this case, is Atienza.

The records show, however, that he failed to present clear, positive and absolute evidence to overcome said presumption and to support his claim.

Atienza's claim is rooted in the decision of the Court of First Instance in the Cadastral Case which was not given much weight by the court a quo, and for good reasons.

Apart from his assertions before this Court, Atienza failed to present proof that he or his predecessor-in-interest was one of the claimants who answered the petition filed by the then Attorney-General in the said cadastral proceedings.

I. Concept of Jura Regalia Natural Resources First Set_Case Digests

Page | 10

The fact that Atienza acquired a title to the land is of no moment, notwithstanding the indefeasibility of titles issued under the Torrens system.

In Bornales v. Intermediate Appellate Court, we ruled that the indefeasibility of a certificate of title cannot be invoked by one who procured the same by means of fraud.

The "fraud" contemplated by the law (Sec. 32, P.D. 1529) is actual and extrinsic, that is, "an intentional omission of fact required by law," which in the case at bench consisted in the failure of Atienza to state that the land sought to be registered still formed part of the unclassified forest lands.

NOTES:

9. Ituralde vs Falcasantos (301 SCRA 293, G.R. No. 128017, January 20, 1999) FACTS:

In 1986, Ramon Ituralde obtained a 6k ha parcel of land in Basilan Province. Falcasantos, meanwhile, applied with the Bureau of Lands to grant him the same parcel of land under free patent. In 1989 the Director of Lands allowed Ituralde to file a public land application for the subject property. 1990 the trial court named Ituralde the owner and possessor of the land. The CA reversed this decision and set aside the trial court’s decision in Ituralde’s favor as the land was found to be forest land, and therefore, inalienable. Ituralde, in his appeal to the SC, claims that since the Director of Lands allowed him to file a public land application, the said land was no longer part of public domain. ISSUE: WON the filing of the public land application in 1989 made the land alienable. NO. HELD:

The CA is their decision found that since 1951, the subject parcel of land was classified as a Forest Reserve Area, and hence, not capable of private appropriation and occupation. Before any land may be declassified from the forest group and converted into alienable or disposable land for agricultural or other purposes, there must be a positive act from the government.

The filing of the public land application is not such positive act. The parcel of land retained its public character and it therefore not susceptible to private ownership. As there was no award or grant to Ituralde of the land by free patent or other ways of acquisition, under the concept of jura regalia, the State retains ownership over the land. POLICY:

Forest land, like mineral timber lands which are public lands, are not subject to private ownership unless they under the Constitution, become private properties. In the absence of such classification from the State, the land remains unclassified public land.

NOTES:

I. Concept of Jura Regalia Natural Resources First Set_Case Digests

Page | 11

10. Republic vs Candy Makers (492 SCRA 722) FACTS:

Candy Maker Inc. purchased a parcel of land of the Cainta-Taytay Cadastre located below the reglementary lake elevation of 12.50 meters, about 900 meters away from the Laguna de Bay, and bounded on the southwest by the Manggahan Floodway, and on the southeast by a legal easement. A Subdivision Plan and a technical description for the property, which divided the property into two lots: Lot No. 3138-A and Lot No. 3138- B, was then prepared and approved. Thereafter, respondent applied before the MTC for the registration of its alleged title over the parcel of land under Presidential Decree (P.D.) No. 1529.

The Community Environment and Natural Resources Officer (CENRO) of Antipolo City filed on his Report declaring that "[t]he land falls within the Alienable and Disposable Zone and that the property is the subject of a CENRO case. On the other hand, the LRA, recommended the exclusion of Lot No. 3138-B on the ground that it is a legal easement and intended for public use, hence, inalienable and indisposable.

The Laguna Lake Development Authority (LLDA) approved Resolution No. 113, Series of 1993, providing that untitled shoreland areas may be leased subject to conditions enumerated therein.

The Republic of the Philippines, the LLDA filed its Opposition alleging that the lot subject of the application for registration may not be alienated and disposed since it is considered part of the Laguna Lake bed, a public land within its jurisdiction pursuant to Republic Act (R.A.) No. 4850, as amended.

The MTC rendered judgment confirming title of the applicants over the real property. On appeal, the petitioner contended that the MTC did not acquire jurisdiction over the application for registration since the actual copies of the Official Gazette (O.G.) where the notice of hearing was published were not adduced in evidence; the applicant likewise failed to establish exclusive ownership over the subject property in the manner prescribed by law.

The applicant averred that Sec. 14, par. 1 of P.D. 1529 is inapplicable since it speaks of possession and occupation of alienable and disposable lands of the public domain. Instead, par. 4 of the same section should govern because the subject parcels of land are lands of private ownership, having being acquired through purchase from its predecessors-in-interest, who, in turn, inherited the same from their parents. CA dismissed the appeal and affirmed in toto the decision of the MTC. ISSUES: (1) WON the property subject of the amended application is alienable and disposable property of the State, and if so (2) Whether respondent adduced the requisite quantum of evidence to prove its ownership over the property under Section 14 of P.D. 1529. HELD:

The SC ruled that the property subject of this application was alienable and disposable public agricultural land until July 18, 1966. However, respondent failed to prove that it possesses registerable title over the property.

Under the Regalian doctrine, all lands not otherwise appearing to be clearly within private ownership are presumed to belong to the State. The presumption is that lands of whatever classification belong to the State.

Unless public land is shown to have been reclassified as alienable or disposable to a private person by the State, it remains part of the inalienable public domain. Property of the public domain is beyond the commerce of man and not susceptible of private appropriation and acquisitive prescription. Occupation thereof in the concept of owner no matter how long cannot ripen into ownership and be registered as a title. The statute of limitations with regard to public agricultural lands does not operate against the State unless the occupant proves possession and occupation of the same after a claim of ownership for the required number of years to constitute a grant from the State.

I. Concept of Jura Regalia Natural Resources First Set_Case Digests

Page | 12

No public land can be acquired by private persons without any grant from the government, whether express or implied. It is indispensable that there be a showing of a title from the State. The rationale for the period "since time immemorial or since June 12, 1945" (in PD 1529) lies in the presumption that the land applied for pertains to the State, and that the occupants or possessor claim an interest thereon only by virtue of their imperfect title as continuous, open and notorious possession.

To prove that the land subject of an application for registration is alienable, an applicant must conclusively establish the existence of a positive act of the government such as a presidential proclamation or an executive order, or administrative action, investigation reports of the Bureau of Lands investigator or a legislative act or statute. Until then, the rules on confirmation of imperfect title do not apply. In this case, the certification of the CENRO in the Department of Environment and Natural Resources proved that the land in question is found to be within the alienable and disposable site. However, the applicant is burdened to offer proof of specific acts of ownership to substantiate the claim over the land. The respondent failed to adduce proof that its predecessors-in-interest had acquired registerable title over the property before July 18, 1966.

The petition is Granted. The application for the registration of the property in question is dismissed.

NOTES:

11. Miners vs Factoran (SY) (240 SCRA 100, G.R. No. 98332 January 16, 1995) FACTS:

The instant case was filed by petitioner (Miners Association Of The Philippines, Inc.) questioning the validity and constitutionality of the administrative orders issued by the respondents (Hon. Fulgencio S. Factoran, Jr., Secretary Of Environment And Natural Resources, And Joel D. Muyco, Director Of Mines And Geosciences Bureau) which sprouted from the respective promulgation by the then president Aquino of Executive Orders for the implementation of the new 1987 constitution regarding mining applications and agreements and for guidelines during the transitory period.

The change was introduced by Article XII, Section 2 of the 1987 Constitution governing the system of exploration, development and utilization of the country’s natural resources. No longer is the utilization of inalienable lands of public domain through “license, concession or lease” under the 1935 and 1973 Constitutions allowed under the 1987 Constitution.

Petitioner alleges that among others, the administrative orders and ultimately the executive orders are unconstitutional because, among others, violates the non-impairment of contract provision since the said orders pre-terminates existing mining agreements and automatically converts them into production-sharing agreements. ISSUE: WON the administrative orders are unconstitutional? NO. HELD:

The adoption of the concept of jura regalia that all natural resources are owned by the State embodied in the 1935, 1973 and 1987 Constitutions, as well as the recognition of the importance of the country's natural resources, not only for national economic development, but also for its security and national defense, ushered in the adoption of the constitutional policy of "full control and supervision by the State" in the exploration, development and utilization of the country's natural resources. The options open to the State are through direct undertaking or by entering into co-production, joint venture; or production-sharing agreements, or by entering into agreement with foreign-owned corporations for large-scale exploration, development and utilization.

I. Concept of Jura Regalia Natural Resources First Set_Case Digests

Page | 13

Upon the effectivity of the 1987 Constitution on February 2, 1987, the State assumed a more dynamic

role in the exploration, development and utilization of the natural resources of the country. Article XII, Section 2 of the said Charter explicitly ordains that the exploration, development and utilization of natural resources shall be under the full control and supervision of the State. Consonant therewith, the exploration, development and utilization of natural resources may be undertaken by means of direct act of the State, or it may opt to enter into co-production, joint venture, or production-sharing agreements, or it may enter into agreements with foreign-owned corporations involving either technical or financial assistance for large-scale exploration, development, and utilization of minerals, petroleum, and other mineral oils according to the general terms and conditions provided by law, based on real contributions to the economic growth and general welfare of the country.

The economic policy on the exploration, development and utilization of the country's natural resources

under Article XII, Section 2 of the 1987 Constitution could not be any clearer. As enunciated in Article XII, Section 1 of the 1987 Constitution, the exploration, development and utilization of natural resources under the new system mandated in Section 2, is geared towards a more equitable distribution of opportunities, income, and wealth; a sustained increase in the amount of goods and services produced by the nation for the benefit of the people; and an expanding productivity as the key to raising the quality of life for all, especially the underprivileged.

The exploration, development and utilization of the country's natural resources are matters vital to the public interest and the general welfare of the people. Accordingly, the State, in the exercise of its police power in this regard, may not be precluded by the constitutional restriction on non-impairment of contract from altering, modifying and amending the mining leases or agreements granted under Executive Orders. Police Power, being co-extensive with the necessities of the case and the demands of public interest; extends to all the vital public needs.

NOTES:

II. Limitations Natural Resources First Set_Case Digests

Page | 14

II. Limitations

1. La Bugal-b’laan v. Ramos

(445 SCRA 1, G.R. No. 127882, December 1, 2004)

FACTS:

President Corazon Aquino issued EO 2796 on July 25, 1987, authorizing DENR Secretary to accept,

consider and evaluate proposals from foreign-owned corporations.

President Fidel V. Ramos approved RA 7942 (Philippine Mining Act of 1995) on March 3, 1995. This law

“shall govern the exploration, development, utilization and processing of all mineral resources (sec 15).” It contained provisions regarding FTAAs, as well as the different modes of mineral agreements for mining:

(1) The State may directly undertake such activities. (2) The State may enter into co-production, joint venture or production-sharing agreements with Filipino citizens or qualified corporations. (3) Congress may, by law, allow small-scale utilization of natural resources by Filipino citizens. (4) For the large-scale exploration, development and utilization of minerals, petroleum and other mineral oils, the President may enter into agreements with foreign-owned corporations involving technical or financial assistance. ISSUE: Is RA 7942 unconstitutional? NO. HELD: (At first Yes, for being violative of Section 2, Article XII of the Constitution. – Jan 204 decision)… ie all other natural resources shall not be alienated, preserved for the Filipinos, etc…

Ultimately, the Court upheld the constitutionality of RA 7942 – Dec 2004 decision. The Constitution should be read in broad, life-giving strokes. It should not be used to strangulate economic growth. It should be construed to grant the President and Congress sufficient discretion to attract foreign investments and expertise, as well as secure for our people blessings of prosperity and peace.

As per Section 2, Art XII: … “The President may enter into agreements with foreign-owned corporations involving either technical or financial assistance for large-scale exploration, development, and utilization of minerals, petroleum, and other mineral oils according to the general terms and conditions provided by law, based on real contributions to the economic growth and general welfare of the country. In such agreements, the State shall promote the development and use of local scientific and technical resources.”

Note: (based from dissenting opinions) Limitations under Sec 2 Article XII

1. State retains legal ownership of all natural resources 2. State shall have full control and supervision over the exploration, development and utilization of natural

resources.

Specific limitations / restrictions in fourth paragraph of Sec 2 (power of president to enter into agreements with foreign corporations):

First, the natural resources that may be subject of the agreement are a limited class, particularly minerals, petroleum, and other mineral oils. Among the natural resources which are excluded from these agreements are lands of the public domain, waters, coal, fisheries, forests or timbers, wildlife, flora and fauna. Most notable of the exclusions are forests and timbers which are in all respects expressly limited to Filipinos.

Name:

II. Limitations Natural Resources First Set_Case Digests

Page | 15

Second, these agreements with foreign-owned corporations can only be entered into for only large-scale exploration, development and utilization of minerals, petroleum, and other mineral oils.

Third, it is only the President who may enter into these agreements. This is another pronounced change from the 1973 Constitution, which allowed private persons to enter into service contracts with foreign corporations.

Fourth, these agreements must be in accord with the general terms and conditions provided by law. This proviso by itself, and more so when taken together, as it should, with another provision, entails legislative intervention and affirmance in the exercise of this executive power. While it is the President who enters into these contracts, he/she must act within such terms and conditions as may be prescribed by Congress through legislation. The value of legislative input as a means of influencing policy should not be discounted. Policy initiatives grounded on particular economic ideologies may find enactment through legislation when approved by the necessary majorities in Congress. Legislative work includes consultative processes with persons of diverse interests, assuring that economic decisions need not be made solely from an ivory tower. There is also the possible sanction of repudiation by the voters of legislators who prove insensate to the economic concerns of their constituents.

Fifth, the President is mandated to base the decision of entering into these agreements on "real contributions to the economic growth and general welfare of the country." In terms of real limitations, this condition has admittedly little effect. The discretion as to whether or not to enter into these agreements is vested solely by the Constitution in the President, and such exercise of discretion, pertaining as it does to the political wisdom of a co-equal branch, generally deserves respect from the courts.

NOTES:

2. JG Summit vs CA (450 SCRA 169, G.R. No. 124293, January 31, 2005) FACTS:

In 1977, the National Investment and Development Corporation (NIDC) and Kawasaki of Japan entered into a joint venture agreement (JVA) to build a shipyard in Subic that would later become the PHILSECO. They agreed to have a shareholding proportion of 60-40 respectively. The JVA also provided that neither party shall dispose of its interest in PHILSECO without granting the other party the right of first refusal under the same terms offered to third parties. By 1987, the NIDC’s interest in PHILSECO had been transferred to the National Government of the Philippines, so that by 1989, Kawasaki only had 2.59% share. In 1986, the amdinistration thought to sell the government’s shares in PHILSECO through to private entities as part of their privatization plan. Before the auction, the gov’t and Kawasaki agreed to allow PHI (Kawasaki is a stockholder in PHI) to top the winning bid by 5%.

The auction was set in 1993, where the consortium of JG Summit, Sembawang and Jurong was declared

the winners, having bid P2.03B. JG Summit learned that PHI was exercising its rights to top their winning bid, and appealed at the CA. JG

Summit alleged that among others, the right of first refusal granted to Kawasaki when it had 40% shares in PHILSECO was contrary to the Constitution. JG Summit avers that PHILSECO is a shipyard and therefore a public utility. Hence, it could only be operated only by a corporation at least 60% of whose capital is owned by Filipino citizens in accordance with Sec 10 Art 12 of the Constitution. PHI countered this with PD 666 which explicitly stated in Section 1 that shipyards were not public utilities.

II. Limitations Natural Resources First Set_Case Digests

Page | 16

ISSUE: WON shipyards are public utilities and whose ownership therefore should adhere to constitutional requirements. YES.

HELD: Subsequent laws repealed PD 666 Section 1. Section 1 of PD 666 which declassifies shipyards as public

utilities were repealed by the Investment Incentive Policy Act of 1983; which was also repealed by EO 226, or the Omnibus Investments Code of 1987.

(Recall StatCon rules on repeals: when law A is repealed by law B, and law B is repealed by C, law A is not revived unless explicitly stated by C.)

Thus, the subsequent repeals of the law effectively erased PD 666 and restored the public utility character of shipyards even before the schedule for bidding was set. Thus, as PHILSECO is a public utility as provided by law, the 60% per centum Filipino requirement in Section 11, Art 12 applies.

Kawasaki, through PHI cannot participate in purchasing PHILSECO’s shares, as it would put them over and beyond the 40% limit of its shareholdings. The auction was awarded to JG Summit.

POLICY: No foreign owned company may own more than the constitutional limit of Philippine public utilities, including shipyards.

NOTES:

3. Tano vs Socrates

(August 21, 1997, G.R. No. 110249. August 21, 1997)

FACTS:

Petitioners (they’re so many—i.e. natural persons who claim to be fishermen but are alleged marine merchants) file a petition to, among others: declare as unconstitutional (a) an ordinance of the Sangguniang Panlungsod of Puerto Princesa (that ban the shipment of all live fish and lobster outside Puerto Princesa City without the required permits/documents,) (b) the Office Order issued by the acting city mayor implementing said city ordinance, and (c) a resolution (that prohibit the catching, gathering, possessing, buying, selling and shipment of live marine coral dwelling aquatic organisms, as listed) of the Sangguniang Panlalawigan of Palawan. The ordinance basically aims to effectively free the city sea waters from cyanide and other obnoxious substance while the Resolution aims to protect and preserve the existence of the remaining excellent corals and allow the devastated ones to reinvigorate and regenerate themselves into vitality within the span of five (5) years

Petitioners argue among others that the ordinances deprived them of due process of law, their livelihood, and unduly restricted them from the practice of their trade, in violation of Section 2, Article XII and Sections 2 and 7 of Article XIII of the 1987 Constitution. ISSUE: WoN the challenged ordinance, Office Order and Resolution are unconstitutional – NO HELD:

The pertinent portion of Section 2 of Article XII reads: SEC. 2. x x x The State shall protect the nation's marine wealth in its archipelagic waters, territorial sea, and exclusive economic zone, and reserve its use and enjoyment exclusively to Filipino citizens.

II. Limitations Natural Resources First Set_Case Digests

Page | 17

The Congress may, by law, allow small-scale utilization of natural resources by Filipino citizens, as well as cooperative fish farming, with priority to subsistence fishermen and fishworkers in rivers, lakes, bays, and lagoons.

Sections 2 and 7 of Article XIII provide: Sec. 2. The promotion of social justice shall include the commitment to create economic opportunities based on freedom of initiative and self-reliance. xxx Sec. 7. The State shall protect the rights of subsistence fishermen, especially of local communities, to the preferential use of the communal marine and fishing resources, both inland and offshore. It shall provide support to such fishermen through appropriate technology and research, adequate financial, production, and marketing assistance, and other services. The State shall also protect, develop, and conserve such resources. The protection shall extend to offshore fishing grounds of subsistence fishermen against foreign intrusion. Fishworkers shall receive a just share from their labor in the utilization of marine and fishing resources.

There is absolutely no showing that any of the petitioners qualifies as a subsistence or marginal fisherman. In their petition, petitioner Airline Shippers Association of Palawan is described as “a private association composed of Marine Merchants”; petitioners Robert Lim and Virginia Lim, as “merchants;” while the rest of the petitioners claim to be “fishermen,” without any qualification, however, as to their status.

Since the Constitution does not specifically provide a definition of the terms “subsistence” or “marginal”

fishermen, they should be construed in their general and ordinary sense. A marginal fisherman is an individual

engaged in fishing whose margin of return or reward in his harvest of fish as measured by existing price levels is

barely sufficient to yield a profit or cover the cost of gathering the fish, while a subsistence fisherman is one whose

catch yields but the irreducible minimum for his livelihood.

Besides, Section 2 of Article XII aims primarily not to bestow any right to subsistence fishermen, but to lay

stress on the duty of the State to protect the nation’s marine wealth. What the provision merely recognizes is

that the State may allow, by law, cooperative fish farming, with priority to subsistence fishermen and fishworkers

in rivers, lakes, bays, and lagoons.

Anent Section 7 of Article XIII, it speaks not only of the use of communal marine and fishing resources, but of their protection, development, and conservation. As hereafter shown, the ordinances in question are meant precisely to protect and conserve our marine resources to the end that their enjoyment by the people may be guaranteed not only for the present generation, but also for the generations to come.

The so-called “preferential right” of subsistence or marginal fishermen to the use of marine resources is not at all absolute. In accordance with the Regalian Doctrine, marine resources belong to the State, and, pursuant to the first paragraph of Section 2, Article XII of the Constitution, their “exploration, development and utilization ... shall be under the full control and supervision of the State.” Moreover, their mandated protection, development, and conservation as necessarily recognized by the framers of the Constitution, imply certain restrictions on whatever right of enjoyment there may be in favor of anyone. What must likewise be borne in mind is the state policy enshrined in the Constitution regarding the duty of the State to protect and advance the right of the people to a balanced and healthful ecology in accord with the rhythm and harmony of nature.

The challenged ordinance and resolution unquestionably involve a valid exercise of police power. There is

nothing in the Office order that is violative of any constitutional or statutory provision. (SC also quoted its decision in Oposa vs. Factoran such as “the right to a balanced and healthful ecology carries with it a correlative duty to refrain from impairing the environment”)

II. Limitations Natural Resources First Set_Case Digests

Page | 18

NOTES:

4. Shell Philippines v. Jalos et.al. (G.R. No. 179918, September 8, 2010) This case is about a question of jurisdiction over an action against a petroleum contractor, whose pipeline operation has allegedly driven the fish away from coastal areas, inflicting loss of earnings among fishermen. FACTS:

On December 11, 1990 petitioner Shell Philippines Exploration B.V. (Shell) and the Republic of the Philippines entered into Service Contract 38 for the exploration and extraction of petroleum in northwestern Palawan.

Shell discovered natural gas in the Camago-Malampaya area and pursued its development of the well under the Malampaya Natural Gas Project. This entailed the construction and installation of a pipeline from Shell’s production platform to its gas processing plant in Batangas. The pipeline spanned 504 kilometers and crossed the Oriental Mindoro Sea.

Respondents Efren Jalos, Joven Campang, Arnaldo Mijares, and 75 other individuals (Jalos, et al) filed a complaint for damages against Shell before the RTC claiming that they were all subsistence fishermen from the Oriental Mindoro whose livelihood was adversely affected by the construction and operation of Shell’s natural gas pipeline.

Jalos, et al claimed that their fish catch became few after the construction of the pipeline. As a result, their average net income per month fell from a high of P4,848.00 to only P573.00. They now have to stay longer and farther out at sea to catch fish, as the pipeline’s operation has driven the fish population out of coastal waters.

Shell moved for dismissal of the complaint. It alleged that the trial court had no jurisdiction over the action, as it is a "pollution case" under Republic Act (R.A.) 3931, as amended by Presidential Decree (P.D.) 984 or the Pollution Control Law.

Under these statutes, the Pollution Adjudication Board (PAB) has primary jurisdiction over pollution cases and actions for related damages.

RTC dismissed the complaint. It ruled that the action was actually pollution-related, although denominated as one for damages. The complaint should thus be brought first before the PAB, the government agency vested with jurisdiction over pollution-related cases. CA reversed RTC decision. ISSUE: WON the complaint is a pollution case that falls within the primary jurisdiction of the PAB. YES HELD:

Although the complaint of Jalos, et al does not use the word "pollution" in describing the cause of the alleged fish decline in the Mindoro Sea, it is unmistakable based on their allegations that Shell’s pipeline produced some kind of poison or emission that drove the fish away from the coastal areas. While the complaint did not specifically attribute to Shell any specific act of "pollution," it alleged that "the pipeline greatly affected biogenically hard-structured communities such as coral reefs and led to stress to the marine life in the Mindoro Sea." This constitutes "pollution" as defined by law.

Section 2(a) of P.D. 984 defines "pollution" as "any alteration of the physical, chemical and biological properties of any water x x x as will or is likely to create or render such water x x x harmful, detrimental or

II. Limitations Natural Resources First Set_Case Digests

Page | 19

injurious to public health, safety or welfare or which will adversely affect their utilization for domestic, commercial, industrial, agricultural, recreational or other legitimate purposes."

It is clear from this definition that the stress to marine life claimed by Jalos, et al is caused by some kind of

pollution emanating from Shell’s natural gas pipeline. The pipeline, they said, "greatly affected" or altered the natural habitat of fish and affected the coastal waters’ natural function as fishing grounds.

Inevitably, in resolving Jalos, et al’s claim for damages, the proper tribunal must determine whether or not the operation of the pipeline adversely altered the coastal waters’ properties and negatively affected its life sustaining function. The power and expertise needed to determine such issue lies with the PAB.

EO 192 (1987) transferred to the PAB the powers and functions of the National Pollution and Control Commission provided in R.A. 3931, as amended by P.D. 984.

These empowered the PAB to "determine the location, magnitude, extent, severity, causes and effects" of water pollution. Among its functions is to "serve as arbitrator for the determination of reparation, or restitution of the damages and losses resulting from pollution."

In this regard, the PAB has the power to conduct hearings, impose penalties for violation of P.D. 984, and issue writs of execution to enforce its orders and decisions.

Respondents had an administrative recourse before filing their complaint with the regular courts.

Respondents complaint for damages against Shell is DISMISSED without prejudice to its refiling with the Pollution Adjudication Board or PAB.

NOTES:

5. Manila Prince vs GSIS (267 SCRA 408) FACTS:

GSIS sold, through public bidding, 51 % of the shares of Manila Hotel Corporation to Renong Berhad, a Malaysian corporation. One of the bidders and herein petitioner is Manila Prince Hotel which assails the sale by invoking the Filipino First Policy enshrined in the 1987 Constitution granting preference to qualified Filipinos regarding the right, privileges, and concessions covering the national economy and patrimony.

This was after GSIS refused petitioner’s proposal to match the winning bid.

ISSUE: Is the 51 % shares of MHC considered a national patrimony as to warrant the application of the Filipino First Policy? YES HELD:

The patrimony of the Nation that should be conserved and developed refers not only to our rich natural resources but also to the cultural heritage of our race. It also refers to our intelligence in arts, sciences and letters. Therefore, we should develop not only our lands, forests, mines and other natural resources but also the mental ability or faculty of our people.

II. Limitations Natural Resources First Set_Case Digests

Page | 20

In its plain and ordinary meaning, the term patrimony pertains to heritage. When the Constitution speaks of national patrimony, it refers not only to the natural resources of the Philippines, as the Constitution could have very well used the term natural resources, but also to the cultural heritage of the Filipinos. MHC has been witness to several historical occassions of the country and is thus a living testimonial of Philippine heritage. The Filipino First Policy which has been decided as a self executing provision warrants application in the matter of the shares.

NOTES:

III. Classification of Lands of the Public Domain Natural Resources First Set_Case Digests

Page | 21

III. CLASSIFICATIONS OF LANDS OF THE PUBLIC DOMAIN

1. Ayog vs Cusi

(118 SCRA 492, G.R. No. L-46729, November 19, 1982)

FACTS: This case is about the application of section 11, Article XIV of the 1973 Constitution (disqualifying a private

corporation from purchasing public lands) to a 1953 sales award made by the Bureau of Lands, for which a sales

patent and Torrens title were issued in 1975, and to the 1964 decision of the trial court, ejecting some of the

petitioners from the land purchased. After an auction in 1953, a 250ha lot in Davao City was awarded by the Dir. of Lands to Biñan Devt Co, Inc.

(BDCI). Some occupants (herein petitioners) of the lot protested the sale, but were later ordered to vacate the lot, as some occupants entered the lot only after it had been awarded to BDCI, thereby classifying them as squatters. In 1961, BDCI filed an ejectment suit against 40 of them. In 1961, PDCI paid the purchase price of 10k. The patent was issued more than 13 years later in 1975, after the 1973 Constitution took effect.

In 1964, the trial court ordered the ejectment of the petitioners. The ‘squatters’ again opposed the execution of the ejectment. They contended that the adoption of the Constitution in 1973 was a supervening fact that effectively legally barred the execution of the lower court’s judgment. They invoked the constitutional prohibition that ‘no private corporation or association may hold alienable land of the public domain except by lease not to exceed one thousand ha’s in area.”

ISSUE: WON the prohibition against private corporations in the 1973 Constitution would bar BDCI from asserting ownership over the land. NO.

HELD: BDCI’s right to the land has already vested. The constitutional prohibition cannot be applied retroactively

to BDCI as its rights to the property had already vested. Under Section 2, Article 13 of the 1935 Constitution allowed private corporations to purchase public agri lands not exceeding 1024 ha.

Recall vested right: it is some right or interest in property which has become fixed and established and no longer open to doubt or controversy.

Legal effects of complying with Public Land Law. As BDCI paid the full purchase price before 17 Jan 1973, BDCI was entitled to a sales patent. Its compliance with the requirements of the Public Land Law had the effect of segregating the said land from the public domain. BDCI’s right to obtain a patent for the land is protected by law.

POLICY: Compliance with requirements of the Public Land Law segregated lands from the public domain. A corporation who has purchased land before the effectivity of the 1973 Constitution has vested rights provided the payment was completed prior to the 1973 Consti’s effectivity.

NOTES:

Name:

III. Classification of Lands of the Public Domain Natural Resources First Set_Case Digests

Page | 22

2. Republic v. Iglesia

(June 29, 1982)

FACTS:

In 1953, the Iglesia ni Cristo acquired 2 parcels of land from Andres Perez in exchage for a lot that INC owned. Both lots were already possessed by Perez since 1933, and it was inside an area certifies as alienable by the Bu of Forestry in 1927. In 1977, the INC a corporation sole duly existing under PHL laws, filed with the CFI an application for the registration of the 2 lots. It alleged that it and its predecessors-in-interest had possessed the land for more than 30 years and had a right to an imperfect title by virtue of Sec 48 Ch 8 of the Public Land Law.

The Republic, through the Dir of Lands, opposed on the grounds that as INC was a private corporation, the INC was disqualified to hold alienable lands of public domain, that the land was in fact public land not susceptible to private apporpriation, and that INC and Perez have not been in open, continuous, exclusive and notorious possession of the land since June 12, 1945. The trial court ruled in favor of the INC and ordered the registration of the two lots in the name of INC represented by Erano Manalo. The Republic appealed. ISSUE: WON the INC may acquire lands of public domain. NO. The lower Court’s decision was reversed and INC’s application dismissed. HELD:

The INC may not hold or acquire lands of public domain. INC, as a corporation sole or a juridical person, is disqualified to acquire alienable lands of public domain precisely because of the constitutional prohibition. Likewise, the INC cannot use the Sec 48 of the Public Land Law to bolster its claim to registry as the benefits of the law applies only to Filipino citizens or natural persons. The lands were not private lands. The lots sought to be registered are still public lands as they were not possessed by a Filipino ‘since time imemorial,’ as in the Susi v Razon case. Likewise, a land registration proceeding under Sec 48 of the Public Land Law presupposes that the land is public. On the public character of the lots: All lands that were not acquired from the Govt, either by pruchase or by grant, belong to the public domain. The exception to the rule would be any land that should have been in the possession of an occupant and of his predecessors-in-interest since time imemorial, for such possession would justify the presumption that the land had never been part of the public domain or that it had been private property even before the Spanish conquest. POLICY: All lands not purchased from or granted by the Government belong to the public domain, with the exception of a land occupied ‘since time immeorial.’ This occupation is enough to rebut the presumption of land’s public character.

NOTES:

3. Meralco v. Casto-Bartolome

(114 SCRA 799, L-49623, June 29, 1982)

FACTS: This case involves the prohibition in section 11, Article XIV of the Constitution that "no private coporation

or associaiton may hold alienable lands of the public domain except by lease not to exceed on a thousand hectares in area". * That prohibition is not found in the 1935 Constitution.

III. Classification of Lands of the Public Domain Natural Resources First Set_Case Digests

Page | 23

The Manila Electric Company, a domestic corporation organized under Philippine laws, more than sixty percent of whose capital stock is owned by Filipino citizens, in its application filed in the CFI praying for the confirmation of its title to two lots located in Tanay, Rizal

The Republic of the Philippines opposed the application on the grounds that the applicant, as a private corporation, is disqualified to hold alienable public lands and that the applicant and its predecessors-in-interest have not been in the open, continuous, exclusive and notorious possession and occupation of the land for at least thirty years immediately preceding the filing of the application

the Province of rizal and the Municipality of Tanay filed a joint opposition to the application on the ground that one of the lots (Tanay cadastre), would be needed for the widening and improvement of Jose Abad Santos and E.Quirino Streets in the town of Tanay.

The land was possessed by Olimpia ramos before the Pacific war Ramos sold the land to the spouses Rafael Piguing and MInerva Inocencio Because the Meralco had installed the "anchor guy" of its steel post on the land, the Piguing spouses sold

the lot to the Meralco The said land was included in the1968 cadastral survey made in Tanay by the Bureau of Lands and was

divided into two lots so as to segregate the lot which would be used to widen the two street serving as the land's eastern and southern boundaries.

It is residential in character as distinguished from a strictly agricultural land. Since 1927, it has formed part of the alienable portion of the public domain.

lower court rendered a decision dismissing the application because in its opinion the Meralco is not qualified to apply for the registration of the said land since under section 48(b) of the Public Land Law only Filipino citizens or natural persons can apply for judicial confirmation of their imperfect titles to public land.

The Meralco is a juridical person. The trial court assumed that the land which it seeks to register is public land.

the Meralco appealed to this Court under Republic Act No. 5440. Meralco contends that the said land, after having been possessed in the concept of owner by Olimpia

Ramos and the Piguing spouses for more than thirty years, had become private land in the hands of the latter, and, therefore, the constitutional prohibition, banning a private corporation from acquiring alienable public land, is not applicable to the said land.

The Meralco further contends that it has invoke section 48(b) of the Public Land Law, not for itself, but for the Piguing spouses who, as Filipino citizens, could secure a judicial confirmation of their imperfect title to the land.

In reply to these contentions, the Solicitor General counters that the said land is not private land because the Meralco and its predecessors-in-interest have no composition title from the Spanish government nor possessory information title or any other means for the acquisition of public lands such as grants or patents

ISSUE: WON the land in question is a public land. YES

HELD: We hold that, as between the State and the Meralco, the said land is still public land. It would cease to be public land only upon the issuance of the certificate of title to any Filipino citizen

claiming it under section 48(b). Because it is still public land and the Meralco, as a juridical person, is disqualified to apply for its registration under section 48(b), Meralco's application cannot be given due course or has to be dismissed.

"all lands that were not acquired from the Government, either by purchase or by grant, belong to the public domain.

An exception to the rule would be any land that should have been in the possession of an occupant and of his predecessors-in-interest since time immemorial, for such possession would justify the presumption that the land had never been part of the public domain or that it had been a private property even before the Spanish conquest

III. Classification of Lands of the Public Domain Natural Resources First Set_Case Digests

Page | 24

Meralco does not pretend that the Piguing spouses and their predecessor had been in possession of the land since time immemorial.

this Court applied section 45(b) of Act No. 2874 which corresponds to what is now section 48(b). It was held that the long possession of the land under a bona fide claim of ownership gave rise to the conclusive presumption that the occupant had complied with all the conditions essential to a Government grant and was thus entitled to a certificate of title.

That means that until the certificate of title is issued, a piece of land, over which an imperfect title is sought to be confirmed, remains public land.

For that same reason, lands over which an imperfect title is sought to be confirmed are governed by the Public Land Law. Such lands would not be covered by the Public Land Law if they were already private lands.

Finally, it may be observed that the constitutional prohibition makes no distinction between alienable agricultural public lands as to which no occupant has an imperfect title and alienable lands of the public domain as to which an occupant has an imperfect title subject to judicial confirmation.

Since section 11 of Article XIV does not distinguish, we should not make any distinction or qualification. The prohibition applies to alienable public lands as to which a Torrens title may be secured under section 48(b). The proceeding under section 48(b) "presupposes that the land is public"

NOTES:

4. Director v. IAC (146 SCRA 509, G.R. No. L-73002, December 29, 1986) FACTS:

The case is a dispute over the registration of 5 parcels of land in Isabela, allegedly acquired in October 1962 by the private respondent ACME Plywood from their ancestors (the Infiels) who were members of the Dumagat tribe, and who have allegedly been in possession of the land since ‘before the Philippines was discovered by Magellan.’ ACME has since been in continuous, adverse and public possession of the land since 1962, and has since introduced more than 45m worth of improvements. Finally, ACME anchors its claim that RA 3872 grants absolute ownership of ancestal lands to the occupying non-Christian tribe, regardless of whether the land is public domain. Under CA 141, the lands were registered to ACME.

The Director of Lands, however state that the registration proceedings for the parcels of land were instituted only in 1981, long after the effectivity of the 1973 Constitution. Section 11, Article 14 of 1973 Constitution prohibits private corporations from holding alienable lands of public domain except in lease not exceeding 1k ha. Thus, according to the DL, the applicable law should be the 1973 Consti and not CA 141.

ISSUE: WON the title sold by the Infiels to ACME in 1962 can be registered in favor of ACME in 1981, when the 1973 Consti was already in effect? YES.

HELD: The Infiels already had vested right to dispose the lands to ACME. The Court in this case recalled the Carino doctrine, which states that when a piece of alienable public land is possessed openly, exclusively and undisputed for the period prescribed by law, the land ceases to be public land and becomes private property by virtue of legal fiction and without the need of judicial or other sanctions. Should these conditions be met pursuant to law, it is equivalent to a Government grant and should be entitled to certificate of title. The registration would not convert the land from public to private, but rather, merely confirm such conversion already affected by operation of law.

III. Classification of Lands of the Public Domain Natural Resources First Set_Case Digests

Page | 25

The Court in this case said that the Carino doctrine was applicable as the Infiels’ claim to the land was not contested, and that the Infiels’ disposition of the land to ACME in 1962 was well within their vested rights. Likewise, as the sale was made under the 1935 Constitution, ACME was not disqualified to make such an acquisition.

POLICY: The character of lands should be determined at the time of the institution of the registration proceedings. If the lands were still public, then registration would not be possible. If the lands were private by some legal reason (as in this case), then the constitutional prohibition against their acquisition by corporations should not apply.

NOTES:

5. De Ocsio v. CA (February 28, 1989)

FACTS: The controversy at bar arose in connection with cadastral proceedings initiated by the Director of Lands,

in behalf of the Republic, for the settlement and adjudication of title to a large tract of land measuring 261.5791 hectares, divided into 1,419 lots, situated in the City of Iligan.

Herein petitioner, Victoria Ong de Ocsio, alleged that she was the owner, by purchase, of two (2) parcels

of land with specific boundaries comprehended in the cadastral proceeding: Lot No. 1272, measuring 256 square meters, and Lot 1273 a road lot, measuring 21 square meters; and that as owner, she had been in possession of both lots for fifteen (15) years, and her predecessors-in-interest, for sixty (60) years. Title to the same parcels of land was however claimed by the respondents, Religious of the Virgin Mary. In its answer, it averred that it had bought the lots from Victoria Ong de Ocsio and had been in possession as owner thereof for over four years, and its possession and that of its predecessors was immemorial.

The Cadastral Court rendered judgment in favor of the respondents stating that the evidence satisfactorily

established that Victoria Ong de Ocsio had in truth sold Lot No. 1272 to the Religious of the Virgin Mary in virtue of a deed of sale dated April 12, 1956, and Lot No. 1273 was a road right of way granted to the City of Iligan.

The CA, on appeal, affirmed the Cadastral court’s decision in toto.

ISSUE: WON the respondents, being a religious corporation, is disqualified to obtain title over public lands. NO. HELD:

As regards the issue of law raised by her, petitioner fares no better. Citing Manila Electric Co. v. Castro-Bartolome and Republic v. Villanueva, in relation to Section 11, Article XIV of the 1973 Constitution, she asserts that as the private respondent is a religious corporation, it is disqualified to obtain judicial confirmation of an imperfect title under Section 48(b) of the Public Land Act which grants that right only to natural persons. The cited rulings no longer control. Current doctrine, first announced by the Court en banc in Director of Lands v. I.A.C., is that open, continuous and exclusive possession of alienable public land for at least thirty (30) years in accordance with the Public Land Act ipso jure converts the land to private property, and a juridical person who thereafter acquires the same may have title thereto confirmed in its name. Virtually the same state of facts obtained in said case that now obtain here. A private corporation had purchased the land originally of the public domain from parties who had, by themselves and through their predecessors-in-interest, possessed and occupied it since time immemorial. It had thereafter instituted proceedings for confirmation of title under Section 48(b) of the Public Land Act. In upholding its right to do so, the court held that the fact that the proceedings had been instituted by said purchaser in its own name and not in the name of the transferors was "xx simply xx (an) accidental circumstance, productive of a defect hardly more than procedural and in nowise affecting the substance and

III. Classification of Lands of the Public Domain Natural Resources First Set_Case Digests

Page | 26

merits of the right of ownership sought to be confirmed." The ruling was reaffirmed in two later cases, Director of Lands v. Manila Electric Co., and Republic v. C.A., where the same question of law was raised. In the latter it was expressly held that the prohibitions in the 1973 and 1987 Constitutions against acquisition or registration of lands by or in behalf of private corporations do not apply to public lands already converted to private ownership by natural persons under the provisions of the Public Land Act. In the present case, Virginia Ong de Ocsio and her predecessors-in-interest having possessed Lot No. 1272 for the period and under the conditions prescribed by law for acquisition of ownership of disposable public land prior to the sale of the property to the Religious of the Virgin Mary, confirmation of title thereto in the latter's name is, under the precedents referred to, entirely in order.

NOTES:

6. Pagkatipunan v. CA (379 SCRA 621, G.R. No. 129682, March 21, 2002) FACTS:

Petitioners (Nestor Pagkatipunan and Rosalina Mañagas-Pagkatipunan) were able to register a parcel of land. 18 years later, the Republic of the Philippines filed with the CA an action to cancel the registration for the certificates of title therein are null and void alleging that at the tile of the filing of the land registration, the subject land was still classified as timberland, hence inalienable and not subject to registration.

Petitioner on the other hand raised the defenses of indefeasibility of title for it took the government 18

years to assail the validity of the certificate of title issued to them. ISSUE: WON the republic is barred by prescription? NO. HELD

Unless public land is shown to have been reclassified or alienated to a private person by the State, it remains part of the inalienable public domain. Occupation thereof in the concept of owner, no matter how long, cannot ripen into ownership and be registered as a title. Furthermore, evidence extant on record showed that at the time of filing of the application for land registration and issuance of the certificate of title over the disputed land in the name of petitioners, the same was timberland and formed part of the public domain.

Under the Regalian doctrine, all lands of the public domain belong to the State, and the State is the source

of any asserted right to ownership in land and charged with the conservation of such patrimony. This same doctrine also states that all lands not otherwise appearing to be clearly within private ownership are presumed to belong to the State. To overcome such presumption, incontrovertible evidence must be shown by the applicant that the land subject of the application is alienable or disposable.

In the case at bar, there was no evidence showing that the land has been reclassified as disposable or

alienable. Before any land may be declassified from the forest group and converted into alienable or disposable land for agricultural or other purposes, there must be a positive act from the government. Even rules on the confirmation of imperfect titles do not apply unless and until the land classified as forest land is released in an official proclamation to that effect so that it may form part of the disposable agricultural lands of the public domain. Declassification of forest land is an express and positive act of Government. It cannot be presumed. Neither should it be ignored nor deemed waived. It calls for proof.

The classification of forest land, or any land for that matter, is descriptive of its legal nature or status, and

does not have to be descriptive of what the land actually looks like. A person cannot enter into forest land and by the simple act of cultivating a portion of that land, earn credits towards an eventual confirmation of imperfect

III. Classification of Lands of the Public Domain Natural Resources First Set_Case Digests

Page | 27

title. The Government must first declare the forest land to be alienable and disposable agricultural land before the year of entry, cultivation, and exclusive and adverse possession can be counted for purposes of an imperfect title.

Thus, it is clear that the applicant must prove not only his open, continuous, exclusive and notorious

possession and occupation of the land either since time immemorial or for the period prescribed therein, but most importantly, he must prove that the land is alienable public land. In the case at bar, petitioners failed to do so.

NOTES:

7. Gordula v. CA (284 SCRA 617, G.R. No. 127296 January 22, 1998) FACTS:

President Marcos issued Proclamation 573 (June 26, 1969), withdrawing from sale and settlement and setting aside as permanent forest reserves, subject to private rights, certain parcels of the public domain (11 parcels of forest reserves). These were primarily for use as watershed area.

The subject parcel of land in this case is Parcel No. 9 (Caliraya-Lumot River Forest Reserve). 3 years after

the land was segregated as a forest reserve, petitioner Edubigis Gordula filed with the Bureau of Lands, an application for a Free Patent over the land. He also declared the land for taxation purposes in his name. His petition for Free Patent was approved.

Meanwhile, the Republic conducted surveys in the area. The petitioner’s land was found to be located in

the saddle area of the watershed recreation for hydro-electric reservoir. Gordula sold the land to petitioner Celso V. Fernandez, Jr. and a Transfer Certificate was issued by the

Register of Deeds in Laguna. Several years after, he sold the lots to petitioner Nora Ellen Estrellado. On November 1987, the Republic filed a Complaint for Annulment of Free Patent and Cancellation of

titles. RTC ruled in favor of the petitioners. This was reversed by CA, and declared the Free Patents null and void. ISSUE: Did the CA err in annulling the Free Patent over the Lots? NO HELD:

We start with the proposition that the sovereign people, represented by their lawfully constituted government, have untrammeled dominion over the forests on their native soil. Forest lands, being the self-replenishing, versatile and all-important natural resource that they are, need to be reserved and saved to promote the people's welfare. By their very nature or by executive or statutory fiat, they are outside the commerce of man, unsusceptible of private appropriation in any form, and inconvertible into any character less than of inalienable public domain, regardless of their actual state, for as long as the reservation subsists and is not revoked by a subsequent valid declassification.

Petitioners do not contest the nature of the land in the case at bar. They, however, contend that Proclamation No. 573 itself recognizes private rights of landowners prior to the reservation. They claim to have established their private rights to the subject land.

We do not agree. No public land can be acquired by private persons without any grant, express or implied from the government; it is indispensable that there be a showing of the title from the state. The facts show that petitioner Gordula, did not acquire title to the subject land prior to its reservation under Proclamation No. 573. He filed his application for free patent only in January, 1973, more than three (3) years after the issuance of

III. Classification of Lands of the Public Domain Natural Resources First Set_Case Digests

Page | 28

Proclamation No. 573 in June, 1969. At that time, the land, as part of the Caliraya-Lumot River Forest Reserve, was no longer open to private ownership as it has been classified as public forest reserve for the public good.

NOTES:

8. REPUBLIC vs De Guzman (326 SCRA 574, G.R. No. 137887, 28 February 2000)

FACTS:

Following a dispute involving conflicting claims of imperfect title, a parcel of land in Cavite was awarded and registered to the de Guzmans. The State appealed this decision, stating that the de Guzmans did not submit proof of their fee simple title or possession in the manner and for the length of time required by law to justify the confirmation of an imperfect title. The State contended that in failing to do so, the de Guzmans failed to overcome the presumption that all lands are public domain belonging to the Republic.

Apparently, the land was released as agricultural (and therefore alienable) only in 1965. As early as 1938 the ancestors of de Guzman has been possessing the property, and has been for 63 years, or more than the required 30 years required by law. The CA ruled in favor of de Guzman. ISSUE: WON the awarding of the imperfect title to the de Guzmans by the CA was proper. NO. RULING:

In this case, the land was declared alienable only in 1965. Prior to that, the land was classified as forest land incapable of private appropriation. It was not registrable and any possession, no matter how lengthy, could not have converted it to private property. From the time of the land’s alienation and the de Guzman’s application for imperfect title, their occupancy had only been for 26 years. The 37 years their ancestors spent tilling the land could not be counted as one cannot acquire forest land through prescription, and should be disregarded. Thus, having failed the requirements set forth by law, the presumption that the land belongs to the State subsists. POLICY: Any occupation on land prior to its declaration as alienable land cannot be considered in the counting of the thirty year possession requirement.

NOTES:

9. CHAVEZ vs PEA (384 SCRA 152, G. R. No. 133250. July 9, 2002) FRANCISCO I. CHAVEZ, petitioner, vs. PUBLIC ESTATES AUTHORITY and AMARI COASTAL BAY DEVELOPMENT CORPORATION, respondents. FACTS: MARCOS REGIME: 1973 – The government, through the Commissioner of Public Highways signed a contract with the Construction and Development Corporation of the Philippines (“CDCP”) to reclaim certain foreshore and offshore areas of Manila Bay. 1977 – Pres. Marcos issued PD 1084 creating PEA. PEA was tasked “to reclaim land, including foreshore and submerged areas,” and “to develop, improve, acquire, x x x lease and sell any and all kinds of lands.”Marcos also

III. Classification of Lands of the Public Domain Natural Resources First Set_Case Digests

Page | 29

issued PD 1085 transferring to PEA the “lands reclaimed in the foreshore and offshore of the Manila Bay” under the Manila-Cavite Coastal Road and Reclamation Project (MCCRRP). 1981 – Marcos directed PEA to amend its contract with CDCP, so that “All future works in MCCRRP x x x shall be funded and owned by PEA.” Accordingly, PEA and CDCP executed a Memorandum of Agreement. CORY REGIME 1988 – Pres. Aquino issued special patent granting and transferring to PEA “the parcels of land so reclaimed under the MCCRRP containing a total area of 191.59Ha.” Hence TCTs were issued in the name of PEA covering what is called to be “Freedom Islands.” This comprises 157.84 Ha. RAMOS REGIME 1995 – PEA entered into a Joint Venture Agreement (“JVA”) with AMARI, a private corporation, to develop the Freedom Islands. The JVA also required the reclamation of an additional 250 ha. of submerged areas surrounding these islands. PEA and AMARI entered into the JVA through negotiation without public bidding. Pres FVR approved the JVA. 1996 – An investigation was made after Sen. Maceda denounced that JVA is the “grandmother of all scams.” Among the conclusions in the investigation is that the reclaimed lands PEA seeks to transfer to AMARI under the JVA are lands of the public domain which the government has not classified as alienable lands and therefore PEA cannot alienate these lands. In between these dates, the PEA and AMARI were on negotiations to amend the JVA. 1999 – The Amended JVA was signed. The Amended JVA involves “the development of the Freedom Islands and further reclamation of about 250 hectares comprising of foreshore and submerged areas. x x x,” plus an option “granted to AMARI to subsequently reclaim another 350 hectares x x x.” The Amended JVA also seeks to transfer to AMARI ownership of a certain portion of the freedom islands as well as the still submerged areas. ISSUES: (1) WON the freedom islands alienable or disposable lands of the public domain. - YES (2) WON AMARI, a private corporation, can acquire and own under the Amended JVA 367.5 hectares of reclaimed foreshore and submerged areas in Manila Bay in view of Sections 2 and 3, Article XII of the 1987 Constitution - NO. HELD:

(1) The Freedom Islands have become alienable and disposable because PD No. 1085, coupled with President Aquino’s actual issuance of a special patent covering the Freedom Islands, is equivalent to an official proclamation classifying the Freedom Islands as alienable or disposable lands of the public domain. PD No. 1085 and President Aquino’s issuance of a land patent also constitute a declaration that the Freedom Islands are no longer needed for public service. The Freedom Islands, which are reclaimed lands are thus alienable or disposable lands of the public domain, open to disposition or concession to qualified parties. Legal Basis: Commonwealth Act No. 141 of the Philippine National Assembly was made as basis for the ruling because this law continues to this day as the general law governing the classification and disposition of lands of the public domain. CA No. 141 empowers the President to classify lands of the public domain into “alienable or disposable” lands of the public domain, which prior to such classification are inalienable and outside the commerce of man. Section the law also authorizes the President to “declare what lands are open to disposition or concession.” Further, the law provides that the government can declare open for disposition or concession only lands that are “officially delimited and classified. Under the Public Land Act (CA 141, as amended), reclaimed lands are classified as alienable and disposable lands of the public domain:

Sec. 59. The lands DISPOSABLE under this title shall be classified as follows: (a) Lands reclaimed by the Government by dredging, filling, or other means; xxx.”

III. Classification of Lands of the Public Domain Natural Resources First Set_Case Digests

Page | 30

NOTE: You might get confused with this paragraph in the full text: Under Section 2, Article XII of the1987 Constitution, the foreshore and submerged areas of Manila Bay are part of the “lands of the public domain, waters x x x and other natural resources” and consequently “owned by the State.” As such, foreshore and submerged areas “shall not be alienated,” unless they are classified as “agricultural lands” of the public domain. The mere reclamation of these areas by PEA does not convert these inalienable natural resources of the State into alienable or disposable lands of the public domain. THERE MUST BE A LAW OR PRESIDENTIAL PROCLAMATION officially classifying these reclaimed lands as alienable or disposable and open to disposition or concession. Moreover, these reclaimed lands cannot be classified as alienable or disposable if the law has reserved them for some public or quasi-public use. Yung nasa paragraph na ‘to specific siya sa reclaimed lands. So actually, the PD issued by Marcos was in effect the law/proclamation, then the actual issuance of patent by Cory was the executive act.

(2) AMARI cannot acquire and own under the Amended JVA the 367.5 hectares of reclaimed foreshore and submerged areas in Manila Bay because for one, there was yet no classification and declaration that such areas are alienable or disposable. Further, private corporations or associations may not hold such alienable lands of the public domain except by lease.

There is no legislative or Presidential act classifying these submerged areas as alienable or disposable

lands of the public domain open to disposition. These submerged areas are not covered by any patent or certificate of title. There can be no dispute that these submerged areas form part of the public domain, and in their present state are inalienable and outside the commerce of man. Until reclaimed from the sea, these submerged areas are, under the Constitution, “waters x x x owned by the State,” forming part of the public domain and consequently inalienable. Only when actually reclaimed from the sea can these submerged areas be classified as public agricultural lands, which under the Constitution are the only natural resources that the State may alienate. Once reclaimed and transformed into public agricultural lands, the government may then officially classify these lands as alienable or disposable lands open to disposition. Thereafter, the government may declare these lands no longer needed for public service. Only then can these reclaimed lands be considered alienable or disposable lands of the public domain and within the commerce of man. Legal Basis: Sections 2 and 3, Article XII of the 1987 Constitution which state that:

“Section 2. All lands of the public domain, waters, minerals, coal, petroleum, and other mineral oils, all forces of potential energy, fisheries, forests or timber, wildlife, flora and fauna, and other natural resources are owned by the State. With the exception of agricultural lands, all other natural resources shall not be alienated.xx x x x Section 3. x x x Alienable lands of the public domain shall be limited to agricultural lands. Private

corporations or associations may not hold such alienable lands of the public domain except by lease for a period not exceeding twenty five years, renewable for not more than twenty-five years, and not to exceed one thousand hectares in area. Citizens of the Philippines may lease not more than five hundred hectares, or acquire not more than twelve hectares thereof by purchase, homestead, or grant.

CA No. 141 Sec. 56. The lands disposable under this title shall be classified as follows: (a) Lands reclaimed by the Government by dredging, filling, or other means; (b) Foreshore; (c) Marshy lands or lands covered with water bordering upon the shores or banks of navigable lakes or rivers; (d) Lands not included in any of the foregoing classes. x x x. Sec. 58. The lands comprised in classes (a), (b), and (c) of section fifty-six shall be disposed of to private parties by lease only and not otherwise. xxx

III. Classification of Lands of the Public Domain Natural Resources First Set_Case Digests

Page | 31

SC’s SUMMARIZED CONCLUSIONS: 1. The 157.84 ha of reclaimed lands comprising the Freedom Islands, now covered by certificates of title in the name of PEA, are alienable lands of the public domain. PEA MAY LEASE these lands to private corporations BUT MAY NOT SELL or transfer ownership of these lands to private corporations. PEA may only sell these lands to Philippine citizens, subject to the ownership limitations in the 1987 Constitution and existing laws. Rationale: The Amended JVA covers a reclamation area of 750 ha. Only 157.84 hectares of the 750-hectare reclamation project have actually been reclaimed, and the rest of the 592.15 hectares are still submerged areas forming part of Manila Bay. Why Lease Only: Government reclaimed, foreshore and marshy public lands for non-agricultural purposes retain their inherent potential as areas for public service. This is the reason the government prohibited the sale, and only allowed the lease, of these lands to private parties. The State always reserved these lands for some future public service. 2. The 592.15 ha of submerged areas of Manila Bay remain inalienable natural resources of the public domain until classified as alienable or disposable lands open to disposition and declared no longer needed for public service. The government can make such classification and declaration only after PEA has reclaimed these submerged areas. Only then can these lands qualify as agricultural lands of the public domain, which are the only natural resources the government can alienate. In their present state, the 592.15 hectares of submerged areas are inalienable and outside the commerce of man. 3. Since the Amended JVA seeks to transfer to AMARI, a private corporation, ownership of 77.34 hectares of the Freedom Islands, such transfer is void for being contrary to Section 3, Article XII of the 1987 Constitution which prohibits private corporations from acquiring any kind of alienable land of the public domain. 4. Since the Amended JVA also seeks to transfer to AMARI ownership of 290.156 ha of still submerged areas of Manila Bay, such transfer is void for being contrary to Section 2, Article XII of the 1987 Constitution which prohibits the alienation of natural resources other than agricultural lands of the public domain. PEA may reclaim these submerged areas. Thereafter, the government can classify the reclaimed lands as alienable or disposable, and further declare them no longer needed for public service. Still, the transfer of such reclaimed alienable lands of the public domain to AMARI will be void in view of Section 3, Article XII of the 1987 Constitution which prohibits private corporations from acquiring any kind of alienable land of the public domain.

The Amended JVA was declared NULL and VOID AB INITIO for being violative of the Constitution.

NOTES:

9.-B Chavez vs PEA (403 SCRA 1, G. R. No. 133250. May 6, 2003) FRANCISCO I. CHAVEZ, petitioner, vs. PUBLIC ESTATES AUTHORITY and AMARI COASTAL BAY DEVELOPMENT CORPORATION, respondents. FACTS:

Several motions were filed by Amari in connection with the Court’s decision on July 9, 2002. Likewise, PEA filed a motion for reconsideration from SC’s decision. All these motions were altogether opposed by petitioner Chavez.

III. Classification of Lands of the Public Domain Natural Resources First Set_Case Digests

Page | 32

AMARI filed its motion to SC, contending among others the SC Decision should be made to apply prospectively, not retroactively to cover the Amended JVA. Amari argues that the existence of a statute or executive order prior to its being adjudged void is an operative fact to which legal consequences are attached. Amari now claims that “assuming arguendo that PDs. 1084 and 1085, and EOs 525 and 654 are inconsistent with the 1987 Constitution, the limitation imposed by the Decision on these decrees and executive orders should only be applied prospectively from the finality of the Decision.” Amari likewise asserts that a new doctrine of the Court cannot operate retroactively if it impairs vested rights. (bale new doctrine ang nasa decision dun sa naunang Chavez case. Sabi sa first case that the petition was a case of first impression. All previous decisions of the Court involving Section 3, Article XII of the 1987 Constitution, or its counterpart provision in the 1973 Constitution, covered agricultural lands sold to private corporations which acquired the lands from private parties. In the instant case, AMARI seeks to acquire from PEA, a public corporation, reclaimed lands and submerged areas for non-agricultural purposes by purchase under PD No. 1084 (charter of PEA) and Title III of CA No. 141.) ISSUE: WON the SC decision declaring the Amended JVA null and void should be upheld - YES HELD:

The prevailing doctrine before, during and after the signing of the Amended JVA is that private corporations cannot hold, except by lease, alienable lands of the public domain. This is one of the two main reasons why the Decision annulled the Amended JVA. The other main reason is that submerged areas of Manila Bay, being part of the sea, are inalienable and beyond the commerce of man, a doctrine that has remained immutable since the Spanish Law on Waters of 1886.

Clearly, the Decision merely reiterates, and does not overrule, any existing judicial doctrine. Even on the

characterization of foreshore lands reclaimed by the government, the Decision does not overrule existing law or doctrine. Since the adoption of the Regalian doctrine in this jurisdiction, the sea and its foreshore areas have always been part of the public domain. And since the enactment of Act No. 1654 on May 18, 1907 until the effectivity of the 1973 Constitution, statutory law never allowed foreshore lands reclaimed by the government to be sold to private corporations. The 1973 and 1987 Constitution enshrined and expanded the ban to include any alienable land of the public domain.

Despite the nullity of the Amended JVA, Amari is not precluded from recovering from PEA in the proper

proceedings, on a quantum meruit basis, whatever Amari may have incurred in implementing the Amended JVA prior to its declaration of nullity.

NOTES:

10. Republic v. Imperial (G.R. No. 130906, February 11, 1999) REPUBLIC OF THE PHILIPPINES represented by the DIRECTOR, LANDS MANAGEMENT BUREAU, petitioner, vs. FELIX S. IMPERIAL JR., FELIZA S. IMPERIAL, ELIAS S. IMPERIAL, MIRIAM S. IMPERIAL, LOLITA ALCAZAR, SALVADOR ALCAZAR, EANCRA CORPORATION, and the REGISTER OF DEEDS of LEGASPI CITY, respondents. FACTS:

In 1917, by result of a cadastral proceeding, Elias Imperial was issued a OCT over a parcel of land in Legazpi City. This parcel of land was later subdivided to his family by way of numerous TCTs. In 1994, the Director of the Lands Management Bu requested to the OSG that the OCT issued in Imperial’s favor be cancelled and that the land covered by it be returned to the State, as it was, allegedly, foreshore land. The DENR conducted a survey and conceded that the land indeed had features of foreshore land.

III. Classification of Lands of the Public Domain Natural Resources First Set_Case Digests

Page | 33

Petitioner insists that the parcels of land in question are foreshore lands, and hence, inalienable and incapable of registration. Consequently, the certificates of title covering said land are void ab initio.The plaintiff further contended that under Public Land Act No. 141, as amended, such land shall be disposed of to private parties by lease only and not otherwise as soon as the President upon recommendation of the Secretary of Agriculture and Natural Resources, now DENR, shall declare that the same are not necessary for public services and are open to disposition. Imperial, however, contended that the cadastral proceedings in 1917 (and the subsequent Torrens registration) were binding even against the State. During trial, the RTC failed to rule upon the character of the land, and case was dismissed on appeal because the OSG failed to file their briefings in the alloted time, despite asking for numerous extensions. ISSUE: WON the land was foreshore land. NOT SURE – the case was remanded to CA to determine WON the lands were foreshore. HELD: a. Procedural matters: OSG took extensions for granted but they must be allowed to present their briefs for public interest etc.

The SC observed that the Office of the Solicitor General (OSG) regularly presents motions for extension of time to file pleadings, taking for granted the court's leniency in granting the same. Instead of contributing to the swift administration of justice as an instrumentality of the State, the OSG contributes to needless delays in litigation. Despite the numerous cases that need the OSG's time and attention, equal importance should be allotted to each and every case. Deadlines must be respected and court warnings not taken lightly.

However, the need, therefore, to determine once and for all whether the lands subject of petitioner's

reversion efforts are foreshore lands constitutes good and sufficient cause for relaxing procedural rules and granting the third and fourth motions for extension to file appellant's brief. Petitioner's appeal presents an exceptional circumstance impressed with public interest and must then be given due course. b. On foreshore lands and role of LMB/DENR note, quizzable: Foreshore land is a part of the alienable land of the public domain and may be disposed of only by lease and not otherwise. It was defined as "that part (of the land) which is between high and low water and left dry by the flux and reflux of the tides." It is also known as "a strip of land that lies between the high and low water marks and, is alternatively wet and dry according to the flow of the tide."

The classification of public lands is a function of the executive branch of government, specifically the director of lands (now the director of the Lands Management Bureau). The decision of the director of lands when approved by the Secretary of the Department of Environment and Natural Resources (DENR) as to questions of fact is conclusive upon the court. The principle behind this ruling is that the subject has been exhaustively weighed and discussed and must therefore be given credit. This doctrine finds no application, however, when the decision of the director of lands is revoked by, or in conflict with that of, the DENR Secretary.

The contradictory views of the Director of Lands and the DENR, Region V, Legazpi City, on the true nature of the land, which contradiction was neither discussed nor resolved by the RTC, cannot be the premise of any conclusive classification of the land involved.

NOTES:

III. Classification of Lands of the Public Domain Natural Resources First Set_Case Digests

Page | 34

11. Palomo v. CA (266 SCRA 392, G.R. No. 130906) FACTS:

In 1913, some 440,530 square meters of land in Albay were reserved for provincial park purposes by virtue of EO No. 40. Of said area, 15 parcels of land were registered in the name of Diego Palomo by the Court of First Instance.

In 1937, Diego Palomo donated these lands to his heirs Ignacio and Carmen Palomo. Claiming that the aforesaid original certificates of title were lost during the Japanese occupation, Ignacio Palomo filed a petition for reconstitution with the Court of First Instance of Albay on May 30, 1950. The Register of Deeds of Albay issued Transfer Certificates of Title Nos. 3911, 3912, 3913 and 3914 sometime in October 1953.

On July 10, 1954, President Magsaysay issued Proclamation No. 47 converting the area covered by EO 40

into the Tiwi Hot Spring National Park. The Palomos contended that they have been in possession of the subject lands and have introduced improvements thereon. ISSUE: Were the Original Certificate of Titles issued to the petitioners valid? –NO

HELD Before the Treaty of Paris in 1899, the lands, whether agricultural, mineral, or forest were under the

exclusive patrimony and dominion of the Spanish crown.Private ownership of land could only be acquired through royal concessions which were documented in various forms, such as (1) Titulo Real or Royal Grant," (2) Concession Especial or Special Grant, (3) Titulo de Compra or Title by Purchase and (4) Informacion Posesoria or Possessory Information title obtained under the Spanish Mortgage Law or under the Royal Decree of January 26, 1889.

The decision of the CFI relied upon by petitioners were not signed by the judge but were merely certified

copies of notification to Diego Palomo bearing the signature of the Clerk of Court. More importantly, the lands in question were not classified as alienable lands. Since the lands were made

part of a reservation for provincial park purposes, they form part of the forest zone. Thus, they cannot be the valid subject of alienation

NOTES:

12. Chavez v. PEA (G.R. No. 133250, November 11, 2003) FACTS:

This petition asked the Court to legitimize a government contract that conveyed to a private entity 157.84 hectares of reclaimed public lands along Roxas Boulevard in Metro Manila at the negotiated price of P1,200 per square meter. However, published reports place the market price of land near that area at that time at a high of P90,000 per square meter. The difference in price is a staggering P140.16billion, equivalent to the budget of the entire Judiciary for seventeen years and more than three times the Marcos Swiss deposits that this Court forfeited in favor of the government. Public Estates Authority (PEA), under the JVA, obligated itself to convey title and possession over the Property, consisting of approximately One Million Five Hundred Seventy Eight Thousand Four Hundred Forty One (1,578,441) Square Meters for a total consideration of One Billion Eight Hundred Ninety Four Million One Hundred Twenty Nine Thousand Two Hundred (P1,894,129,200.00) Pesos, or a price of One Thousand Two Hundred (P1,200.00) Pesos per square meter.

III. Classification of Lands of the Public Domain Natural Resources First Set_Case Digests

Page | 35

ISSUE: WON stipulations in the Amended JVA for the transfer to AMARI of lands, reclaimed or to be reclaimed, violate the Constitution? – YES HELD:

On November 29, 1919, the Philippine Legislature enacted Act No. 2874, the Public Land Act. The salient provisions of Act No. 2874, on reclaimed lands, were as follows:

"Sec. 6. The Governor-General, upon the recommendation of the Secretary of Agriculture and Natural Resources, shall from time to time classify the lands of the public domain into – (a) Alienable or disposable, (b) Timber, and (c) Mineral lands, x x x. Sec. 56. The lands disposable under this title shall be classified as follows:

(a) Lands reclaimed by the Government by dredging, filling, or other means; (b) Foreshore; (c) Marshy lands or lands covered with water bordering upon the shores or banks of navigable lakes or rivers; (d) Lands not included in any of the foregoing classes.

x x x. Sec. 58. The lands comprised in classes (a), (b), and (c) of section fifty-six shall be disposed of to private parties by lease only and not otherwise, as soon as the Governor-General, upon recommendation by the Secretary of Agriculture and Natural Resources, shall declare that the same are not necessary for the public service and are open to disposition under this chapter. The lands included in class (d) may be disposed of by sale or lease under the provisions of this Act." (Emphasis supplied) Section 6 of Act No. 2874 authorized the Governor-General to "classify lands of the public domain into x x

x alienable or disposable" lands. Section 7 of the Act empowered the Governor-General to "declare what lands are open to disposition or concession." Section 8 of the Act limited alienable or disposable lands only to those lands which have been "officially delimited and classified.

1. The 157.84 hectares of reclaimed lands comprising the Freedom Islands, now covered by certificates of title in the name of PEA, are alienable lands of the public domain. PEA may lease these lands to private corporations but may not sell or transfer ownership of these lands to private corporations. PEA may only sell these lands to Philippine citizens, subject to the ownership limitations in the 1987 Constitution and existing laws. 2. The 592.15 hectares of submerged areas of Manila Bay remain inalienable natural resources of the public domain until classified as alienable or disposable lands open to disposition and declared no longer needed for public service. The government can make such classification and declaration only after PEA has reclaimed these submerged areas. Only then can these lands qualify as agricultural lands of the public domain, which are the only natural resources the government can alienate. In their present state, the 592.15 hectares of submerged areas are inalienable and outside the commerce of man. 3. Since the Amended JVA seeks to transfer to AMARI, a private corporation, ownership of 77.34 hectares

110 of the Freedom Islands, such transfer is void for being contrary to Section 3, Article XII of the

1987 Constitution which prohibits private corporations from acquiring any kind of alienable land of the public domain. 4. Since the Amended JVA also seeks to transfer to AMARI ownership of 290.156 hectares

111 of still

submerged areas of Manila Bay, such transfer is void for being contrary to Section 2, Article XII of the

III. Classification of Lands of the Public Domain Natural Resources First Set_Case Digests

Page | 36

1987 Constitution which prohibits the alienation of natural resources other than agricultural lands of the public domain. PEA may reclaim these submerged areas. Thereafter, the government can classify the reclaimed lands as alienable or disposable, and further declare them no longer needed for public service. Still, the transfer of such reclaimed alienable lands of the public domain to AMARI will be void in view of Section 3, Article XII of the 1987 Constitution which prohibits private corporations from acquiring any kind of alienable land of the public domain. Clearly, the Amended JVA violates glaringly Sections 2 and 3, Article XII of the 1987 Constitution. Under

Article 1409112

of the Civil Code, contracts whose "object or purpose is contrary to law," or whose "object is outside the commerce of men," are "inexistent and void from the beginning." The Court must perform its duty to defend and uphold the Constitution, and therefore declares the Amended JVA null and void ab initio.

NOTES:

IV. Transfer of Private Lands Natural Resources First Set_Case Digests

Page | 37

IV. TRANSFER OF PRIVATE LANDS

1. Ramirez vs Vda. De Ramirez (111 SCRA 704, February 15, 1982) TESTATE ESTATE OF JOSE EUGENIO RAMIREZ, MARIA LUISA PALACIOS, Administratrix, petitioner-appellee, vs. MARCELLE D. VDA. DE RAMIREZ, ET AL., oppositors, JORGE and ROBERTO RAMIREZ, legatees, oppositors- appellants. FACTS:

Jose Ramirez, a Filipino national, died with only his widow as his compulsory heir. Maria Luisa Palacios was appointed administratrix of Ramirez’s estate, who submitted an inventory of the estate.

Palacios, as administratrix, submitted a project of partition which provides that the property of the deceased is divided into 2 parts, where 1 part will go to the widow, while the other part shall go to his grandnephews (Jorge and Roberto, among the respondents). The ‘other portion’ that will go to the grandnephews is charged 1/3 with the widow’s usufruct, while the remaining 2/3 with a usufruct in favor of his companion (Wanda).

Grandnephews Jorge and Roberto opposed the partition on these grounds: o Provisions of vulgar substitution in favor of Wanda with respect to the widow’s usufruct, and in

favor of Juan Pablo and Horacio with respect to Wanda’s usufruct are invalid o Fideicommissary substitutions are also invalid as the first heirs and second heirs or substitutes

are not related with the 1st

degree o Grant of usufruct in Wanda’s, an alien, favor is violative of the Constitution o The proposed partition is violative of the testator’s express will

ISSUE: WON the partition is invalid as to: 1) the widow’s legitime, 2) substitutions, 3) usufruct of Wanda HELD: Widow’s legitime

- The appellants do not question the legality of Marcelle being entitled to ½ of the estate in full ownership. However, what they are averring is the fact that the wife is entitled 1/3

usufruct over the ‘free portion’.

The Court held that the lower court erred for Marcelle who is entitled to ½ of the estate as her legitime, and which is more than what she us given under the will. Marcelle is not entitled to have any additional share in the estate, and to give her more than her legitime will run counter to the testator’s intention.

Substitutions

- The Court declared the vulgar substitution entered into by Wanda in favor of Juan Pablo and Horacio, as valid because the dying before the testator is not the only case for vulgar substitution. It includes, refusal or incapacity to accept the inheritance. (Art 859 of the Civil Code)

- However, the fideicomissary substitutions are declared by the Court as void because the substitutes are not relate to Wanda, who is the original heir. Art 863 of the Civil Code provides that fideicommissary substitution is valid provided that such substitution does not go beyond 1 degree from the original heir instituted. Thus, the fideicommissary can only be either a child or parent of the first heir—but in this case, Wanda is neither the parent nor child of the fideicommissaries (Juan Pablo and Horacio). Furthermore, the Court adds that there is no absolute duty imposed on Wanda to transmit usufruct to the substitutes. (Art 856 and 867 of the Civil Code)

Usufruct of Wanda

- The Court upheld the usufruct in favor of Wanda because a usufruct, albeit a real right, does not vest title to the land in the usufructuary and it is the vesting of title to land in favor of aliens which is proscribed by the Constitution.

Name:

IV. Transfer of Private Lands Natural Resources First Set_Case Digests

Page | 38

Decision: - ½: for his widow as her legitime - ½: the free portion to Roberto & Jorge Ramirez in naked ownership, and the usufruct to Wanda with a

simple substitution in favor of Juan Pablo and Horace

NOTES:

2. Phil. Banking Corp. vs Lui She (21 SCRA 52, G.R. No. L-17587, December18, 1967) PHILIPPINE BANKING CORPORATION, representing the estate of JUSTINIA SANTOS Y CANON FAUSTINO, deceased, plaintiff-appellant, vs. LUI SHE, in her own behalf and as administratrix of the intestate estate of Wong Heng, deceased, defendant-appellant. FACTS:

Justina Santos and her sister Lorenza were the owners of a piece of land in Manila located on Rizal Avenue and opens into Florentino Torres street at the back and Katubusan street on one side. In it are two residential houses with entrance on Florentino Torres street and the Hen Wah Restaurant with entrance on Rizal Avenue. The sisters lived in one of the houses, while Wong Heng, a Chinese, lived with his family in the restaurant. Wong had been a long-time lessee of a portion of the property, having a monthly rental of P2,620.

On Sept. 22, 1957 Justina Santos became the owner of the entire property as her sister died with no other heir. Then already well advanced in yrs., being at the time 90 yrs. old, blind, crippled and an invalid, she was left with no other relative to live with. Her only companions in the house were her 17 dogs and 8 maids

Wong himself was the trusted man to whom she delivered various amounts for safekeeping, including rentals from her property at the corner of Ongpin and Salazar streets and the rentals which Wong himself paid as lessee of a part of the Rizal Avenue property. Wong also took care of the payment, in her behalf, of taxes, lawyers’ fees, funeral expenses, masses, salaries of maids and security guard, and her household expenses.

“In grateful acknowledgement of the personal services of the Lessee to her,”Santos executed on NoV 15, 1957, a contract of lease in favor of Wong, covering the portion then already leased to him and another portion fronting Florentino Torres street. The lease was for 50 yrs., although the lessee was given the right to withdraw at any time from the agreement; the monthly rental was P3,120. The contract covered only a portion of the land. 10 days later (NoV 25), the contract was amended so as to make it cover the entire property, including the portion on which the house of Santos stood, at an additional monthly rental of P360. For his part Wong undertook to pay, out of the rental due from him, an amount not exceeding P1,000 a month for the food of her dogs and the salaries of her maids.

On Dec. 21 she executed contract giving Wong the option to buy the leased premises for P120,000, payable within ten yrs. at a monthly installment of P1,000. The option, written in Tagalog, imposed on him the obligation to pay for the food of the dogs and the salaries of the maids in her household, the charge not to exceed P1,800 a month and was conditioned on his obtaining Philippine citizenship, a petition for which was then pending in the CFI of Rizal. However, this application for naturalization was withdrawn when it was discovered that he was not a resident of Rizal. On Oct 28, 1958 she filed a petition to adopt him and his children on the erroneous belief that adoption would confer on them Philippine citizenship. The error was discovered and the proceedings were abandoned.

On NoV 18, 1958 she executed 2 other contracts, one extending the term of the lease to 99 yrs., and another fixing the term of the option at 50 yrs.

In two wills executed on Aug 24 and 29, 1959, she bade her legatees to respect the contracts she had entered into with Wong, but in a codicil (a post-script) of a later date (NoV 4, 1959) she appears to have a

IV. Transfer of Private Lands Natural Resources First Set_Case Digests

Page | 39

change of heart. Claiming that the various contracts were made by her because of machinations and inducements practiced by him, she now directed her executor to secure the annulment of the contracts.

Note: both Santos and Wong died while case was pending, and both were replaced by Phil. Banking Corp. (PBC) and Lui She (the wife) respectively. Lower court declared contracts null and void save for the lease contract executed Nov 15, 1957. Both parties appealed. Petitioner’s arguments

the contracts were obtained by Wong “through fraud, misrepresentation, inequitable conduct, undue influence and abuse of confidence and trust of and (by) taking advantage of the helplessness of the plaintiff and were made to circumvent the constitutional prohibition prohibiting aliens from acquiring lands in the Philippines and also of the Philippine Naturalization Laws.”

lease contract should have been annulled with the 4 other contracts because it lacks mutuality; because it included a portion which, at the time, was in custodia legis because the contract was obtained in violation of the fiduciary relations of parties; because her consent was obtained through undue influence, fraud and misrepresentation; and because the lease contract, like the other contracts, is absolutely simulated.

Respondent’s arguments

admitted he enjoyed her trust and confidence as proof of which he volunteered the information that, in addition to the sum of P3,000 which he said she had delivered to him for safekeeping, another sum of P22,000 had been deposited in a joint account which he had with one of her maids. But he denied having taken advantage of her trust in order to secure the execution of the contracts in question.

ISSUES: 1. WON the provision “lessee may at any time withdraw from the agreement” makes the contract invalid. 2. WON the property cannot be leased for being in custodio legis, thus making contract of lease void. 3. WON contracts are void for being violative of fiduciary relationship, contrary to Art. 16468. 4. WON consent of Santos was valid. 5. WON the contracts are void for trying to circumvent Philippine laws against alienation of property to foreigners.

HELD: 1. NO. The stipulation in the contract does not violate Art.1308 CC, for it is considered as a resolutory condition to the contract, permitting the cancellation of such contract. Reasoning : It is claimed that the stipulation offends Art. 1308 CC which provides that "the contract must bind both contracting parties; its validity or compliance cannot be left to the will of one of them." As was held in Taylor v Uy Tiong Piao, “art. 1256 [now art. 1308] CC in our opinion creates no impediment to the insertion in a contract for personal service of a resolutory condition permitting the cancellation of the contract by one of the parties. Such a stipulation, as can be readily seen, does not make either the validity or the fulfillment of the contract dependent upon the will of the party to whom is conceded the privilege of cancellation; for where the contracting parties have agreed that such option shall exist, the exercise of the option is as much in the fulfillment of the contract as any other act which may have been the subject of agreement, Indeed, the cancellation of a contract in accordance with conditions agreed upon beforehand is fulfillment.” The right of the lessee to continue the lease or to terminate it is so circumscribed by the term of the contract that it cannot be said that the continuance of the lease depends upon his will. At any rate, even if no term had been fixed in the agreement, this case would at most justify the fixing of a period but not the annulment of the contract. 2. NO. Ownership of the property (that used to belong to her sister) transferred to Santos upon the death of the sister, thus enabling her to bind such in contracts Reasoning: Santos became the owner of the entire property upon the death of her sister Lorenza on September 22, 1957 by force of art. 777 CC. Hence, when she leased the property on NoV15, she did so already as owner thereof.

IV. Transfer of Private Lands Natural Resources First Set_Case Digests

Page | 40

3. NO. Art.1646 is not violated for Wong was never an agent for Santos. Reasoning: The relationship of the parties, although admittedly close and confidential, did not amount to an agency so as to bring the case within the prohibition of the law. Atty. Yumol testified that she signed the lease contract in the presence of her close friend and her maid who was constantly by her side. Any of them could have testified on the undue influence that Wong supposedly wielded over Santos, but neither of them was presented as a witness. The truth is that even after giving his client time to think the matter over, the lawyer could not make her change her mind. 4. YES. Consent given by her voluntarily and on well-informed bases, as shown by the testimonies of her lawyer, is valid. Reasoning: There is no merit in the claim that her consent to the lease contract, as well as to the rest of the contracts in question, was given out of a mistaken sense of gratitude to Wong who, she was made to believe, had saved her and her sister from a fire that destroyed their house during the liberation of Manila. As it was with the lease contract so it was with the rest of the contracts – the consent of Santos was given freely and voluntarily. 5. YES, contracts are void. The collection of contracts, as well as the intentions of Santos, shows a scheme to circumvent the Constitutional prohibition against the transfer of land of aliens, thus making each contract void. Reasoning: “The illicit purpose then becomes the illegal cause rendering the contracts void.” Taken singly, the contracts show nothing that is necessarily illegal, but considered collectively, they reveal an insidious pattern to subvert by indirection what the Constitution directly prohibits. To be sure, a lease to an alien for a reasonable period is valid, so is an option giving an alien the right to buy real property on condition that he is granted Philippine citizenship. However, if an alien is given not only a lease of, but also an option to buy, a piece of land, by virtue of which the Filipino owner cannot sell or otherwise dispose of his property, this to last for 50 years, then it becomes clear that the arrangement is a virtual transfer of ownership whereby the owner divests himself in stages not only of the right to enjoy the land (jus possidendi, jus utendi, jus fruendi and jusabutendi), but also of the right to dispose of it (jus disponendi) rights the sum total of which make up ownership. It is just as if today the possession is transferred, tomorrow, the use, the next day, the disposition, and so on, until ultimately all the rights of which ownership is made up are consolidated in an alien. And yet this is just exactly what the parties in this case did within this pace of one year, with the result that Santos’ ownership of her property was reduced to a hollow concept. If this can be done, then the Constitutional ban against alien landholding in the Phil, as announced in Krivenko, is in grave peril.

NOTES:

3. Republic vs Quasha (46 SCRA 160, G.R. No. L-30299 August 17, 1972) REPUBLIC OF THE PHILIPPINES and/or THE SOLICITOR GENERAL petitioners, vs. WILLIAM H. QUASHA, respondent. FACTS:

William Quasha is an American who has owned a parcel of land in Makati since 1954. In 1974, the Parity Amendment was to expire, and along with it the rights acquired by US citizens. The Parity Amendment was signed after the war in 1946, and allowed US citizens to participate in the disposition, development, utilization etc etc of Filipino lands. According to Quasha the Amendment created exceptions to the prohibition of non-Filipinos to acquire Filipino land without hereditary succession. Quasha was worried that his rights to own his property would expire along with the Amendment.

The SocGen later contended that Quasha’s land constituted private agricultural land, and that his acquisition violated Sec 5, Article XIII of the Constitution which prohibits the transfer of such lands to non-Filipinos

IV. Transfer of Private Lands Natural Resources First Set_Case Digests

Page | 41

except by hereditary succession. The SocGen also said that even if Quasha acquired the land validly, his rights to it would expire with the Parity Amendment. The CFI ruled in favor of Quasha, declaring him to have the right to retain ownership over the land even if the Parity Amendment expires. The State appealed to the SC. ISSUE: WON Quasha should be allowed to retain ownership of his property by virtue of the Parity Amendment? NO. HELD: When the Republic of the Philippines was established in 1946, all lands were effectively reserved for Filipinos. The SC looked at the intention of the framers of the Constitution and the drafters of the Parity Agreement and found that the parity secured for US citizens extended only in two matters:

1. the exploitation, dev’t and utilization of lands of public domain and other natural resources of the PHL and

2. the operation of public utilities The Parity Amendment merely reopened the aforementioned activities to US citizens, but it did not allow them to acquire or exploit private agricultural lands. Quasha cannot find refuge in the Parity Amendment as it itself has prescribed that its effects were to expire in 1974. The decision of the CFI was reversed, and the decision with what to do with Quasha’s property was deferred to Congress. POLICY: The Parity Agreement did not allow US citizens to acquire land without hereditary succession.

NOTES:

4. Krivenko vs ROD (79 Phil. 461, G.R. No. L-630, November 15, 1947) ALEXANDER A. KRIVENKO, petitioner-appellant, vs. THE REGISTER OF DEEDS, CITY OF MANILA, respondent and appellee. FACTS:

Alexander A. Krivenko, an alien, bought a residential lot from the Magdalena Estate, Inc, the registration of which was interrupted by the war. In May, 1945, he sought to accomplish said registration but was denied by the register of deeds of Manila on the ground that, being an alien, he cannot acquire land in this jurisdiction.

Krivenko then brought the case to the Court of First Instance of Manila but the court rendered judgment sustaining the refusal of the register of deeds, from which Krivenko appealed to this Court. ISSUE: WON an alien under our Constitution may acquire residential land. NO. HELD:

The phrase "public agricultural lands" appearing in section 1 of Article XIII of the Constitution must be construed as including residential lands.

In section 9 of said Commonwealth Act No. 141, "alienable or disposable public lands" which are the same "public agriculture lands" under the Constitution, are classified into agricultural, residential, commercial, industrial and for other puposes. The term "public agricultural lands" in the same Act classifies "public agricultural lands" for purposes of alienation or disposition, into lands that are stricly agricultural or actually devoted to cultivation for

IV. Transfer of Private Lands Natural Resources First Set_Case Digests

Page | 42

agricultural puposes; lands that are residential; commercial; industrial; or lands for other purposes. The fact that these lands are made alienable or disposable in favor of Filipino citizens, is a conclusive indication of their character as public agricultural lands under said statute and under the Constitution.

Under section 1 of Article XIII of the Constitution, "natural resources, with the exception of public agricultural land, shall not be aliented," and with respect to public agricultural lands, their alienation is limited to Filipino citizens. But this constitutional purpose conserving agricultural resources in the hands of Filipino citizens may easily be defeated by the Filipino citizens themselves who may alienate their agricultural lands in favor of aliens. It is partly to prevent this result that section 5 is included in Article XIII, and it reads as follows:

Sec. 5. Save in cases of hereditary succession, no private agricultural land will betransferred or assigned except to individuals, corporations, or associations qualified to acquire or hold lands of the public domain in the Philippines. This constitutional provision closes the only remaining avenue through which agricultural resources may leak into aliens' hands. It would certainly be futile to prohibit the alienation of public agricultural lands to aliens if, after all, they may be freely so alienated upon their becoming private agricultural lands in the hands of Filipino citizens. Undoubtedly, as above indicated, section 5 is intended to insure the policy of nationalization contained in section 1. Both sections must, therefore, be read together for they have the same purpose and the same subject matter. It must be noticed that the persons against whom the prohibition is directed in section 5 are the very same persons who under section 1 are disqualified "to acquire or hold lands of the public domain in the Philippines." And the subject matter of both sections is the same, namely, the non-transferability of "agricultural land" to aliens. Since "agricultural land" under section 1 includes residential lots, the same technical meaning should be attached to "agricultural land under section 5.

NOTES:

5. Register of Deeds Rizal vs. Ung Sui Si Temple (G.R. No. L-6776, May 21, 1955) THE REGISTER OF DEEDS OF RIZAL, petitioner-appellee, vs. UNG SIU SI TEMPLE, respondent-appellant. FACTS

Jesus Dy, a Filipino citizen, conveyed a parcel of residential land, in favor of the unregistered religious organization “Ung Siu Si Temple”, operating through three trustees all of Chinese nationality. The donation was duly accepted by Yu Juan, of Chinese nationality, founder and deaconess of the Temple, acting in representation and in behalf of the latter and its trustees.

However, the registry of deeds refused to accept for record the deed of donation. Which was upheld by

the CFI, it appearing from the record of the Consulta that UNG SIU SI TEMPLE is a religious organization whose deaconess, founder, trustees and administrator are all Chinese citizens, this Court is of the opinion and so hold that in view of the provisions of the sections 1 and 5 of Article XIII of the Constitution of the Philippines limiting the acquisition of land in the Philippines to its citizens, or to corporations or associations at least sixty per centum of the capital stock of which is owned by such citizens.

Respondent alleged that acquisition of the land in question, for religious purposes, is authorized and

permitted by Act No. 271 of the old Philippine Commission, and that the refusal of the Register of Deeds violates the freedom of religion clause of our Constitution. ISSUE: WON the deed of donation is valid? NO.

IV. Transfer of Private Lands Natural Resources First Set_Case Digests

Page | 43

HELD: The provisions of Act No. 271 must be deemed repealed since the Constitution was enacted, in so far as

incompatible therewith. In providing that – “Save in cases of hereditary succession, no private agricultural land shall be transferred or assigned

except to individuals, corporations or associations qualified to acquire or hold lands of the public domain in the Philippines.”

The Constitution makes no exception in favor of religious associations. Neither is there any such saving

found in sections 1 and 2 of Article XIII, restricting the acquisition of public agricultural lands and other natural resources to "corporations or associations at least sixty per centum of the capital of which is owned by such citizens" (of the Philippines).

The fact that the appellant religious organization has no capital stock does not suffice to escape the

Constitutional inhibition, since it is admitted that its members are of foreign nationality. The purpose of the sixty per centum requirement is obviously to ensure that corporations or associations allowed to acquire agricultural land or to exploit natural resources shall be controlled by Filipinos; and the spirit of the Constitution demands that in the absence of capital stock, the controlling membership should be composed of Filipino citizens.

As to the complaint that the disqualification under article XIII is violative of the freedom of religion guaranteed by Article III of the Constitution, we are by no means convinced (nor has it been shown) that land tenure is indispensable to the free exercise and enjoyment of religious profession or worship; or that one may not worship the Deity according to the dictates of his own conscience unless upon land held in fee simple.

NOTES:

IV. Transfer of Private Lands Natural Resources Second Set_Case Digests

Page | 44

IV. TRANSFER OF PRIVATE LANDS

Continuation…

6. Roman Catholic Administrator of Davao vs LRC (G.R. No. L-8451, December 20, 1957)

THE ROMAN CATHOLIC APOSTOLIC ADMINISTRATOR OF DAVAO, INC., petitioner, vs. THE LAND REGISTRATION

COMMISSION and THE REGISTER OF DEEDS OF DAVAO CITY, respondents.

FACTS:

Mateo Rodis, a Filipino citizen and resident of Davao City, executed a deed of sale of a parcel of land in Davao City in favour of the Roman Catholic Apostolic Administrator of Davao, Inc. (RCAAD Inc.). RCAAD inc. is a corporation sole organized in accordance with Philippine laws, with Msgr. Clovis Thibault, a Canadian national, as actual incumbent (or administrator).

The Register of Deeds (ROD) Davao required the said corporation to submit an affidavit declaring that 60 per cent of the members thereof are Filipino Citizens for purposes of registration. In the required affidavit, RCAADI Inc. said that the totality of the Catholic Population of Davao would become the owner of the property bought to be registered. ROD being doubtful as to the ‘registerability’ of the land, the matter was referred to the Land Registration Commission (LRC). After proper hearing, LRC rendered a resolution holding that the vendee RCAADI Inc. was not qualified to acquire private lands in the Philippines in the absence of conformity to the constitutional requirement of at least 60% of the capital, property, or assets belonging to a Filipino Citizen, as postulated under Sections 1 and 5 of Article 8 of the Constitution. That it is not owned by the Filipino Citizens but rather the only incumbent Msgr. Thibault who is a Canadian Citizen, hence, the denial of registration of the subject property to ROD Davao.

A motion for reconsideration was filed but later denied. Subsequently, an action for mandamus before the SC is filed. RCAAD Inc. alleges that under the Corporation Law as well as the settled jurisprudence on the matter, petitioner is qualified to acquire private lands for the establishment and maintenance of places of worship. That by acquiring private lands, the petitioner is not considered the owner but rather a mere administrator. That there are more than 80% of Filipino Citizens who are Roman Catholic in Davao as per the Bureau of Census’ records which signifies that the mentioned constitutional requirement has been fully satisfied.

On the other hand, respondent averred that though the petitioner does not own the land, yet he has control over the same. With full power to administer, alienate, encumber, and sell or dispose of thereby exercising all rights of ownership on the property. Also, respondents say that a conglomeration of people cannot just be pointed out as the recipient beneficiaries of the Catholic Church. This set-up, according to the petitioners, falls short of trust. ISSUE: WON the petitioner RCAADI Inc. is qualified to own private agricultural lands in the Philippines pursuant to the provisions of Article 8 of the Constitution? YES HELD:

Under the circumstances of the present case, it is safe to state that even before the establishment of the Phil. Commonwealth and of the Republic of the Philippines every corporation sole (special form of corporation usually associated with clergy) then organized and registered had by express provision of law, the Corporation Law or Public Act No. 1459, the necessary power and qualification to purchase in its name private lands located in the territory in which it exercised its functions or ministry. A corporation sole is created not only to administer church properties but also to hold and transmit the same to his successor in said office.

That the Roman Catholic Apostolic Church of the Constitution has no nationality and did not have in mind the religious corporation sole when they provided that 60% of the capital thereof be owned by Filipino Citizens.

Name:

IV. Transfer of Private Lands Natural Resources Second Set_Case Digests

Page | 45

Thus, if this constitutional provision were not intended for corporation sole, it is obvious that this could not be regulated or restricted by said provision.

Corporation soles cannot be considered aliens because they have no nationality at all. In determining, therefore, whether the constitutional provision requiring 60 % Filipino capital is applicable to corporations sole, the nationality of the constituents of the diocese, and not the nationality of the actual incumbent of the parish, must be taken into consideration. In the case at bar, even if the question of nationality be considered, the aforementioned constitutional requirement is fully met and satisfied considering that the corporation sole in question (RCAAD Inc.) is composed of an overwhelming majority of Filipinos.

With these, the LRC resolution which holds that the petitioner corporation sole is not qualified to acquire private land is hereby reversed. The ROD is ordered to register the deed of sale subject of the litigation.

NOTES:

7. Sasosa Vda. De Barsobia vs Cuenco (113 SCRA 547, G.R. No. L-33048 April 16, 1982)

EPIFANIA SARSOSA VDA. DE BARSOBIA and PACITA W. VALLAR, petitioners, vs. VICTORIANO T.

CUENCO, respondent.

FACTS:

Leocadia Balisado sold her coconut land located at Camiguin, Misamis Oriental in favor of spouses Patricio Barsobia and Epifania Sarsosa.

Epifania, then a widow, sold the subject property to a Chinese named Ong King Po. Po sold the land to Victoriano Cuenco, the respondent herein. Cuenco immediately took possession of the said property and harvested the fruits therefrom.

Subsequently, Epifania sold a ½ portion of the land in question to Pacita Vallar. Petitioner claimed that it was not her intention to sell the land to Ong King Po and that she merely signed the document of sale to simply evidence her indebtedness to the latter. Epifania has been in possession ever since except that land sold to Pacita.

Respondent filed a forcible entry case against Epifania before the MTC. However, it was dismissed for lack of jurisdiction and since the question as to ownership is still undetermined. Epifania filed an appeal before Court of Appeals contending that the contract she and Po entered into is void ab intio. ISSUE: WON the contract of sale entered into by Ong King Po and Epifania is valid? HELD:

Firstly, the sale of the land in question by Epifania to Ong King Po is inexistent and void from the beginning because it was a contract executed against the mandatory provision of the 1935 Contitution, which is an expression of public policy to conserve lands for the Filipinos.

However, in the case at bar, the litigated property is now in the hands of a naturalized Filipino. It is no longer owned by a disqualified vendee. Cuenco was a naturalized citizen and therefore he is constitutionally qualified to own the subject property. With this, there is no more basis for Epifania to recover the land as ruled in the case of Vasquez vs. Giap.

IV. Transfer of Private Lands Natural Resources Second Set_Case Digests

Page | 46

Also, Epifania is considered to have slept on her rights for 26 years from 1936 to 1962. By this inaction and inexcusable neglect, she is barred by laches from asserting her claim to the litigated property. Pacita vallar is no liable to damages since she acted in good faith. 8. Republic vs IAC (G.R. No. 74170, July 18, 1989) REPUBLIC OF THE PHILIPPINES, petitioner, vs. INTERMEDIATE APPELLATE COURT, GUILLERMO GONZALVES, ** respondents. FACTS:

The chief question presented in the appeal at bar concerns the validity of a conveyance of residential land to an alien prior to his acquisition of Filipino citizenship by naturalization.

The Trial Court's description of the factual background is largely undisputed. The case principally concerns Chua Kim @ Uy Teng Be, who became a naturalized Filipino citizen, taking his oath as such, on January 7,1977. 1 He was the adopted son of Gregorio Reyes Uy Un.

The case involved three (3) parcels of land, which were among those included in Land Registration Cases Numbered 405 and 14817 of the Court of First Instance of Quezon Province.

These lands were acquired through simplee fee by the father of Chua Kim. Upon his death, the said propery was passed on to him and upon application for a title, the Republic contested the approval by the court of the same reasoning that Chua Kim acquired the said land while he was still an alien. ISSUE: WON a naturalized Filipino may acquire a property during his status when he was not yet naturalized as a Filipino citizen. YES HELD:

It is a fact that the lands in dispute were properly and formally adjudicated by a competent Court to the Spouses Gaspar and to the Spouses Marquez in fee simple, and that the latter had afterwards conveyed said lands to Gregorio Reyes Uy Un, Chua Kim's adopting parent.

Plainly, the conveyance were made before the 1935 Constitution went into effect, i.e., at a time when there was no prohibition against acquisition of private agricultural lands by aliens. Gregorio Reyes Uy Un therefore acquired good title to the lands thus purchased by him.

That since the death of Gregorio Reyes Uy Un, Chua Kim had been in continuous possession of the lands in concept of owner, as the putative heir of his adoptive father, said Gregorio Reyes; this, without protest whatever from any person. It was indeed Chua Kim's being in possession of the property in concept of owner, and his status as adopted son of Gregorio Reyes, that were the factors that caused his involvement in Civil Case No. C-385 of the CFI at Calauag, Quezon, at the instance of the original parties thereto, 22 and his participation in the Compromise Agreement later executed by all parties. As already mentioned, that compromise agreement, approved by judgment rendered on July 29, 1970, 23 implicity recognized Chua Kim's title to the lands in question.

In Sarsosa Vda. de Barsobia v. Cuenco, 113 SCRA 547, for instance, the ruling was as follows:

... The litigated property is now in the hands of a naturalized Filipino. It is no longer owned by a disqualified vendee. Respondent, as a naturalized citizen, was constitutionally qualified to own the subject property. There would be no more public policy to be served in allowing petitioner Epifania to recover the land as it is already in the hands of a qualified person. Applying by analogy the ruling of this Court in Vasquez vs. Giap and Li Seng Giap & Sons (96 Phil. 447 [1955]),

IV. Transfer of Private Lands Natural Resources Second Set_Case Digests

Page | 47

... if the ban on aliens from acquiring not only agricultural but also urban lands, as construed by this Court in the Krivenko case, is to preserve the nation's land for future generations of Filipinos, that aim or purpose would not be thwarted but achieved by making lawful the acquisition of real estate by aliens who became Filipino citizens by naturalization.

PETITION DENIED. CA AFFIRMED.

NOTES:

9. United Church vs Sebastian (159 SCRA 446, G.R. No. L-34672, 30 March 1988) UNITED CHURCH BOARD FOR WORLD MINISTRIES, as owner of BROKENSHIRE MEMORIAL HOSPITAL,petitioner, vs. HON. JUDGE ALEJANDRO E. SEBASTIAN, as Presiding Judge of the CFI of Davao del Norte, and MELENCIO B. DELENA and MAURO GEMENTIZA as Co-Executors of the Testate Estate of DAVID, Jacobson, respondents. FACTS:

David Jacobson was an American citizen who had been a resident of the Philippines for more than thirty years and up to the time of his death in 1970. He left a will in which he "devised and bequeathed" to the Brokenshire Memorial Hospital 60% of his shares of stocks in the Tagdangua Plantation Co., inc. which was incorporated under Philippine law in 1948. This corporation was the registered owner of a tract of land in Pantuhan Davao del Norte, with a total area of about 445 hectares acquired by virtue of a sales patent issued to it in 11953.

The CFI of Davao del Norte, Judge Alejandro E. Sebastian disallowed the above-described legacy on the ground that it was in effect an alienation of private agricultural land in favor of a transferee which was not qualified under the Constitution of 1935. The finding was that the Brokenshire Memorial Hospital was owned by the United Church Board for World Ministries (UCBWM) ,the herein petitioner, which was a non-stock corporation organized in the United States by virtue of a charter granted by the state legislature of Massachussets.

Documents have now made it clear that the United Church for Christ in the Philippines and not the United Church Board for World Ministries was the owner of the Hospital at the time of the execution of the win in 1966 and of the testator's death in 1970. It is also not disputed that such ownership passed to the Brokenshire Memorial Hospital itself upon its incorporation in 1970 when it thus became the proper party-in-interest to claim the property directly devised by Jacobson to it. ISSUE: WON the eventual transfer of ownership from the foreign owned company to the Filipino owned company cured the defect that prevented the transfer of the legacy. YES HELD:

Even on the assumption that the foreign company was really the owner of the Hospital at the time of the effectivity of the will and that the devise was for that reason unenforceable, the defect in the will should be deemed rectified by the subsequent transfer of the property to the Brokenshire Memorial Hospital, Inc. Our consistent ruling on this matter is that if land is invalidly transferred to an alien who subsequently becomes a citizen or transfers it to a ctitizen, the flaw in the original transaction is considered cured and the title of the transferee is rendered valid. PETITION GRANTED.

IV. Transfer of Private Lands Natural Resources Second Set_Case Digests

Page | 48

NOTES:

10. Vasquez vs Giap and Lee Seng Giap (96 Phil 447, G.R. No. L-3676, January 31, 1955)

SOCORRO VASQUEZ, plaintiff-appellant, vs. LI SENG GIAP and LI SENG GIAP & SONS, defendants-appellees.

FACTS:

In 1940, Vasquez sold and transferred to Li Seng Giap, then Chinese citizen, a 423 sq. m. parcel of land with a house in Tondo, Manila, for P14,500.

Subsequently, Li Seng Giap sold and transferred to Li Seng Giap & Sons, Inc., a corporation with shareholdings owned by Chinese citizens, the parcel of land for P14,500.

In 1941, Li Seng Giap was duly naturalized as a Filipino citizen under Certificate of Naturalization No. 515.

Li Seng Giap & Sons, Inc. also became a Filipino corporation, with 96.67% of its stock owned by Filipinos, and duly authorized by its articles of incorporation to own, acquire or dispose of real properties.

In 1948, Vasquez filed an action to rescind the sale of said parcel together with improvements, on the ground that Li Seng Giap was an alien and incapable to own and hold title to lands under the Constitution.

Vasquez argues that if at the time of the conveyance of the real property the Li Seng Giap was incapable of holding title to such real estate, the contract of sale was null or void and may be annulled, and his subsequent naturalization as a Filipino citizen cannot retroact to the date of the conveyance to make it lawful and valid.

ISSUE: WON the transfer to Li Seng Giap and subsequently to the corporation was valid? YES HELD:

YES. The transfer was valid by virtue of Li Seng Giap’s subsequent naturalization and majority of the corporation being owned by Filipinos before the State commenced proceedings to escheat or forfeit subject property which was originally transferred to aliens.

The SC ruled that in sales of real estate to aliens incapable of holding title thereto by virtue of the provisions of the Constitution both vendor and vendee are deemed to have committed the constitutional violation and being thus in pari delicto the courts will not afford protection to either party.

The action is not of rescission because it is not postulated upon any of the grounds provided for in Article 1291 of the old Civil Code and because the action of rescission involves lesion or damage and seeks to repair it.

It is an action for annulment under Chapter VI, Title II, Book II, on nullity of contracts, based on a defect in the contract which invalidates it independently of such lesion or damages.

In the United States the rule is that in a sale of real estate to an alien disqualified to hold title thereto the vendor divests himself of the title to such real estate and has no recourse against the vendee despite the latter's disability on account of alienage to hold title to such real estate and the vendee may hold it against the whole except as against the State.

It is only the State that is entitled by proceedings in the nature of office found to have a forfeiture or escheat declared against the vendee who is incapable of holding title to the real estate sold and conveyed to him

.

However, if the State does not commence such proceedings and in the meantime the alien becomes naturalized citizen the State is deemed to have waived its right to escheat the real property and the title of the alien thereto becomes lawful and valid as of the date of its conveyance or.

The Rule in the United States that in a sale of real estate to an alien disqualified to hold title thereto, the vendor divests himself of the title to such real estate and is not permitted to sue for the annulment of his contract, is also the rule under the Civil Code. Article 1302 of the old Civil Code provides that persons sui juris cannot, however, avail themselves of the incapacity of those with whom they contracted.

However, if the ban on aliens from acquiring not only agricultural but also urban lands, as construed by this Court in the Krivenko case, is to preserve the nation's lands for future generations of Filipinos, that aim or

IV. Transfer of Private Lands Natural Resources Second Set_Case Digests

Page | 49

purpose would not be thwarted but achieved by making lawful the acquisition of real estate by aliens who became Filipino citizens by naturalization.

The title to the parcel of land of the vendee, a naturalized Filipino citizen, being valid that of the domestic corporation to which the parcel of land has been transferred, must also be valid, 96.67% (or more than 60%) of its capital stock being owned by Filipinos.

SC AFFIRMED judgment appealed from.

NOTES:

11. Ong Ching Po vs CA (239 SCRA 341, G.R. Nos. 113472-73, December 20, 1994)

ONG CHING PO, YU SIOK LIAN DAVID ONG and JIMMY ONG, petitioners, vs. COURT OF APPEALS and SOLEDAD

PARIAN, respondents.)

FACTS:

In 1947, Ong Joi Jong sold a parcel of land in San Nicolas to respondent Parian. The sale was evidenced by a notarized Deed of Sale written in English. It was registered with the Register of Deeds in Manila and a TCT was issued in the name of Parian.

Petitioner Ong Ching Po is the brother-in-law of Parian. Po died in 1986.

Parian said that she entrusted the administration of the lot and building to Po when she and her husband settled in Iloilo.

When Parian’s husband died in 1983, she demanded the lot be vacated since she was going to sell it. However, Po and petitioners refused.

Po and petitioners claim that in 1946, Po bought the lot from Ong Joi Jong. A photocopy of the Deed of Sale written in Chinese evidenced said sale. The lot was only named under Parian as a subterfuge since Po, a Chinese, could not own real property in the Philippines.

A consolidated case was filed with petitioners filing an action for reconveyance and damages against Parian and Parian filing for quieting of title.

The CA and RTC ruled in favor of Parian. ISSUE: WON there was a valid transfer from Ong Joi Jong to Parian? YES HELD:

YES. Po was primarily not qualified to acquire and hold public and private lands, being a Chinese citizen.

As stated by petitioners themselves, what is in dispute is not so much as to the validity of the evidences of sale and transfer, but whether the documents are a deed of conveyance in favor of Parian or a resort to or executed as a subterfuge because the real buyer, Po, was an alien and it was agreed upon between Ong Ching Po and his brother, Parian’s husband, that the land be registered in Parian’s name in order to avoid legal complications and to facilitate registration and transfer and that the said title would be transferred by Soledad to Ong Ching Po or his successors-in-interest and that she would be holding the title in trust for him.

SC cannot go along with the claim that Po merely used Parian as a dummy to have the title over the parcel of land registered in her name because being an alien he was disqualified to own real property in the Philippines. To sustain such an outrageous contention would be giving a high premium to a violation of our nationalization laws.

Assuming that the Chinese Deed of Sale was duly executed, still petitioners cannot claim ownership of the disputed lot by virtue thereof.

Section 5, Article XIII of the 1935 Constitution provides that save in cases of hereditary succession, no private agricultural land shall be transferred or assigned except to individuals, corporations, or associations qualified to acquire or hold lands of the public domain in the Philippines.

IV. Transfer of Private Lands Natural Resources Second Set_Case Digests

Page | 50

Section 14, Article XIV of the 1973 Constitution provides that save in cases of hereditary succession, no private land shall be transferred or conveyed except to individuals, corporations, or associations qualified to acquire or hold lands in the public domain.

Section 7, Article XII of the 1987 Constitution provides that save in cases of hereditary succession, no private lands shall be transferred or conveyed except to individuals, corporations, or associations qualified to acquire or hold lands in the public domain.

The capacity to acquire private land is made dependent upon the capacity to acquire or hold lands of the public domain. Private land may be transferred or conveyed only to individuals or entities "qualified to acquire lands of the public domain"

The 1935 Constitution reserved the right to participate in the "disposition, exploitation, development and utilization" of all "lands of the public domain and other natural resources of the Philippines" for Filipino citizens or corporations at least 60% of the capital of which was owned by Filipinos. Aliens, whether individuals or corporations, have been disqualified from acquiring public lands; hence, they have also been disqualified from acquiring private lands.

Po was a Chinese citizen; therefore, he was disqualified from acquiring and owning real property. Assuming that the genuineness and due execution of the Chinese Deed of Sale has been established, the same is null and void, it being contrary to law.

On the other end of the legal spectrum, the deed of sale executed by Ong Joi Jong in favor of Parian is a notarized document.

To remove the mantle of validity bestowed by law on said document, petitioners claim that Parian admitted that she did not pay anything as consideration for the purported sale in her favor. In the same breath, petitioners said that Parian implied in her deposition that it was her husband who paid for the property. It appears, therefore, that the sale was financed out of conjugal funds and that it was her husband who handled the transaction for the purchase of the property. Such transaction is a common practice in Filipino-family affairs.

It is not correct to say that Parian never took possession of the property. Under the law, possession is transferred to the vendee by virtue of the notarized deed of conveyance. Under Article 1498 of the Civil Code of the Philippines, "when the sale is made through a public instrument, the execution thereof shall be equivalent to the delivery of the object of the contract, if from the deed the contrary does not appear or cannot clearly be inferred." Parian never lived in the building constructed on said land because her family had settled in Iloilo.

There is no document showing the establishment of an express trust by Po as trustor and Parian as trustee. The oral testimony to prove the existence of the express trust will not suffice. Under Article 1443 of the Civil Code of the Philippines, "No express trust concerning an immovable or any interest therein may be proved by parole evidence."

Undaunted, petitioners argue that if they cannot prove an express trust in writing, they can prove an implied trust orally. While an implied trust may be proved orally under Art 1457, the evidence must be trustworthy and received by the courts with extreme caution, because such kind of evidence may be easily fabricated It cannot be made to rest on vague and uncertain evidence or on loose, equivocal or indefinite declarations (.

As to the contention of Po that all the tax receipts, tax declaration, rental receipts, Chinese Deed of Sale and transfer certificate of title were in their possession, Parian explained that she and her husband entrusted said lot and building to petitioners when they moved to Iloilo.

SC found, however, that these acts, even if true, are not necessarily reflective of dominion, as even a mere administrator or manager may lawfully perform them pursuant to his appointment or employment. It is markworthy that all the tax receipts were in the name of Parian and her husband. The rental receipts were also in the name of her husband.

Petition was DISMISSED.

NOTES:

IV. Transfer of Private Lands Natural Resources Second Set_Case Digests

Page | 51

12. Frenze vs Catito (G.R. No. 143958, July 11, 2003) ALFRED FRITZ FRENZEL, petitioner, vs. EDERLINA P. CATITO, respondent. FACTS: Petitioner Alfred Fritz Frenzel is an Australian citizen of German descent. Sometime in 1983 he arrived in Sydney and met Ederlina Catito, a Filipina and a native of Bajada, Davao City. Unknown to Alfred, she was married to Klaus Muller when she was in Germany. Thereafter, they move to the Philippines and get married. They bought several properties in Manila and Davao using the money of Alfred. Ederlina went to Germany to file a divorce however Ederlina had not been able to secure a divorce from Klaus. Alfred and Ederlina’s relationship started deteriorating. They lived separately. Alfred filed a Complaint dated October 28, 1985, against Ederlina, with the Regional Trial Court of Quezon City, for recovery of real and personal properties located in Quezon City and Manila. Alfred alleged, inter alia, that Ederlina, without his knowledge and consent, managed to transfer funds from their joint account in HSBC Hong Kong, to her own account with the same bank. In the meantime, on November 7, 1985, Alfred also filed a complaint against Ederlina with the Regional Trial Court, Davao City, for specific performance, declaration of ownership of real and personal properties, sum of money, and damages. Quezon City Trial Court decided in favor of Alfred but the Davao Trial Court is in favor of Ederlina . The trial court ruled that based on documentary evidence, the purchaser of the three parcels of land subject of the complaint was Ederlina. The court further stated that even if Alfred was the buyer of the properties, he had no cause of action against Ederlina for the recovery of the same because as an alien, he was disqualified from acquiring and owning lands in the Philippines. The sale of the three parcels of land to the petitioner was null and void ab initio. Applying the pari delicto doctrine, the petitioner was precluded from recovering the properties from the respondent. CA affirmed the decision of Davao City Court.

ISSUE: WON Alfred has the qualification to own land in the Philippines. HELD: The petition is bereft of merit.

Section 14, Article XIV of the 1973 Constitution provides, as follows: Save in cases of hereditary succession, no private land shall be transferred or conveyed except to individuals, corporations, or associations qualified to acquire or hold lands in the public domain.

Lands of the public domain, which include private lands, may be transferred or conveyed only to individuals or entities qualified to acquire or hold private lands or lands of the public domain. Aliens, whether individuals or corporations, have been disqualified from acquiring lands of the public domain. Hence, they have also been disqualified from acquiring private lands.[3][51] Even if, as claimed by the petitioner, the sales in question were entered into by him as the real vendee, the said transactions are in violation of the Constitution; hence, are null and void ab initio. A contract that violates the Constitution and the law, is null and void and vests no rights and creates no obligations. It produces no legal effect at all. The petitioner, being a party to an illegal contract, cannot come into a court of law and ask to have his illegal objective carried out. One who loses his money or property by knowingly engaging in a contract or transaction which involves his own moral turpitude may not maintain an action for his losses. To him who moves in deliberation and premeditation, the law is unyielding. The law will not aid either party to an illegal contract or agreement; it leaves the parties where it finds them. Under Article 1412 of the New Civil Code, the petitioner cannot have the subject properties deeded to him or allow him to recover the money he had spent for the purchase thereof. Equity as a rule will follow the law and will not permit that to be done indirectly which, because of public policy, cannot be done directly. Where the wrong of one party equals that of the other, the defendant is in the stronger position … it signifies that in such a situation, neither a court of equity nor a court of law will

IV. Transfer of Private Lands Natural Resources Second Set_Case Digests

Page | 52

administer a remedy. The rule is expressed in the maxims: EX DOLO MALO NON ORITUR ACTIO and IN PARI DELICTO POTIOR EST CONDITIO DEFENDENTIS

NOTES:

13. Lee vs Republic (366 SCRA 524, G.R. No. 128195, October 3, 2001) ELIZABETH LEE and PACITA YU LEE, HON. JUDGE JOSE D. ALOVERA,* Presiding Judge, Regional Trial Court, Branch 17, Roxas City, THE REGISTER OF DEEDS OF ROXAS CITY, petitioners, vs. REPUBLIC OF THE PHILIPPINES, represented by THE DIRECTOR OF LANDS AND THE ADMINISTRATOR, LAND REGISTRATION AUTHORITY and THE HON. COURT OF APPEALS,* respondents. FACTS: Sometime in March 1936, The Dinglasan sold to Lee Liong, a Chinese citizen, a parcel of land covered by Original Certificate of Title No. 3389, situated at the corner of Roxas Avenue and Pavia Street, Roxas City. However, in 1948, the former owners filed with the Court of First Instance, Capiz an action against the heirs of Lee Liong for annulment of sale and recovery of land. The plaintiffs assailed the validity of the sale because of the constitutional prohibition against aliens acquiring ownership of private agricultural land, including residential, commercial or industrial land. Rebuffed in the trial court and the Court of Appeals, plaintiffs appealed to the Supreme Court. On June 27, 1956, the Supreme Court upheld the validity of the sale because the parties were in pari delicto. On September 7, 1993, Elizabeth Manuel-Lee and Pacita Yu Lee filed with the Regional Trial Court, Roxas City a petition for reconstitution of the subject land. Petitioners alleged that they were the widows of the deceased Lee Bing Hoo and Lee Bun Ting, who were the heirs of Lee Liong, the owner of the lot. On June 10, 1994, the Regional Trial Court, Roxas City, Branch 17, ordered the reconstitution of the lost or destroyed certificate of title in the name of Lee Liong on the basis of an approved plan and technical description. On the other hand, the Solicitor General contended that the petitioners were not the proper parties in the reconstitution of title, since their predecessor-in-interest Lee Liong did not acquire title to the lot because he was a Chinese citizen and was constitutionally not qualified to own the subject land. ISSUE: WON Lee Liong has the qualification to own land in the Philippines. HELD: The sale of the land in question was consummated sometime in March 1936, during the effectivity of the 1935 Constitution. Under the 1935 Constitution, aliens could not acquire private agricultural lands, save in cases of hereditary succession. Thus, Lee Liong, a Chinese citizen, was disqualified to acquire the land in question. The fact that the Court did not annul the sale of the land to an alien did not validate the transaction, for it was still contrary to the constitutional proscription against aliens acquiring lands of the public or private domain. Although ownership of the land cannot revert to the original sellers, because of the doctrine of pari delicto, the Solicitor General may initiate an action for reversion or escheat of the land to the State, subject to other defenses, as hereafter set forth. While it took the Republic more than sixty years to assert itself, it is not barred from initiating such action. Prescription never lies against the State. In this case, subsequent circumstances militate against escheat proceedings because the land is now in the hands of Filipinos. The original vendee, Lee Liong, has since died and the land has been inherited by his heirs

IV. Transfer of Private Lands Natural Resources Second Set_Case Digests

Page | 53

and subsequently their heirs, petitioners herein. Petitioners are Filipino citizens, a fact the Solicitor General does not dispute. The constitutional proscription on alien ownership of lands of the public or private domain was intended to protect lands from falling in the hands of non-Filipinos. In this case, however, there would be no more public policy violated since the land is in the hands of Filipinos qualified to acquire and own such land. “If land is invalidly transferred to an alien who subsequently becomes a citizen or transfers it to a citizen, the flaw in the original transaction is considered cured and the title of the transferee is rendered valid.” Thus, the subsequent transfer of the property to qualified Filipinos may no longer be impugned on the basis of the invalidity of the initial transfer. The objective of the constitutional provision to keep our lands in Filipino hands has been achieved.

NOTES:

14. Muller vs Muller (500 SCRA 65, G.R. No. 149615, August 29, 2006) IN RE: PETITION FOR SEPARATION OF PROPERTY ELENA BUENAVENTURA MULLER, Petitioner, vs. HELMUT MULLER, Respondent.

FACTS:

Petitioner Elena Buenaventura Muller and respondent Helmut Muller were married in Hamburg, Germany on September 22, 1989. The couple lived in a house in Germany owned by respondent’s parents but decided to move and reside permanently in the Philippines in 1992. By this time, respondent had inherited the house in Germany from his parents which he sold and used the proceeds for the purchase of a parcel of land in Antipolo, Rizal at the cost of P528,000.00 and the construction of a house amounting to P2,300,000.00. The Antipolo property was registered in the name of petitioner under a Transfer Certificate of Title of the Register of Deeds of Marikina, Metro Manila.

Due to incompatibilities and respondent’s alleged womanizing, drinking, and maltreatment, the spouses eventually separated. On September 26, 1994, respondent filed a petition for separation of properties before the Regional Trial Court of Quezon City.

RTC rendered a decision which terminated the regime of absolute community of property between the petitioner and respondent and ordered the equal partition of the personal properties situated in the Philippines. Thus:

o Pursuant to Article 92 of the Family Code, properties acquired by gratuitous title by either spouse during the marriage shall be excluded from the community property. The real property, therefore, inherited by petitioner in Germany is excluded from the absolute community of property of the herein spouses. Necessarily, the proceeds of the sale of said real property as well as the personal properties purchased thereby, belong exclusively to the petitioner.

o The part of that inheritance used by the petitioner for acquiring the house and lot in this country cannot be recovered by the petitioner, its acquisition being a violation of Section 7, Article XII of the Constitution which provides that “save in cases of hereditary succession, no private lands shall be transferred or conveyed except to individuals, corporations or associations qualified to acquire or hold lands of the public domain.” The law will leave the parties in the situation where they are in without prejudice to a voluntary partition by the parties of the said real property.

CA modified RTC’s decision. It held that respondent merely prayed for reimbursement for the purchase of the Antipolo property, and not acquisition or transfer of ownership to him. It also considered petitioner’s ownership over the property in trust for the respondent. As regards the house, the Court of Appeals ruled that there is nothing in the Constitution which prohibits respondent from acquiring the same.

Hence, the instant petition for review.

IV. Transfer of Private Lands Natural Resources Second Set_Case Digests

Page | 54

ISSUE: WON respondent is entitled to reimbursement of the funds used for the acquisition of the Antipolo property. HELD:

Petitioner contends that respondent, being an alien, is disqualified to own private lands in the Philippines; that respondent was aware of the constitutional prohibition but circumvented the same; and that respondent’s purpose for filing an action for separation of property is to obtain exclusive possession, control and disposition of the Antipolo property.

Respondent claims that he is not praying for transfer of ownership of the Antipolo property but merely reimbursement; that the funds paid by him for the said property were in consideration of his marriage to petitioner; that the funds were given to petitioner in trust; and that equity demands that respondent should be reimbursed of his personal funds.

Aliens, whether individuals or corporations, are disqualified from acquiring lands of the public domain. Hence, they are also disqualified from acquiring private lands. The primary purpose of the constitutional provision is the conservation of the national patrimony.

Respondent was aware of the constitutional prohibition and expressly admitted his knowledge thereof to this Court. He declared that he had the Antipolo property titled in the name of petitioner because of the said prohibition. His attempt at subsequently asserting or claiming a right on the said property cannot be sustained.

The Court of Appeals erred in holding that an implied trust was created and resulted by operation of law in view of petitioner’s marriage to respondent. Save for the exception provided in cases of hereditary succession, respondent’s disqualification from owning lands in the Philippines is absolute. Not even an ownership in trust is allowed. To do so would lead to the circumvention of the law.

Thus, in the instant case, respondent cannot seek reimbursement on the ground of equity where it is clear that he willingly and knowingly bought the property despite the constitutional prohibition.

Further, the distinction made between transfer of ownership as opposed to recovery of funds is a futile exercise on respondent’s part. To allow reimbursement would in effect permit respondent to enjoy the fruits of a property which he is not allowed to own. Thus, it is likewise proscribed by law.

NOTES:

15. RP vs Gonzalves (175 SCRA 398, G.R. No. 74170 July 18, 1989) REPUBLIC OF THE PHILIPPINES, petitioner, vs. INTERMEDIATE APPELLATE COURT, GUILLERMO GONZALVES,** respondents. FACTS:

This case involves 3 parcels of land (Lot 1, 2, 549). These were respectively adjudicated in land registration cases with 2 persons: o Lots 1 and 2, to spouses Benigno Manosca and Julia Daguison. o Lots 549, to Gaspar Marquez, married to Marcela Masaganda.

However, no decree of confirmation and registration was entered at the time.

December 30, 1934 – Lots 1 and 2 were sold by the owners, the Manosca Spouses, to Gregorio Reyes Uy Un.

December 27, 1934 – Lot 549 was sold to Gregorio Reyes Uy Un as well.

1946, Gergorio Reyes Uy Un died, and his adopted son Chua Kim (alias Uy Teng Be), who became a naturalized Filipino citizen after taking his oath on Jan. 7, 1977, took possession of the property.

The 3 parcels of land together with several others became a subject of a compromise agreement in a litigation.

IV. Transfer of Private Lands Natural Resources Second Set_Case Digests

Page | 55

In the compromise agreement, it was stated that the opposition parties (Domingo Reyes, Lourdes Abustan, et al) waives any claims and transfers ownership over to Chua Kim. Such agreement was submitted to the Court with approved it.

Chua Kim then filed a petition for issuance of decree of confirmation and registration with the CFI, the same granted his petition.

The RP, through the Solicitor General challenged the corrected of the Order and appealed before the CA. CA affirmed the order of the CFI.

The Republic’s theory is that the conveyances to Chua Kim were made while he was still an alien, at a time when was disqualified to acquire ownership of land in the Philippines (Art. XIII, Sec. 15, 1935 Constitution; Art. XIV, Sec. 14, 1873 Constitution. Therefore, his asserted titles are null and void.

Hence this present petition. ISSUE: WON the conveyance of residential land to an alien prior to his acquisition of Filipino citizenship by naturalization is valid. YES HELD:

It is a fact that lands in dispute were properly and formally adjudicated by a competent Court to the Spouses Gaspar and the Spouses Marquez in fee simple, and that the latter had afterwards conveyed said lands to Gregorio Reyes Uy Un, Chua Kim’s adopting parent, by deeds executed in due form on Dec. 27 and 30, 1934.

Plainly, the conveyances were made before the 1935 Constitution went into effect. At a time when there was no prohibition against acquisition of private agricultural lands by aliens. Gregorio Reyes Uy Un therefore acquired good title to the lands thus purchased by him and his ownership was not at all affected either (1) by the principle subsequently enunciated in the 1935 Constitution that aliens were incapacitated to acquire lands in the country, since that constitutional principle has no respective application, or (2) by his and his successor’s omission to procure the registration of the property prior to the coming into effect of the Constitution.

Furthermore, since the death of G.R. Uy Un, Chua Kim had been in continuous possession of the lands in concept of owner, as the putative heir of his adoptive father, without protest from any person.

Be as it may, the acquisition by Chua Kim of Philippine citizenship should foreclose any further debate regarding the title to the property in controversy.

Petition is DISMISSED.

NOTES:

IV. Transfer of Private Lands + Other Topics Natural Resources Third Set_Case Digests

Page | 56

IV. TRANSFER OF PRIVATE LANDS

Continuation…

16. Yap vs Grajeda (121 SCRA 244, G.R. No. L-31606, March 28, 1983) DONATO REYES YAP and MELITONA MARAVILLAS, petitioners, vs. HON. EZEKIEL S. GRAGEDA, as Judge of the Court of First Instance of Albay and JOSE A. RICO, respondents. FACTS:

Maximino Rico executed a Deed of Absolute Sale over Lot 339 and a portion of Lot 327 in favor of petitioner Yap (who was then a Chinese national). Respondent Jose Rico is the eldest son of Maximino. Petitioner caused the registration of the instrument of sale and cancellation of original certificates of title.

After nearly 15 years from execution of the Deed of Absolute Sale, Yap was admitted as Filipino citizen

and allowed to take his oath of allegiance to the Republic of the Philippines. He was issued a Certificate of Naturalization.

Yap ceded the major portion of Lot 327 in favor of his son Felix Yap, who was also a Filipino citizen

(Filipina mother, naturalization of father). Subsequently, Lourdes Rico (aunt and co-heir of respondent Jose) sold remaining portion of Lot 327 to the petitioner.

CFI declared the sale null and void because it was sold to a Chinese national, inspite the fact that Yap had

been a naturalized Filipino citizen. CFI considered Section 5, Article XIII of the 1935 Constitution that "no private agricultural land shall be transferred or assigned except to individuals, corporations, or associations qualified to acquire or hold lands of the public domain in the Philippines" to be an absolute and unqualified prohibition and, therefore, ruled that a conveyance contrary to it would not be validated nor its void nature altered by the subsequent naturalization of the vendee. ISSUE: Was the sale null and void despite the fact that vendee had been a naturalized Filipino citizen? NO HELD: As stated in the case of Sarosa Vda de Bersabia, “there should be no question that the sale of the land in question… was inexistent and void from the beginning because it was a contract executed against the mandatory provision of the Constitution, which is an expression of public policy to conserve lands for the Filipinos” But the factual set-up has changed. The litigated property is now in the hands of a naturalized Filipino. It is no longer owned by a disqualified vendee. Respondent, as a naturalized citizen, was constitutionally qualified to own the subject property. There would be no more public policy to be served in allowing petitioner Epifania to recover the land as it is already in the hands of a qualified person. Applying by analogy the ruling of this Court in Vasquez vs. Giap and Leng Seng Giap & Sons:

... if the ban on aliens from acquiring not only agricultural but also urban lands, as construed by this Court in the Krivenko case, is to preserve the nation's lands for future generations of Filipinos, that aim or purpose would not be thwarted but achieved by making lawful the acquisition of real estate by aliens who became Filipino citizens by naturalization

Complaint is dismissed. Judgment of CFI is reversed.

NOTES:

Name:

IV. Transfer of Private Lands + Other Topics Natural Resources Third Set_Case Digests

Page | 57

17. Godinez vs Pak Luen (120 SCRA 223, G.R. No. L-36731, January 27, 1983) VICENTE GODINEZ, ET AL., plaintiffs-appellants, vs. FONG PAK LUEN ET AL., defendants, TRINIDAD S. NAVATA, defendant-appellee. FACTS:

The plaintiffs filed this case to recover a parcel of land in Jolo, Sulu sold by their father, now deceased, to Fong Pak Luen, an alien, on the ground that the sale was null and void ab initio since it violates applicable provisions of the Constitution and the Civil Code.

The plaintiffs are heirs of Jose and Martina Godinez. They allege that their parents acquired the subject parcel of land during their marriage. After the death of their mother Martina, Jose sold the land in 1941 without their knowledge, to the defendant Fong Pak Luen, a Chinese citizen. Later, Fong Pak Luen executed a power of attorney in favor of his co-defendant Kwan Pun Ming, also an alien, who conveyed and sold the parcel of land to co-defendant Trinidad S. Navata (a Filipino), who is aware of and with full knowledge that Fong Pak Luen is a Chinese citizen as well as Kwan Pun Ming, who under the law are prohibited and disqualified to acquire real property. The heirs argue that no valid title or interest was conveyed to Navata over said property and so Navata had not acquired anything from said grantor since the TCT issued by the ROD in favor of the alien is null and void ab initio. ISSUE: WON the heirs of a person who sold a parcel of land to an alien in violation of a constitutional prohibition may recover the property if it had, in the meantime, been conveyed to a Filipino citizen qualified to own and possess it. NO HELD: Trinidad Navata, the titled owner of the property is declared the rightful owner by the Court.

There can be no dispute that the sale in 1941 by Jose to Fong Pak Luen, a Chinese citizen residing in Hongkong, was violative of Section 5, Article XIII of the 1935 Constitution. (reference was made to the Krivenko vs.ROD case on the meaning of this consti provision)

Nonetheless, the land sold in 1941 is now in the hands of a Filipino citizen against whom the

constitutional prescription was never intended to apply. UNDERLYING PRINCIPLE

The Court discussed its ruling in Vasquez v. Li Seng Giap and Li Seng Giap & Sons, where the alien vendee later sold the property to a Filipino corporation. The SC in that case dismissed the complaint to rescind the sale of real property to the defendant alien, on the ground that the vendee was an alien and under the Constitution incapable to own and hold title to lands. The Court discussed the rule applied in the U.S.

In the United States the rule is that in a sale of real estate to an alien disqualified to hold title thereto the

vendor divests himself of the title to such real estate and has no recourse against the vendee despite the latter's disability on account of alienage to hold title to such real estate and the vendee may hold it against the whole world except as against the State. It is only the State that is entitled by proceedings in the nature of office found to have a forfeiture or escheat declared against the vendee who is incapable of holding title to the real estate sold and conveyed to him.

HOWEVER, if the State does not commence such proceedings and in the meantime the alien becomes naturalized citizen, the State is deemed to have waived its right to escheat the real property and the title of the alien thereto becomes lawful and valid as of the date of its conveyance or transfer to him. The rule in the United States that in a sale of real estate to an alien disqualified to hold title thereto, the vendor divests himself of the title to such real estate and is not permitted to sue for the annulment Of his Contract, is also the rule under the Civil Code. ... Article 1302 of the old Civil Code provides: ... Persons sui juris cannot, however, avail themselves of the incapacity of those with whom they contracted; ...

IV. Transfer of Private Lands + Other Topics Natural Resources Third Set_Case Digests

Page | 58

So, if the ban on aliens from acquiring not only agricultural but, also urban lands, as construed by this Court in the Krivenko case, is to preserve the nation's land for future generations of Filipinos, that aim or purpose would not be thwarted but achieved by making lawful the acquisition of real estate by aliens who became Filipino citizens by naturalization. The title to the parcel of land of the vendee, a naturalized Filipino citizen, being valid that of the domestic corporation to which the parcel of land has been transferred, must also be valid, 96.67 per cent of its capital stock being owned by Filipinos. Discussion on prescription of actions (if Atty. Coronel would ask)

Navata alleges that the cause of action of the heirs has been barred by the statute of limitations as the alleged document of sale executed by Jose in 1941, conveyed the property to Fong Pak Luen as a result of which a title was issued to Fong Pak Luen. She further alleged that under Article 1144 (1) of the Civil Code, an action based upon a written contract must be brought within 10 years from the time the right of action accrues; that the right of action accrued in 1941 but the complaint was filed only in 1966, beyond the 10 year period provided for by law.

The Court ruled here that under the Constitution “aliens may not acquire private or agricultural lands, including residential lands". This statement is a declaration of an imperative constitutional policy. Consequently, prescription may never be invoked to defend that which the Constitution prohibits. Insofar as the vendee is concerned, prescription is unavailing. But neither can the vendor or his heirs rely on an argument based on imprescriptibility because the land sold in 1941 is now in the hands of a Filipino citizen against whom the constitutional prescription was never intended to apply.

From the fact that prescription may not be used to defend a contract which the Constitution prohibits, it

does not necessarily follow that the appellants may be allowed to recover the property sold to an alien. Fong Pak Luen, the disqualified alien vendee later sold the same property to Trinidad S. Navata, a Filipino citizen qualified to acquire real property.

NOTES:

18. Halili vs CA (83 SCRA 633, G.R. No. 113539, March 12, 1998) CELSO R. HALILI and ARTHUR R. HALILI, petitioners, vs. COURT OF APPEALS, HELEN MEYERS GUZMAN, DAVID REY GUZMAN and EMILIANO CATANIAG, respondents. FACTS:

Simeon de Guzman, an American citizen, died sometime in 1968, leaving real properties in the Philippines. His forced heirs were his widow, Helen Meyers Guzman, and his son, David Rey Guzman, both of whom are also American citizens. In 1989, Helen executed a deed of quitclaim, assigning, transferring and conveying to David Rey all her rights, titles and interests in and over six parcels of land which the two of them inherited from Simeon.

Among the said parcels of land, is the 6,695 sqm land in Sta. Maria Bulacan (which is now subject of the litigation). In 1991, David Rey sold the aforesaid parcel of land to Emiliano Cataniag (Pinoy).

Petitioners (“the Halilis”), who are owners of the adjoining lot, filed a complaint before the RTC, (1)

questioning the constitutionality and validity of the two conveyances — between Helen and David Rey, and between the latter and Emiliano— and (2) claiming ownership thereto based on their right of legal redemption under Art. 1621 of the Civil Code.

IV. Transfer of Private Lands + Other Topics Natural Resources Third Set_Case Digests

Page | 59

RTC dismissed the complaint and said that on the first issue, Helen's waiver of her inheritance in favor of her son was not contrary to the constitutional prohibition against the sale of land to an alien, since the purpose of the waiver was simply to authorize David Rey to dispose of their properties in accordance with the Constitution and the laws of the Philippines, and not to subvert them. On the second issue, it held that the subject land was urban; hence, the Halilis had no reason to invoke their right of redemption under Art. 1621 of the CC. CA affirmed. ISSUE: WON the sale to Emiliano Catiniag was valid. - YES HELD:

The SC ruled that Helen Guzman's deed of quitclaim — in which she assigned, transferred and conveyed to David Rey all her rights, titles and interests over the property she had inherited from her husband — collided with the Constitution, Article XII, Section 7 of which provides:

Sec. 7. Save in cases of hereditary succession, no private lands xxx qualified to acquire or hold lands of the public domain.

The deed of quitclaim is therefore not valid.

Nonetheless, the sale to Catiniag was valid because according to jurisprudence, if land is invalidly transferred to an alien who subsequently becomes a citizen or transfers it to a citizen, the flaw in the original transaction is considered CURED and the title of the transferee is rendered valid. This is the effect of a subsequent sale by the disqualified alien vendee to a qualified Filipino citizen. So, since the disputed land is now owned by Cataniag, a Filipino citizen, the prior invalid transfer can no longer be assailed. The objective of the constitutional provision — to keep our land in Filipino hands — has been served. BASIS: Cited jurisprudence such as: De Castro vs. Tan (which has similar facts to the present case), United Church Board of Word Ministries vs. Sebastian, Sarsosa vs. Cuenco, Godinez vs. Pak Luen among others. RATIONALE FOR SUCH PRINCIPLE: If the ban on aliens from acquiring not only agricultural but also urban lands, as construed by the SC in the Krivenko case, is to preserve the nation's lands for future generations of Filipinos, that aim or purpose would not be thwarted but achieved by making lawful the acquisition of real estate by aliens who became Filipino citizens by naturalization. Accordingly, since the disputed land is now owned by a Filipino citizen (Emiliano), the prior invalid transfer can no longer be assailed. The objective of the constitutional provision — to keep our land in Filipino hands — has been served.

NOTES:

19. Lee vs Director of Lands (366 SCRA 524, G.R. No. 128195 October 3, 2001) ELIZABETH LEE and PACITA YU LEE, HON. JUDGE JOSE D. ALOVERA, Presiding Judge, Regional Trial Court, Branch 17, Roxas City, THE REGISTER OF DEEDS OF ROXAS CITY, petitioners, vs. REPUBLIC OF THE PHILIPPINES, represented by THE DIRECTOR OF LANDS AND THE ADMINISTRATOR, LAND REGISTRATION AUTHORITY and THE HON. COURT OF APPEALS, respondents. FACTS:

Sometime in 1936, the Dinglasans sold to Lee Liong, a Chinese citizen a parcel of land with an approximate area of 1,631 square meters, designated as Lot 398 and covered by Original Certificate of Title No. 3389, situated at the corner of Roxas Avenue and Pavia Street, Roxas city.

IV. Transfer of Private Lands + Other Topics Natural Resources Third Set_Case Digests

Page | 60

In 1948, the Dinglasans filed a an action for annulment of sale and recovery of land against the heirs of Lee Liong on the ground of the Constitutional prohibition against aliens acquiring ownership of private agricultural land, including residential, commercial or industrial land. It was dismissed by the trial Court and the CA. The Supreme Court also dismissed the same applying the principle of pari delicto.

Another action was filed in the CFI by the same vendors Dinglasan but the same failed because of res judicata.

In 1993, Pacita Yu Lee and Elizabeth manuel Lee (heirs of Lee Liong) filed with the RTC a petition for reconstitution of the Lot 398 which was granted. This time, the Solicitor General filed with the CA a petition for annulment of judgment conontending that the petitioners were not the proper parties in the reconstitution of title, since their predecessor-in-interest Lee Liong did not acquire title to the lot because he was a Chinese citizen and was constitutionally not qualified to own the subject land. CA granted the petition and denied the MFR filed by the Lees.

ISSUE: Is the Solicitor General estopped from assailing the reconstitution after failing to object during the reconstitution proceedings before the trial Court despite due notice? NO HELD:

Even if the Court did not initially annul the sale because of pari delicto, it did not validate the acquisition of Lee Liong of the subject land.. “In sales of real estate to aliens incapable of holding title thereto by virtue of the provisions of the Constitution both the vendor and the vendee are deemed to have committed the constitutional violation and being thus in pari delicto the courts will not afford protection to either party.” If anything, the effect is that they (sellers) are not anymore the proper party to assail the sale . This time, the proper party to assail the sale is the Solicitor General. This was what was done in this case when the Solicitor General initiated an action for annulment of judgment of reconstitution of title. While it took the Republic more than sixty years to assert itself, it is not barred from initiating such action. Prescription never lies against the State.

Although ownership of the land cannot revert to the original sellers, because of the doctrine of pari delicto, the Solicitor General may initiate an action for reversion or escheat of the land to the State, subject to other defenses, as hereafter set forth.

In this case, subsequent circumstances militate against escheat proceedings because the land is now in the hands of Filipinos. The original vendee, Lee Liong, has since died and the land has been inherited by his heirs and subsequently their heirs, petitioners herein. Petitioners are Filipino citizens, a fact the Solicitor General does not dispute.

The constitutional proscription on alien ownership of lands of the public or private domain was intended to protect lands from falling in the hands of non-Filipinos. In this case, however, there would be no more public policy violated since the land is in the hands of Filipinos qualified to acquire and own such land. “If land is invalidly transferred to an alien who subsequently becomes a citizen or transfers it to a citizen, the flaw in the original transaction is considered cured and the title of the transferee is rendered valid.” Thus, the subsequent transfer of the property to qualified Filipinos may no longer be impugned on the basis of the invalidity of the initial transfer The objective of the constitutional provision to keep our lands in Filipino hands has been achieved. Incidentally, it must be mentioned that reconstitution of the original certificate of title must be based on an owner’s duplicate, secondary evidence thereof, or other valid sources of the title to be reconstituted. In this case, reconstitution was based on the plan and technical description approved by the Land Registration Authority. This renders the order of reconstitution void for lack of factual support. A judgment with absolutely nothing to support it is void.

As earlier mentioned, a reconstitution of title is the re-issuance of a new certificate of title lost or destroyed in its original form and condition. It does not pass upon the ownership of the land covered by the lost or destroyed title. Any change in the ownership of the property must be the subject of a separate suit. Thus, although

IV. Transfer of Private Lands + Other Topics Natural Resources Third Set_Case Digests

Page | 61

petitioners are in possession of the land, a separate proceeding is necessary to thresh out the issue of ownership of the land.

Reconstitution was disallowed for lack of factual support. There is a need to adjudicate the facts of the subsequent transfer from Lee Liong to the heirs who are Filipino citizens.

NOTES:

B.P. No. 185

1. Republic vs CA (G.R. No. 108998, August 24, 1994, 235 SCRA 567) REPUBLIC OF THE PHILIPPINES, petitioner, vs. THE COURT OF APPEALS AND SPOUSES MARIO B. LAPIÑA AND FLOR DE VEGA, respondents. FACTS:

On June 17, 1978, respondent spouses bought Lots, as their residence with a total area of 91.77 sq. m. situated in San Pablo City, from one Cristeta Dazo Belen. At the time of the purchase, respondent spouses where then natural-born Filipino citizens.

On February 5, 1987, the spouses filed an application for registration of title of the two (2) parcels of land before the RTC of San Pablo City. This time, however, they were no longer Filipino citizens and have opted to embrace Canadian citizenship through naturalization.

RTC rendered a decision confirming the title of the Spouses. Republic filed an opposition to the decision of RTC. On appeal, CA affirmed the decision of RTC.

Republic submits that private respondents have not acquired proprietary rights over the subject properties before they acquired Canadian citizenship through naturalization to justify the registration thereof in their favor. It maintains that even privately owned unregistered lands are presumed to be public lands under the principle that lands of whatever classification belong to the State under the Regalian doctrine. Thus, before the issuance of the certificate of title, the occupant is not in the juridical sense the true owner of the land since it still pertains to the State. Petitioner further argued that it is only when the court adjudicates the land to the applicant for confirmation of title would the land become privately owned land, for in the same proceeding, the court may declare it public land, depending on the evidence. ISSUE: WON a foreign national apply for registration of title over a parcel of land which he acquired by purchase while still a citizen of the Philippines, from a vendor who has complied with the requirements for registration under the Public Land Act (CA 141)? YES HELD: Since the spouses’ predecessor in interest has been in open, continuous and exclusive possession for at least 30 years of alienable public land, such possession ipso jure converts the same to private property (Recall ruling of Director of Lands v. IAC). This means that occupation and cultivation for more than 30 years by an applicant and his predecessors-in-interest, vest title on such applicant so as to segregate the land from the mass of public.

IV. Transfer of Private Lands + Other Topics Natural Resources Third Set_Case Digests

Page | 57

What is important is that private respondents were formerly natural-born citizens of the Philippines, and as transferees of a private land, they could apply for registration in accordance with the mandate of Section 8, Article XII of the Constitution. In the case at bar, the spouses were undoubtedly natural-born Filipino citizens at the time of the acquisition of the properties and by virtue thereof, acquired vested rights thereon, tacking in the process, the possession in the concept of owner and the prescribed period of time held by their predecessors-in-interest under the Public Land Act. But what should not be missed in the disposition of this case is the fact that the Constitution itself allows private respondents to register the contested parcels of land in their favor. Sections 7 and 8 of Article XII of the Constitution contain the following pertinent provisions, to wit:

Sec. 8. Notwithstanding the provisions of Section 7 of this Article, a natural-born citizen of the Philippines who has lost his Philippine citizenship may be a transferee of private lands, subject to limitations provided by law. (Emphasis supplied)

Section 8, Article XII of the 1987 Constitution above quoted is similar to Section 15, Article XIV of the then 1973 Constitution which reads:

Sec. 15. Notwithstanding the provisions of Section 14 of this Article, a natural-born citizen of the Philippines who has lost his citizenship may be a transferee of private land, for use by him as his residence, as the Batasang Pambansa may provide.

Pursuant thereto, Batas Pambansa Blg. 185 was passed into law, the relevant provision of which provides:

Sec. 2. Any natural-born citizen of the Philippines who has lost his Philippine citizenship and who has the legal capacity to enter into a contract under Philippine laws may be a transferee of a private land up to a maximum area of one thousand square meters, in the case of urban land, or one hectare in the case of rural land, to be used by him as his residence. In the case of married couples, one of them may avail of the privilege herein granted; Provided, That if both shall avail of the same, the total area acquired shall not exceed the maximum herein fixed. In case the transferee already owns urban or rural lands for residential purposes, he shall still be entitled to be a transferee of an additional urban or rural lands for residential purposes which, when added to those already owned by him, shall not exceed the maximum areas herein authorized.

From the adoption of the 1987 Constitution up to the present, no other law has been passed by the

legislature on the same subject. Thus, what governs the disposition of private lands in favor of a natural-born Filipino citizen who has lost his Philippine citizenship remains to be BP 185.

Even if private respondents were already Canadian citizens at the time they applied for registration of the properties in question, said properties as discussed above were already private lands; consequently, there could be no legal impediment for the registration thereof by respondents in view of what the Constitution ordains. The parcels of land sought to be registered no longer form part of the public domain. They are already private in character since private respondents' predecessors-in-interest have been in open, continuous and exclusive possession and occupation thereof under claim of ownership prior to June 12, 1945 or since 1937. The law provides that a natural-born citizen of the Philippines who has lost his Philippine citizenship may be a transferee of a private land up to a maximum area of 1,000 sq.m., if urban, or one (1) hectare in case of rural land, to be used by him as his residence (BP 185).

It is undisputed that private respondents, as vendees of a private land, were natural-born citizens of the Philippines. For the purpose of transfer and/or acquisition of a parcel of residential land, it is not significant

IV. Transfer of Private Lands + Other Topics Natural Resources Third Set_Case Digests

Page | 58

whether private respondents are no longer Filipino citizens at the time they purchased or registered the parcels of land in question. What is important is that private respondents were formerly natural-born citizens of the Philippines, and as transferees of a private land, they could apply for registration in accordance with the mandate of Section 8, Article XII of the Constitution. Considering that private respondents were able to prove the requisite period and character of possession of their predecessors-in-interest over the subject lots, their application for registration of title must perforce be approved.

NOTES:

Remedies 1. Reliosa vs Gaw Chee Hun (93 Phil 827, GR No. L-1411, September 29, 1953) DIONISIO RELLOSA, Petitioner, vs. GAW CHEE HUN, Respondent. FACTS:

Herein petitioner Dionisio Rellosa sold a parcel of land together with the house erected thereon to Gaw Chee Gun. Although vendor (Rellosa) has issued a contract of lease, he remained in possession of the property for the reason that as the vendee (Gaw Chee Hun) is a Chinese citizen, has to obtain first the approval of the Japanese Military Administration. Because the approval of the Japanese authorities has not been obtained and because it is contrary to Art XIII of the Constitution, said sale is considered void. Thus, the vendor-petitioner instituted an action seeking the annulment of the sale and the lease covering the subject property, and that said sale and lease should be declared null and void. Defendant however contends that the sale referred to in the complaint is an absolute and unconditional, thus in every respect valid and binding between the parties. Furthermore, Gaw Chee Hun contends that petitioner Rellosa is guilty of estoppel, as he has already executed a deed of lease over the property thus recognizing the title of respondent-vendee to that property. ISSUES: (1) W/N the sale entered into between the petitioner and respondent is void? YES (2) W/N the petitioner can have the sale declared null and void? NO HELD: The Court held that the sale entered into between the petitioner and the respondent is void for the law governing such sale is the 1943 Constitution of the Philippines as the transaction was executed on 1944. In the 1943 Constitution, specifically Art VIII Sec 5, it provides that “no private agricultural land shall be transferred or assigned except to individuals, corporations, or associations qualified to acquire or hold lands of the public domain in the Philippines." The Court further held that “private agricultural land” also includes residential lands, and that is included in the private or public agricultural lands that cannot be acquired by aliens as provided in the Constitution. However, on the issue whether the petitioner can validly have the sale declared null and void, the Court held in the negative. In ruling so, the Court said that the petitioner cannot invoke the Constitution for the purpose of recovering the lands that they have voluntarily parted with because of their guilty knowledge that what they were doing was in violation of the Constitution.

Thus, the Court held that although the sale in question is null and void, the plaintiff is barred from taking the present action under the principle of pari delicto.

IV. Transfer of Private Lands + Other Topics Natural Resources Third Set_Case Digests

Page | 59

NOTES:

2. Republic vs CA

(172 SCRA 1) Case is very difficult to find in the internet or in SCRA. Mali ata ang SCRA # na na-cite

NOTES:

3. Baguio vs Republic (301 SCRA 450, G.R. No. 119682 January 21, 1999) FRANCISCO BAGUIO, petitioner, vs. REPUBLIC OF THE PHILIPPINES, RICARDO T. MICHAEL, in his capacity as Heir-Successor of WILLIAM MICHAEL, SR., and as President of MICHAEL SLIPWAYS, INC., and COURT OF APPEALS, respondent. FACTS:

Since 1963, William Michael had been occupying and developeing a parcel foreshore land in Cebu. Originally, Michael was only granted a lease, but he has since developed the area (even reclaiming land) and even started running a drydock business on the property, Michael eventually applied for a miscellaneous sales application.

In 1978, a free patent for the same land occupied by Michael was granted to Fransisco Baguio. Baguio, in securing the free patent claimed that the land was agircultural and alienable, and that no one was occupying the same. By virtue of this patent, an OCT in his favor was registered. Baguio started to demand payment of rentals from Michael, and opposed Michael’s miscellaneous sales application.

William Michael in turn protested the issuance by the Bureau of Lands of a free patent to petitioner. He claimed that he had been in actual possession of the land since 1963 and that he had introduced substantial improvements thereon. On July 20, 1992, the trial court rendered a decision canceling the free patent and the certificate of title of petitioner Baguio, ordering the reversion of the land to the public domain, and declaring private respondent Michael the true and lawful occupant of the land. The trial court ruled that the false statements made by petitioner Baguio in his application for free patent had the effect of ipso facto canceling the free patent and the title of petitioner. ISSUE: Is Baguio entitled to the land? NO. HELD: Effect of patent:

It is true that, once a patent is registered and the corresponding certificate of title is issued, the land covered by them ceases to be part of the public domain and becomes private property, and the Torrens Title issued pursuant to the patent becomes indefeasible upon the expiration of one year from the date of issuance of such patent.

IV. Transfer of Private Lands + Other Topics Natural Resources Third Set_Case Digests

Page | 60

Remedy in case of fraud However, even after the lapse of one year, the State may still bring an action under §101 of Commonwealth Act No. 141 for the reversion to the public domain of land which has been fraudulently granted to private individuals.

Such action is not barred by prescription, and this is settled law.

Indeed, the indefeasibility of a certificate of title cannot be invoked by one who procured the title by

means of fraud. Public policy demands that one who obtains title to public land through fraud should not be

allowed to benefit therefrom.

In his free patent application, petitioner declared under oath that the land in question was an agricultural land not claimed or occupied by any other person; that he had continuously possessed and occupied it; and that he had introduced improvements thereon. These declarations constitute fraud and misrepresentation. Petitioner is guilty of making false statements in his application for a free patent thus justifying the annulment of his title.

As already stated, the indefeasibility of a title does not attach to titles secured by fraud and misrepresentation. The registration of a patent under the Torrens System merely confirms the registrant's title. It does not vest title where there is none because registration under this system is not a mode of acquiring ownership. Further remedies in Sec 91 of CA 141:

The statements made in the application shall be considered as essential conditions and parts of any concession, title, or permit issued on the basis of such application, and any false statement therein or omission of facts altering, changing or modifying the consideration of the facts set forth in such statements, and any subsequent modification, alteration, or change of the material facts set forth in the application shall ipso facto produce the cancellation of the concession, title or permit granted. It shall be the duty of the Director of Lands, from time to time and whenever he may deem it advisable; to make the necessary investigations for the purpose of ascertaining whether the material facts set out in the application are true, or whether they continue to exist and are maintained and preserved in good faith, and for the purpose of such investigation, the Director of Lands is hereby empowered to issue subpuenas and subpoenas duces tecum and, if necessary, to obtain compulsory process from the courts. In every investigation made in accordance with this section, the existence of bad faith, fraud, concealment, or fraudulent and illegal modification of essential facts shall be presumed if the grantee or possessor of the land shall refuse or fail to obey a subpoena or subpoena duces tecum lawfully issued by the Director of Land or his authorized delegates or agents, or shall refuse or fail to give direct and specific answers to pertinent questions, and on the basis of such presumption, an order of cancellation may issue without further proceedings.

NOTES:

4. Urquiaga vs CA (301 SCRA 738, G.R. No. 127833, January 22, 1999) TEODORO URQUIAGA and MARIA AGUIRRE, petitioners, vs. THE COURT OF APPEALS, VICENTE CASES and ANITA CRISOSTOMO, respondents. FACTS:

Vicente Cases and Anita Crisostomo, respondent spouses, are the registered owners of a parcel of land located in Sicayab, Dipolog City. They acquired such land pursuant to a Sales Patent in 1969. Later, they caused the subdivision of a land which resulted to the issuance of to Transfer Certificate of Title (TCT).

The workers of the petitioner once entered one of the said lots and gathered Nipa palms therefrom. Respondents called their attention regarding the intrusion but the latter allegedly assured them that such incident would not be repeated.

IV. Transfer of Private Lands + Other Topics Natural Resources Third Set_Case Digests

Page | 61

However, petitioner’s workers entered the said premises on several occasions this time claiming that the

property is owned by Urquiaga, that their parents owned and possessed the said piece of land since time immemorial and that the respondents ascribed actual fraud in its acquisition. With this, respondents applied for the quieting of title, recovery of possession, and damages before the RTC.

In 1992, RTC declared lawful owners the respondent spouses. Petitioners appealed before the CA. CA affirmed the decision of RTC with some modifications in the award of damages. Hence, this petition in SC. ISSUE: WON the petitioners should be declared lawful owners? NO HELD: No, SC affirmed the decision of CA.

Evidence shows that during the Cadastral proceedings, the original subject lot has been declared a public land and was owned by the Commonwealth of the Philippines before a sales patent was awarded to the respondents. That it is free from any private claim, hence, open to disposition according to law. Therefore, the petitioners’ contention that the subject land was possessed by their predecessors – in –interest since time immemorial is completely untenable.

Even assuming that fraud attended the sales application, it is only the State which may institute reversion proceedings considering that it is a public land at the time of the sales application. In other words, petitioners have no standing to question the validity of the respondents’ title.

NOTES:

5. Tankiko vs Cesar (302 SCRA 559, G.R. No. 131277, February 2, 1999) FACTS:

Respondents are applicants for sales patents of a public land. They question the issuance of an OCT in favor of the petitioners contending that they are the actual occupants of the said land and had been paying the taxes religiously. ISSUE: WON the private respondents may be deemed the proper parties to initiate the present suit. NO. HELD:

Petitioners insist that respondents had no legal capacity to file the Complaint, because they were not the owners of the land but mere applicants for sales patent thereon. Therefore, petitioners argue that respondents, not being the real parties in interest, have no legal standing to institute the Complaint in the trial court.

We agree with petitioners. The Court is not persuaded that the circumstances of this case justify the exercise of equity jurisdiction that would allow a suit to be filed by one who is not a real party in interest.

First, equity is invoked only when the plaintiff, on the basis of the action filed and the relief sought, has a clear right that he seeks to enforce, or that would obviously be violated if the action filed were to be dismissed for lack of standing. In the present case, respondents have no clear enforceable right, since their claim over the land in question is merely inchoate and uncertain. Admitting that they are only applicants for sales patents on the land, they are not and they do not even claim to be owners thereof. In fact, there is no certainty that their applications

IV. Transfer of Private Lands + Other Topics Natural Resources Third Set_Case Digests

Page | 62

would even be ruled upon favorably, considering that some of the applications have been pending for more than ten years already.

Second, it is evident that respondents are not the real parties in interest. Because they admit that they are not the owners of the land but mere applicants for sales patents thereon, it is daylight clear that the land is public in character and that it should revert to the State. This being the case, Section 101 of the Public Land Act categorically declares that only the government may institute an action to recover ownership of a public land.

Verily, the Court stressed that “[i]f the suit is not brought in the name of or against the real party in interest, a motion to dismiss may be filed on the ground that the complaint states no cause of action.” In fact, a final judgment may be invalidated if the real parties in interest are not included.

In the present dispute, only the State can file a suit for reconveyance of a public land. Therefore, not being the owners of the land but mere applicants for sales patents thereon, respondents have no personality to file the suit. Neither will they be directly affected by the judgment in such suit.

Indeed, “for all its conceded merits, equity is available only in the absence of law and not as its replacement. Equity is described as justice without legality, which simply means that it cannot supplant although it may, as often happens, supplement the law.” To grant respondents standing in the present case is to go against the express language of the law. Equity cannot give them this privilege. Equity can only supplement the law, not supplant it.

NOTES:

6. Republic vs CA (301 SCRA 366, G.R. No. 116111, January 21, 1999) REPUBLIC OF THE PHILIPPINES, (Represented by the Acting Commissioner of Land Registration), petitioner, vs. COURT OF APPEALS, Spouses CATALINO SANTOS and THELMA BARRERO SANTOS, ST. JUDE'S ENTERPRISES, INC., Spouses DOMINGO CALAGUIAN and FELICIDAD CALAGUIAN, VIRGINIA DELA FUENTE and LUCY MADAYA, respondents. FACTS:

Defendant St. Jude’s Enterprises, Inc. is the registered owner of a parcel of land (with an area of 40, 623 sq m) included in a subdivision plan located in Caloocan City. Defendant St. Jude’s subdivided the lot under the subdivision plan and as a result thereof, it was later found to have expanded and enlarged from its original area of 40,523 square meters to 42,044 square meters or an increase of 1,421 square meters. This expansion or increase in area was confirmed by the land Registration Commission.

Subsequently, St. Jude’s Enterprises, Inc. sold some areas of its land to defendant Spouses Catalino Santos

and Thelma Barreto Santos, to defendant Sps. Domingo Calaguian and Felicidad de Jesus, to defendant Virginia dela Fuente, and to defendant Lucy Madaya.

The Solicitor General filed an action for the annulment and cancellation of the certificate titles principally on the ground that said Certificates of Title were issued on the strength of [a] null and void subdivision plan which expanded the original area in the name of St. Jude's Enterprises, Inc. from 40,623 square meters to 42,044 square meters upon its subdivision.

The defendants interposed their defense that they acquired the lots in question in good faith from their former owner, defendant St. Jude's Enterprises, Inc. and for value and that the titles issued to the said defendants

IV. Transfer of Private Lands + Other Topics Natural Resources Third Set_Case Digests

Page | 63

were rendered incontrovetible, conclusive and indefeasible after one year from the date of the issuance of the titles by the Register of Deeds of Caloocan City.

St. Jude’s also answered that that the cause of action of plaintiff is barred by prior judgment; that the subdivision plan submitted having been approved by the LRC, the government is now in estoppel to question the approved subdivision plan; and the plaintiff's allegation that the area of the subdivision increased by 1,421 square meters is without any basis in fact and in law. RTC: dismissed the complaint finding no fraud on the part of St. Jude’s when it submitted its subdivision plan to the LRC. It also concluded that the government was already “in estoppel to question the approved subdivision plan.” CA: Affirmed the decision of the trial court. It berated petitioner for bringing the suit only after nineteen (19) years had passed since the issuance of St. Jude’s title and the approval of the subdivision plan. ISSUE: WON the government is estopped from questioning the approved subdivision plan which expanded the areas covered by the transfer certificates of title in question. YES. HELD:

The general rule is that the State cannot be put in estoppel by the mistakes or error of its officials or agents. However, like all general rules, this is also subject to exceptions, viz.:

“Estoppels against the public are little favored. They should not be invoked except in rate and unusual circumstances, and may not be invoked where they would operate to defeat the effective operation of a policy adopted to protect the public. They must be applied with circumspection and should be applied only in those special cases where the interests of justice clearly require it. Nevertheless, the government must not be allowed to deal dishonorably or capriciously with its citizens, and must not play an ignoble part or do a shabby thing; and subject to limitations x x x, the doctrine of equitable estoppel may be invoked against public authorities as well as against private individuals.”

In the case at bar, for nearly twenty years (starting from the issuance of St. Jude’s titles in 1966 up to

the filing of the Complaint in 1985), petitioner failed to correct and recover the alleged increase in the land area of St. Jude. Its prolonged inaction strongly militates against its cause, as it is tantamount to laches, which means “the failure or neglect, for an unreasonable and unexplained length of time, to do that which by exercising due diligence could or should have been done earlier; it is negligence or omission to assert a right within a reasonable time, warranting a presumption that the party entitled to assert it either has abandoned it or declined to assert it.”

Significantly, the other private respondents -- Spouses Santos, Spouses Calaguian, Dela Fuente and

Madaya -- bought such “expanded” lots in good faith, relying on the clean certificates of St. Jude, which had no notice of any flaw in them either. It is only fair and reasonable to apply the equitable principle of estoppel by laches against the government to avoid an injustice to the innocent purchasers for value.

NOTES:

IV. Transfer of Private Lands + Other Topics Natural Resources Third Set_Case Digests

Page | 64

Archipelagic Baseline of the Philippines (R.A. No. 9522) 1. Magallona vs Ermita (G.R. No. 187167, July 16, 2011)

NATURE: Original action for the writs of certiorari and prohibition assailing the constitutionality of RA 9522 which adjusts the country’s archipelagic baselines and classifying the baseline regime of nearby territories. FACTS:

In 1961, Congress passed RA 3046, demarcating the maritime baselines of the Philippines as an archipelagic State.

The law followed the Convention on the Territorial Sea and the Contiguous Zone (UNCLOS), codifying the sovereign right of State parties over their territorial sea, although the breadth was left undetermined.

In 2009, Congress amended RA 3046 by enacting RA 9522 to comply with the terms of (UNCLOS III) or the United Nations Convention on the Law of the Sea. It was ratified on February 27, 1984.

UNCLOS III prescribed the water-land ratio, length and contour of baselines of archipelagic States and set the deadline for the filing of application for the extended continental shelf.

To comply with the requirements, RA 9522 shortened one baseline, optimized the location of some basepoints around the Philippine archipelago and classified adjacent territories, namely, the Kalayaan Island Group (KIG) and the Scarborough Shoal, as “regimes of islands” whose islands generate their own applicable maritime zones.

Petitioners are professors of law, law students and a legislator, in their capacities as citizens, taxpayers and legislators, assail the constitutionality of RA 9522 on two grounds:

a. RA 9522 reduces Philippine maritime territory and the reach of the Philippine state’s sovereign power in violation of Art 1 of the 1987 Constitution, embodying the terms of the Treaty of Paris and ancillary treaties.

b. RA 9522 opens the country’s waters landward of the baselines to maritime passage by all vessels and aircrafts, undermining Philippine sovereignty and national security, contravening the country’s nuclear-free policy, and damaging marine resources, in violation of relevant constitutional provisions.

In addition, petitioners contend that RA 9522’s treatment of the KIG as "regime of islands" not only results in the loss of a large maritime area but also prejudices the livelihood of subsistence fishermen.

Respondents defended RA 9522 as the country’s compliance with the terms of UNCLOS III, preserving Philippine territory over the KIG or Scarborough Shoal. Respondents add that RA 9522 does not undermine the country’s security, environment and economic interests or relinquish the Philippines’ claim over Sabah. They also question the locus standi of petitioners.

ISSUE: WON RA 9522 is unconstitutional? NO HELD:

The petitioners have locus standi as citizens with constitutionally sufficient interest. However, they don’t have locus standi as legislators and taxpayers because the petitioner alleges neither infringement of legislative prerogative nor misuse of public funds.

The writs of certiorari and prohibition are proper remedies to test the constitutionality of statues. It is the

judiciary’s exercise of its constitutional power of judicial review. 1. RA 9522 is Constitutional

As for the constitutionality of RA 9522, it is constitutional because it is a statutory tool to demarcate the country’s maritime zones and continental shelf under UNCLOS III, not to delineate Philippine territory.

UNCLOS III has nothing to do with the acquisition or loss of territory. It is a multilateral treaty regulating, among others, sea-use rights over maritime zones (i.e., the territorial waters [12 nautical miles from the

IV. Transfer of Private Lands + Other Topics Natural Resources Third Set_Case Digests

Page | 65

baselines], contiguous zone [24 nautical miles from the baselines], exclusive economic zone [200 nautical miles from the baselines]), and continental shelves that UNCLOS III delimits.

UNCLOS III was the culmination of decades-long negotiations among United Nations members to codify norms regulating the conduct of States in the world’s oceans and submarine areas, recognizing coastal and archipelagic States’ graduated authority over a limited span of waters and submarine lands along their coasts.

On the other hand, baselines laws such as RA 9522 are enacted by UNCLOS III States parties to mark-out specific basepoints along their coasts from which baselines are drawn, either straight or contoured, to serve as geographic starting points to measure the breadth of the maritime zones and continental shelf. Article 48 of UNCLOS III on archipelagic States like ours could not be any clearer:

Article 48. Measurement of the breadth of the territorial sea, the contiguous zone, the exclusive economic zone and the continental shelf. – The breadth of the territorial sea, the contiguous zone, the exclusive economic zone and the continental shelf shall be measured from archipelagic baselines drawn in accordance with article 47.

Even under petitioners’ theory that the Philippine territory embraces the islands and all the waters within the rectangular area delimited in the Treaty of Paris, the baselines of the Philippines would still have to be drawn in accordance with RA 9522 because this is the only way to draw the baselines in conformity with UNCLOS III. The baselines cannot be drawn from the boundaries or other portions of the rectangular area delineated in the Treaty of Paris, but from the "outermost islands and drying reefs of the archipelago.

UNCLOS III and its ancillary baselines laws play no role in the acquisition, enlargement or, as petitioners claim, diminution of territory. Under traditional international law typology, States acquire or conversely, lose territory through occupation, accretion, cession and prescription, not by executing multilateral treaties on the regulations of sea-use rights or enacting statutes to comply with the treaty’s terms to delimit maritime zones and continental shelves. Territorial claims to land features are outside UNCLOS III, and are instead governed by the rules on general international law.

2. RA 9522 Use of the Framework of Regime of Islands Actually Increases the Country’s Total Maritime Space

Petitioners next submit that RA 9522’s use of UNCLOS III’s regime of islands framework to draw the baselines, and to measure the breadth of the applicable maritime zones of the KIG, "weakens our territorial claim" over that area.

Petitioners add that the KIG’s and Scarborough Shoal’s exclusion from the Philippine archipelagic baselines results in the loss of "about 15,000 square nautical miles of territorial waters," prejudicing the livelihood of subsistence fishermen.

The configuration of the baselines drawn under RA 3046 and RA 9522 shows that RA 9522 merely followed the basepoints mapped by RA 3046, save for at least nine basepoints that RA 9522 skipped to optimize the location of basepoints and adjust the length of one baseline (and thus comply with UNCLOS III’s limitation on the maximum length of baselines).

Under RA 3046, as under RA 9522, the KIG and the Scarborough Shoal lie outside of the baselines drawn around the Philippine archipelago. This undeniable cartographic fact takes the wind out of petitioners’ argument branding RA 9522 as a statutory renunciation of the Philippines’ claim over the KIG, assuming that baselines are relevant for this purpose.

On the contrary, RA 9522, by optimizing the location of basepoints, increased the Philippines’ total maritime space by 145,216 square nautical miles.

Further, petitioners’ argument that the KIG now lies outside Philippine territory because the baselines that RA 9522 draws do not enclose the KIG is negated by RA 9522 itself. Section 2 of the law commits to text the Philippines’ continued claim of sovereignty and jurisdiction over the KIG and the Scarborough Shoal:

Sec 2. The baselines in the following areas over which the Philippines likewise exercises sovereignty and jurisdiction shall be determined as "Regime of Islands" under the Republic of the Philippines consistent with Article 121 of the United Nations Convention on the Law of the Sea (UNCLOS):

IV. Transfer of Private Lands + Other Topics Natural Resources Third Set_Case Digests

Page | 66

a) The Kalayaan Island Group as constituted under Presidential Decree No. 1596 and b) Bajo de Masinloc, also known as Scarborough Shoal.

Although the Philippines has consistently claimed sovereignty over the KIG and the Scarborough Shoal for several decades, these outlying areas are located at an appreciable distance from the nearest shoreline of the Philippine archipelago, such that any straight baseline loped around them from the nearest basepoint will inevitably "depart to an appreciable extent from the general configuration of the archipelago."

The principal sponsor of RA 9522 in the Senate, Senator Miriam Defensor-Santiago, took pains to emphasize the foregoing during the Senate deliberations:

“ What we call the Kalayaan Island Group or Spratlys and the Scarborough Shoal are outside our archipelagic baseline because if we put them inside our baselines we might be accused of violating the provision of international law which states: "The drawing of such baseline shall not depart to any appreciable extent from the general configuration of the archipelago." So sa loob ng ating baseline, dapat magkalapit ang mga islands. Dahil malayo ang Scarborough Shoal, hindi natin masasabing malapit sila sa atin although we are still allowed by international law to claim them as our own.”

Hence, far from surrendering the Philippines’ claim over the KIG and the Scarborough Shoal, Congress’ decision to classify the KIG and the Scarborough Shoal as "‘Regimes of Islands’ under the Republic of the Philippines consistent with Article 121" of UNCLOS III manifests the Philippine State’s responsible observance of its pacta sunt servanda obligation under UNCLOS III.

3. Statutory Claim Over Sabah Under RA 5446 Retained

Petitioners’ argument for the invalidity of RA 9522 for its failure to textualize the Philippines’ claim over Sabah in North Borneo is also untenable.

Section 2 of RA 5446, which RA 9522 did not repeal, keeps open the door for drawing the baselines of Sabah:

Section 2. The definition of the baselines of the territorial sea of the Philippine Archipelago as provided in this Act is without prejudice to the delineation of the baselines of the territorial sea around the territory of Sabah, situated in North Borneo, over which the Republic of the Philippines has acquired dominion and sovereignty.

4. UNCLOS III and RA 9522 Not Incompatible with the Constitution’s Delineation of Internal Waters

As their final argument against the validity of RA 9522, petitioners contend that the law unconstitutionally "converts" internal waters into archipelagic waters, hence subjecting these waters to the right of innocent and sea lanes passage under UNCLOS III, including overflight.

Petitioners extrapolate that these passage rights indubitably expose Philippine internal waters to nuclear and maritime pollution hazards.

Whether referred to as Philippine "internal waters" under Article I of the Constitution or as "archipelagic waters" under UNCLOS III, the Philippines exercises sovereignty over the body of water lying landward of the baselines, including the air space over it and the submarine areas underneath. UNCLOS III affirms:

Article 49. Legal status of archipelagic waters, of the air space over archipelagic waters and of their bed and subsoil. – 1. The sovereignty of an archipelagic State extends to the waters enclosed by the archipelagic baselines drawn in accordance with article 47, described as archipelagic waters, regardless of their depth or distance from the coast. 2. This sovereignty extends to the air space over the archipelagic waters, as well as to their bed and subsoil, and the resources contained therein. 4. The regime of archipelagic sea lanes passage established in this Part shall not in other respects affect the status of the archipelagic waters, including the sea lanes, or the exercise by the archipelagic State of its sovereignty over such waters and their air space, bed and subsoil, and the resources contained therein.

IV. Transfer of Private Lands + Other Topics Natural Resources Third Set_Case Digests

Page | 67

The fact of sovereignty, however, does not preclude the operation of municipal and international law norms subjecting the territorial sea or archipelagic waters to necessary, if not marginal, burdens in the interest of maintaining unimpeded, expeditious international navigation, consistent with the international law principle of freedom of navigation. Thus, domestically, the political branches of the Philippine government, in the competent discharge of their constitutional powers, may pass legislation designating routes within the archipelagic waters to regulate innocent and sea lanes passage.

In the absence of municipal legislation, international law norms, now codified in UNCLOS III, operate to grant innocent passage rights over the territorial sea or archipelagic waters, subject to the treaty’s limitations and conditions for their exercise.

Significantly, the right of innocent passage is a customary international law, thus automatically incorporated in the corpus of Philippine law. No modern State can validly invoke its sovereignty to absolutely forbid innocent passage that is exercised in accordance with customary international law without risking retaliatory measures from the international community.

More importantly, the recognition of archipelagic States’ archipelago and the waters enclosed by their baselines as one cohesive entity prevents the treatment of their islands as separate islands under UNCLOS III. Separate islands generate their own maritime zones, placing the waters between islands separated by more than 24 nautical miles beyond the States’ territorial sovereignty, subjecting these waters to the rights of other States under UNCLOS III.

In fact, the demarcation of the baselines enables the Philippines to delimit its exclusive economic zone, reserving solely to the Philippines the exploitation of all living and non-living resources within such zone. Such a maritime delineation binds the international community since the delineation is in strict observance of UNCLOS III. If the maritime delineation is contrary to UNCLOS III, the international community will of course reject it and will refuse to be bound by it.

UNCLOS III favors States with a long coastline like the Philippines. UNCLOS III creates a sui generis maritime space – the exclusive economic zone – in waters previously part of the high seas. UNCLOS III grants new rights to coastal States to exclusively exploit the resources found within this zone up to 200 nautical miles. UNCLOS III, however, preserves the traditional freedom of navigation of other States that attached to this zone beyond the territorial sea before UNCLOS III.

RA 9522 and the Philippines’ Maritime Zones

The prerogative of choosing to pass RA 9522 belongs to Congress, not to this Court. Moreover, the luxury of choosing this option comes at a very steep price.

Absent an UNCLOS III compliant baselines law, an archipelagic State like the Philippines will find itself devoid of internationally acceptable baselines from where the breadth of its maritime zones and continental shelf is measured.

This is recipe for a two-fronted disaster: first, it sends an open invitation to the seafaring powers to freely enter and exploit the resources in the waters and submarine areas around our archipelago; and second, it weakens the country’s case in any international dispute over Philippine maritime space. These are consequences Congress wisely avoided.

The enactment of UNCLOS III compliant baselines law for the Philippine archipelago and adjacent areas, as embodied in RA 9522, allows an internationally-recognized delimitation of the breadth of the Philippines’ maritime zones and continental shelf. RA 9522 is therefore a most vital step on the part of the Philippines in safeguarding its maritime zones, consistent with the Constitution and our national interest.

Petition DISMISSED.

NOTES:

IV. Transfer of Private Lands + Other Topics Natural Resources Third Set_Case Digests

Page | 68

IPRA (R.A. No. 8371) 1. Cruz vs Secretary (347 SCRA 128, G.R. No. 135385, December 6, 2000) ISAGANI CRUZ and CESAR EUROPA, petitioners, vs. SECRETARY OF ENVIRONMENT AND NATURAL RESOURCES, SECRETARY OF BUDGET AND MANAGEMENT and CHAIRMAN and COMMISSIONERS OF THE NATIONAL COMMISSION ON INDIGENOUS PEOPLES,respondents. FACTS: Cruz, a noted constitutionalist, assailed the validity of the RA 8371 or the Indigenous People’s Rights Act on the ground that the law amount to an unlawful deprivation of the State’s ownership over lands of the public domain as well as minerals and other natural resources therein, in violation of the regalian doctrine embodied in Section 2, Article XII of the Constitution. The IPRA law basically enumerates the rights of the indigenous peoples over ancestral domains which may include natural resources. Cruz et al contend that, by providing for an all-encompassing definition of “ancestral domains” and “ancestral lands” which might even include private lands found within said areas, Sections 3(a) and 3(b) of said law violate the rights of private landowners. ISSUE: WON IPRA law is unconstitutional. NO HELD: The SC deliberated upon the matter. After deliberation they voted and reached a 7-7 vote. They deliberated again and the same result transpired. Since there was no majority vote, Cruz’s petition was dismissed and the IPRA law was sustained. Hence, ancestral domains may include public domain – somehow against the regalian doctrine. NOTES: ( just in case sir will ask about ani :)

Petitioners assail the constitutionality of the following provisions of the IPRA and its Implementing Rules on the ground that they amount to an unlawful deprivation of the State’s ownership over lands of the public domain as well as minerals and other natural resources therein, in violation of the regalian doctrine embodied in Section 2, Article XII of the Constitution:

“(1) Section 3(a) which defines the extent and coverage of ancestral domains, and Section 3(b) which, in

turn, defines ancestral lands; “(2) Section 5, in relation to section 3(a), which provides that ancestral domains including inalienable

public lands, bodies of water, mineral and other resources found within ancestral domains are private but community property of the indigenous peoples;

“(3) Section 6 in relation to section 3(a) and 3(b) which defines the composition of ancestral domains and

ancestral lands; “(4) Section 7 which recognizes and enumerates the rights of the indigenous peoples over the ancestral

domains; “(5) Section 8 which recognizes and enumerates the rights of the indigenous peoples over the ancestral

lands;

“(6) Section 57 which provides for priority rights of the indigenous peoples in the harvesting, extraction, development or exploration of minerals and other natural resources within the areas claimed to be their ancestral domains, and the right to enter into agreements with nonindigenous peoples for the development and utilization of natural resources therein for a period not exceeding 25 years, renewable for not more than 25 years; and

“(7) Section 58 which gives the indigenous peoples the responsibility to maintain, develop, protect and

conserve the ancestral domains and portions thereof which are found to be necessary for critical watersheds, mangroves, wildlife sanctuaries, wilderness, protected areas, forest cover or reforestation.”

IV. Transfer of Private Lands + Other Topics Natural Resources Third Set_Case Digests

Page | 69

Petitioners also content that, by providing for an all-encompassing definition of “ancestral domains” and “ancestral lands” which might even include private lands found within said areas, Sections 3(a) and 3(b) violate the rights of private landowners.

In addition, petitioners question the provisions of the IPRA defining the powers and jurisdiction of the

NCIP and making customary law applicable to the settlement of disputes involving ancestral domains and ancestral lands on the ground that these provisions violate the due process clause of the Constitution. These provisions are: “(1) sections 51 to 53 and 59 which detail the process of delineation and recognition of ancestral domains and which vest on the NCIP the sole authority to delineate ancestral domains and ancestral lands; “(2) Section 52[i] which provides that upon certification by the NCIP that a particular area is an ancestral domain and upon notification to the following officials, namely, the Secretary of Environment and Natural Resources, Secretary of Interior and Local Governments, Secretary of Justice and Commissioner of the National Development Corporation, the jurisdiction of said officials over said area terminates; “(3) Section 63 which provides the customary law, traditions and practices of indigenous peoples shall be applied first with respect to property rights, claims of ownership, hereditary succession and settlement of land disputes, and that any doubt or ambiguity in the interpretation thereof shall be resolved in favor of the indigenous peoples; “(4) Section 65 which states that customary laws and practices shall be used to resolve disputes involving indigenous peoples; and “(5) Section 66 which vests on the NCIP the jurisdiction over all claims and disputes involving rights of the indigenous peoples.”

Finally, petitioners assail the validity of Rule VII, Part II, Section 1 of the NCIP Administrative Order No. 1, series of 1998, which provides that “the administrative relationship of the NCIP to the Office of the President is characterized as a lateral but autonomous relationship for purposes of policy and program coordination.” They contend that said Rule infringes upon the President’s power of control over executive departments under Section 17, Article VII of the Constitution.

NOTES:

Mining Act (R.A. No. 7942) 1. Miners vs Factoran (240 SCRA 100, G.R. No. 98332 January 16, 1995) MINERS ASSOCIATION OF THE PHILIPPINES, INC., petitioner, vs. HON. FULGENCIO S. FACTORAN, JR., Secretary of Environment and Natural Resources, and JOEL D. MUYCO, Director of Mines and Geosciences Bureau, respondents. FACTS

The instant case was filed by petitioner questioning the validity and constitutionality of the administrative orders issued by the respondents which sprouted from the respective promulgation by the then president Aquino of Executive Orders for the implementation of the new 1987 constitution regarding mining applications and agreements and for guidelines during the transitory period.

The change was introduced by Article XII, Section 2 of the 1987 Constitution governing the system of exploration, development and utilization of the country’s natural resources. No longer is the utilization of inalienable lands of public domain through “license, concession or lease” under the 1935 and 1973 Constitutions allowed under the 1987 Constitution.

IV. Transfer of Private Lands + Other Topics Natural Resources Third Set_Case Digests

Page | 70

Petitioner alleges that among others, the administrative orders and ultimately the executive orders are unconstitutional because, among others, violates the non-impairment of contract provision since the said orders pre-terminates existing mining agreements and automatically converts them into production-sharing agreements.

ISSUE WON the administrative orders are unconstitutional? NO. HELD: This argument is untenable.

The adoption of the concept of jura regalia that all natural resources are owned by the State embodied in the 1935, 1973 and 1987 Constitutions, as well as the recognition of the importance of the country's natural resources, not only for national economic development, but also for its security and national defense, ushered in the adoption of the constitutional policy of "full control and supervision by the State" in the exploration, development and utilization of the country's natural resources. The options open to the State are through direct undertaking or by entering into co-production, joint venture; or production-sharing agreements, or by entering into agreement with foreign-owned corporations for large-scale exploration, development and utilization.

Upon the effectivity of the 1987 Constitution on February 2, 1987, the State assumed a more dynamic role in the exploration, development and utilization of the natural resources of the country. Article XII, Section 2 of the said Charter explicitly ordains that the exploration, development and utilization of natural resources shall be under the full control and supervision of the State. Consonant therewith, the exploration, development and utilization of natural resources may be undertaken by means of direct act of the State, or it may opt to enter into co-production, joint venture, or production-sharing agreements, or it may enter into agreements with foreign-owned corporations involving either technical or financial assistance for large-scale exploration, development, and utilization of minerals, petroleum, and other mineral oils according to the general terms and conditions provided by law, based on real contributions to the economic growth and general welfare of the country.

The economic policy on the exploration, development and utilization of the country's natural resources under Article XII, Section 2 of the 1987 Constitution could not be any clearer. As enunciated in Article XII, Section 1 of the 1987 Constitution, the exploration, development and utilization of natural resources under the new system mandated in Section 2, is geared towards a more equitable distribution of opportunities, income, and wealth; a sustained increase in the amount of goods and services produced by the nation for the benefit of the people; and an expanding productivity as the key to raising the quality of life for all, especially the underprivileged.

The economic policy on the exploration, development and utilization of the country's natural resources under Article XII, Section 2 of the 1987 Constitution could not be any clearer. As enunciated in Article XII, Section 1 of the 1987 Constitution, the exploration, development and utilization of natural resources under the new system mandated in Section 2, is geared towards a more equitable distribution of opportunities, income, and wealth; a sustained increase in the amount of goods and services produced by the nation for the benefit of the people; and an expanding productivity as the key to raising the quality of life for all, especially the underprivileged.

The exploration, development and utilization of the country's natural resources are matters vital to the public interest and the general welfare of the people. Accordingly, the State, in the exercise of its police power in this regard, may not be precluded by the constitutional restriction on non-impairment of contract from altering, modifying and amending the mining leases or agreements granted under Executive Orders. Police Power, being co-extensive with the necessities of the case and the demands of public interest; extends to all the vital public needs.

NOTES:

IV. Transfer of Private Lands + Other Topics Natural Resources Third Set_Case Digests

Page | 71

2. La Bugal-b’laan vs Ramos (445 SCRA 1, G.R. No. 127882, December 1, 2004) FACTS:

On July 25, 1987, then President Corazon C. Aquino issued Executive Order (E.O.) No. 2796 authorizing the DENR Secretary to accept, consider and evaluate proposals from foreign-owned corporations or foreign investors for contracts or agreements involving either technical or financial assistance for large-scale exploration, development, and utilization of minerals, which, upon appropriate recommendation of the Secretary, the President may execute with the foreign proponent.

On March 3, 1995, then President Fidel V. Ramos approved R.A. No. 7942 (The Philippine Mining Act of

1995) to "govern the exploration, development, utilization and processing of all mineral resources." R.A. No. 7942 defines the modes of mineral agreements for mining operations, outlines the procedure for their filing and approval, assignment/transfer and withdrawal, and fixes their terms. Similar provisions govern financial or technical assistance agreements.

On April 9, 1995, 30 days following its publication on March 10, 1995 in Malaya and Manila Times, two newspapers of general circulation, R.A. No. 7942 took effect. Shortly before the effectivity of R.A. No. 7942, however, or on March 30, 1995, the President entered into an FTAA with WMCP covering 99,387 hectares of land in South Cotabato, Sultan Kudarat, Davao del Sur and North Cotabato.

On August 15, 1995, then DENR Secretary Victor O. Ramos issued DENR Administrative Order (DAO) No.

95-23, s. 1995, otherwise known as the Implementing Rules and Regulations of R.A. No. 7942. This was later repealed by DAO No. 96-40, s. 1996 which was adopted on December 20, 1996.

On January 10, 1997, counsels for petitioners sent a letter to the DENR Secretary demanding that the

DENR stop the implementation of R.A. No. 7942 and DAO No. 96-40, giving the DENR fifteen days from receipt to act thereon. The DENR, however, has yet to respond or act on petitioners' letter.

Petitioners claim that the DENR Secretary acted without or in excess of jurisdiction. They pray that the

Court issue an order: (a) Permanently enjoining respondents from acting on any application for Financial or Technical Assistance Agreements; (b) Declaring the Philippine Mining Act of 1995 or Republic Act No. 7942 as unconstitutional and null and void; (c) Declaring the Implementing Rules and Regulations of the Philippine Mining Act contained in DENR Administrative Order No. 96-40 and all other similar administrative issuances as unconstitutional and null and void; and (d) Cancelling the Financial and Technical Assistance Agreement issued to Western Mining Philippines, Inc. as unconstitutional, illegal and null and void.

ISSUE (1):Whether RA 7942 and its Implementing Rules enable the government to exercise that degree of control sufficient to direct and regulate the conduct of affairs of individual enterprises and restrain undesirable activities. YES. RULING (1):

Petitioners charge that RA 7942, as well as its Implementing Rules and Regulations, makes it possible for FTAA contracts to cede full control and management of mining enterprises over to fully foreign-owned corporations, with the result that the State is allegedly reduced to a passive regulator dependent on submitted plans and reports, with weak review and audit powers. The State does not supposedly act as the owner of the natural resources for and on behalf of the Filipino people; it practically has little effective say in the decisions made by the enterprise. Petitioners then conclude that the law, the implementing regulations, and the WMCP FTAA cede “beneficial ownership” of the mineral resources to the foreign contractor.

IV. Transfer of Private Lands + Other Topics Natural Resources Third Set_Case Digests

Page | 72

A careful scrutiny of the provisions of RA 7942 and its Implementing Rules belies petitioners’ claims. Paraphrasing the Constitution, Section 4 of the statute clearly affirms the State’s control thus:

“Sec. 4. Ownership of Mineral Resources. – Mineral resources are owned by the State and the exploration, development, utilization and processing thereof shall be under its full control and supervision. The State may directly undertake such activities or it may enter into mineral agreements with contractors. “The State shall recognize and protect the rights of the indigenous cultural communities to their ancestral lands as provided for by the Constitution.”

The aforequoted provision is substantively reiterated in Section 2 of DAO 96-40 as follows: “Sec. 2. Declaration of Policy. All mineral resources in public and private lands within the territory and exclusive economic zone of the Republic of the Philippines are owned by the State. It shall be the responsibility of the State to promote their rational exploration, development, utilization and conservation through the combined efforts of the Government and private sector in order to enhance national growth in a way that effectively safeguards the environment and protects the rights of affected communities.” In other words, the FTAA contractor is not free to do whatever it pleases and get away with it; on the

contrary, it will have to follow the government line if it wants to stay in the enterprise. Ineluctably then, RA 7942 and DAO 96-40 vest in the government more than a sufficient degree of control and supervision over the conduct of mining operations.

ISSUE (2): WON Sec. 3 (aq) of RA 7942 is unconstitutional for allowing foreign-owned companies to hold the exploration permits. NO. RULING (2):

An objection has been expressed that Section 3(aq) of RA 7942 -- which allows a foreign contractor to apply for and hold an exploration permit -- is unconstitutional. The reasoning is that Section 2 of Article XII of the Constitution does not allow foreign-owned corporations to undertake mining operations directly. They may act only as contractors of the State under an FTAA; and the State, as the party directly undertaking exploitation of its natural resources, must hold through the government all exploration permits and similar authorizations. Hence, Section 3(aq), in permitting foreign-owned corporations to hold exploration permits, is unconstitutional.

The objection, however, is not well-founded. While the Constitution mandates the State to exercise full control and supervision over the exploitation of mineral resources, nowhere does it require the government to hold all exploration permits and similar authorizations. In fact, there is no prohibition at all against foreign or local corporations or contractors holding exploration permits.

The exploration permit serves a practical and legitimate purpose in that it protects the interests and preserves the rights of the exploration permit grantee (the would-be contractor) -- foreign or local -- during the period of time that it is spending heavily on exploration works, without yet being able to earn revenues to recoup any of its investments and expenditures. Minus this permit and the protection it affords, the exploration works and expenditures may end up benefiting only claim-jumpers. Such a possibility tends to discourage investors and contractors. ISSUE (3): WON our mineral resources are given away for free by RA 7942. NO. RULING (3): Foreign contractors do not just waltz into town one day and leave the next, taking away mineral resources without paying anything.

They need to expend a great deal more of their funds for facilities, equipment and supplies, fuel, salaries

of local labor and technical staff, and other operating expenses. In the meantime, they also have to pay taxes, duties, fees, and royalties. All told, the exploration, pre-feasibility, feasibility, development and construction phases together add up to as many as eleven years. The contractors have to continually shell out funds for the

IV. Transfer of Private Lands + Other Topics Natural Resources Third Set_Case Digests

Page | 73

duration of over a decade, before they can commence commercial production from which they would eventually derive revenues. All that money translates into a lot of “pump-priming” for the local economy.

Granted that the contractors are allowed subsequently to recover their pre-operating expenses, still, that

eventuality will happen only after they shall have first put out the cash and fueled the economy. Moreover, in the process of recouping their investments and costs, the foreign contractors do not actually pull out the money from the economy. Rather, they recover or recoup their investments out of actual commercial production by not paying a portion of the basic government share corresponding to national taxes, along with the additional government share, for a period of not more than five years counted from the commencement of commercial production. It must be noted that there can be no recovery without commencing actual commercial production. In the meantime that the contractors are recouping costs, they need to continue operating; in order to do so, they have to disburse money to meet their various needs. In short, money is continually infused into the economy.

The foregoing discussion should serve to rid us of the mistaken belief that, since the foreign contractors are allowed to recover their investments and costs, the end result is that they practically get the minerals for free, which leaves the Filipino people none the better for it.

NOTES: